You are on page 1of 197
H. E. Hpogos SAQAYA NO OBLIER @HSHKE aperencrao «Hayuas Moons .E.lrodov Problems in General Physics Mir Publishers Moscow Sacietommam | Prd the Urn of Saket Scat Repbie Ise sesomned © Isbrencry capa. Fama reg © Bogan crnsatln, Mie Publier, 158 PREFACE ‘This book of problems is intended as a textbook for student a higher educational isitutions studying advanced course in physics. Besides, because ofthe great number of simple problems my be wed by students studying a general couse in pis. ‘The book contains about 1900 problems With hints fr solving the most complicated ones For students’ convenience cach chapter opens with time-saving summary of the principal formulas Tor the felevant area of physics AS 8 fale the formulas are sven without detaled explanations sce sti ‘Sent, ating solving s problem, i assumed to know the meaming of the ‘guaniis appearing ia the formulas. Explanatory notes are only given ‘those cases when misunderstanding may arise "All che formulas inthe text and answers are in SI ster, except in Part Si where the Gaussian system Is used. Quantitative dia and Answers ave presented in accordance wih the Fle of spproximation and rumerial accuracy. "The main physcal constants and tales are summarised at the end of the book “The Perec System of Elements sprinted at the frontend set and the Table of Elementary Particles at the back shes ofthe book. In the present etn, same misprints are corrected, ad a number of problems are substituted by new ones, or Ue quantity daa in them are change refined (1.273, 1 36h, 2.189, 3249, 8.97, 4.198 and sh, In conelusion, the author wants to express his deep gratitude tocol leagues from MIPAI and to readers who sent thet remarks on some prob- Jems helping thereby to improve the Book LE. Fradov CONTENTS: rls é SFE sin Saving ‘te Foes aed Rotation i Hee PART ONE. PIIYSICAL FUNDAMENTALS OF MECHANICS LE Me"Pinsaménial equation’ of Dyndiics : 15 Eek Giacaet obey Mowat il Ani 414, Univeral" Grasitiion, = ° 2 1 bpm ESSN as EE RIG Baan aft se ni ER Rate eke PANT TWO. THERMODYNAMICS AND MOLECULAR PHYSICS 2. Eqn af the On Sn Prone Ee Bae alten ete a cg : BE Rete tpn Cele Beaman Lad tals ay wa 25 RTM Lt aint Ee $5 ise ta : Eh Has rao 25 ENE monn PANT THREE, BLECERODYNAMICS u Ge tod Duco in tn Ble ad 34 Bone Contes enn LE Steal indecen Mian Ei ‘37 Mion of Charged Parties Becca agile Faas PART FOUN, OSCILLATIONS AND WAVES “Mechanica Oxilaine = = =» Ss Maa title : EE BSR Wt ai CE Bhtromgmeie Waves Radiata PABT FIVE. OFTKE see ec ete eee Ef, Photometry and Giuswteeal Opts 3 Rleraction of ay : 55 Dinemion and. Abert of Light SE Qi onng Seles : 5 Thermal Nadtaos. Quanta Nite of Lani PART SIX. ATOMIC AND NUCLEAR PHYSICS . Elsnenary Putte APPENDICES. {Baie Trio Formaan E Sine Punetion Vlog, 5 ee tac Vales Expgoeaua Fonetine ovr) Cai ad A Some Data on Verto ne Dineen Tol ‘Therma “expansion “Cost Eisen Tele Gis Constante Scan Barats gs wn Sls MagurieSosnptbites of Par: iaiipeie ‘Rafnctive talons i Plame Henle Pin ulin A bnog Abeorpiog Hage ts nrc fig hte A. Sef aval Qunts ‘The Basic ermlaset Hloeteodynanies tn ik aid oot 90, Putdetental” Gone > Wine Pie of Pare Senger Pues B # 6 me Be Et x ns ey as s = a A FEW HINTS FOR SOLVING THE PROBLEMS 4, Fics ofall, look through the ta probloas cannot be solved without them. Besides, the refarence di {uoted in the tables will make your work easier and save your time "2 Begin the problem hy recognising its meaning and its formula- tion: Make sure that the data given age sufflent for salving the froblem. Missing date con be found tn the tables in the Append, Wherever posse, draw diagram elucidating the essence of the problem it many tare this toplies both the search fora sltin End the solution itl 'f, Solve each problem, as a rule, in the general form, that ig in a Ielier notations so thag the quantity sought will be expressed in ths a the en dln A satin i he gna fom Dorticuarly valuable since It makos clea the roationship between Efe sought quantity end the given data. What moro, an answer ob tainod inthe general form allows one to make a fairy accurate Judge- ment on the correctness af the solution isl (see the next item), ‘i. Having obtained the solution In the goneral form. check Uo ste it ihe the sight dimensions, The weong dimensions fv an obvious indieation of 9 ong solution. If possible, investigate the beliaviour Gf tg solution jn some extreme special eases For example, whatever the form of the expression for the previtetional fore betwoon two tnlenl! bodies it imust turn into the well-known law of gravitational Interaction of mass polats asthe distance betwee the boos increases Otherwiso, Ht com he immediately infeed that tho solution is wrong, ‘5, When starting eafeulations, remember that the numerical values of pitysicat quantities axe alsvays known only approtimstly. fore, in ealetlations you should employ the cules for operating with fapproximate numbers. In particular, in presenting the quantitative Adste and answers strict atteation should be paid to the rules of pen the ity. In some cases tuch an result obtained. For eanmple, «stone cannot be theown by «man ‘over the distance of tho order af 1 ka, the velocity of e body eannat SSnpase that of light to'a vacuum, ete NOTATION Vectors are, writen in boldface upright typo, et. r.Fs the same letters printed in lightface alte type (r, B) denote the modulus of Voit vectors 1, J, are the unit vector of the Cartesian coordinates 9,5 ome times the unit vectors are denoted at ey eye 830 fos uy ey ae the nit vestors a te eslindrieal cognates py 9,2, 2’ ree unit vectors of 8 normal ane tangent Mean values are taken in angle brackets (9, e, (), (P- Symlols 4, ¢, and 6 in rom of quantities donate Ay the tne acrement of 8 quently, eg At = Fs — Ed diernal(nntasnalincromend, ee, aU, 8: She Slmentary Value of quamy. eg. 84, te slementary work Time desvative of an arbiteary fanetion f # denoted by dit, or by # dot over a lte,j Veetor operator © (aabla”). 1 ie ud to denote the fllowing operations the gradient of (grad 9) Eb the divergence of E(B the url of E (uel ES Integrals of any multiplicity se denoted by a single sign f and Aller osly by the integration element: 6¥, » volume element, 8, 4 surface element, aad dr, tine element, The sign § denotes an Integral over close site, or around closed lop. w= PART ONE PHYSICAL FUNDAMENTALS OF MECHANICS fut, KINEMATICS 1 Average vectors of wlacity and excleration of © pot: wn. wad, 1m hae the planes vel (n ice of rade wa 7 Makly and solemn «pane co cu) fo, Acelaratin of «pont expres in protons on the tongent and the soma te oabe wnt a) ‘et i the i carat he ayo he en po lean cond bys poe t= fea a8 ee th made be ely vc of pit. Raat ya mga acleto ld Body edt, pod t lon between linge aod angular quantita for a rotating sold re Oe a wey # the radu ect of the considered point lative oa aiteny poi ‘he tition sus, sud ete distance fem the station ae 1.4, A motorboat going dowastseom overcame a raft a point A: +¢ 260 min ine Nt turned back and after some time pased the raft favs distance 6.0 km from the point . Find the fow velocity ‘suming the duty of tho engine to be constant. 12.'A point traversed hall tho distance with’a velocity vy, The remaining part of the distance was cavered with velocity 0, for hall ‘the time, and with velocity oy for the other half of the time. Find ‘the mend velocity of the point averaged over the whole tine of mo- 1,3, A cor starte moving roctilinearly, est with scesleration w = ‘340 mis (he iitial velocity is equal o ero), then uniformly, and finally, decalerating at the seme rato w, comes toe sop. Tho total time of totjon equals x =-25's. The average velocity daring that time is equal to) = 72 km por hour. How Tong does the ear move ‘uniformly? kc A point moves rectilineaely fn one direction. Fig. 1-4 shows so i } aan Were At VE A 4 wat FREE) tho distance «traversed by the point ay a funetion of the time f Using the plot find: {a) the averege velocity of the point during the time of motion: (@) the maximum velocity; {3} the time moment fy at which tho instantaneous velocity is qual to the mean velocity averaged over the fst fy sends 1.5. Two purtils, Zand 2, move with constant vlan vy and sp Ai the til moment thor rata veto are equ lof ah Fn mus tose fur secon be iatereste for tw partite ta cok ms r hn th th vel V6. A ship moyse slong the equator to the owt with velocity vp = 30 kan/hour, The southeastern wind blows at aa angle = 60° 18 die toustor in volocty0 = Te know, Find the wind Yaocey 2 rlasiven he ship an The angle ¢” betwen the eat ad the ‘tip etin in the reece frame fed the ship "11. Te enimiper lene pint -ton one banc ofthe Hive to each pola Tying right nose on th other bank Ope of them coset ovriver tone i hoe 4 while the other swims taht opto the freeman thn walks the sance tha he bas been ‘Ren by the eu to ge to pnt. What wes the wlocey neously? Th ‘0 krmfhour and the velocity * tof each swimmer with respect to water equals 2.3 kin per hour. 18. "Two boats, A and B, move away from a biov anchored atthe rilidie of a river slong the mutually’ perpendicular stesight lines ‘ier, aul the boat B across the river- Having “Hstoneo eam tho Ddy the hosts returned. Find the ratio of times of motion af boats tq/ty ifthe velocity of "1.2 fies greater than the = 150 times less than th river low velocity. At what angle to the Sram diretion must the foat move to minimize Uiflng? 40. Two bodies were thrown simultaneous from the sime poi aight up. and the other, at an eagle of 6 — 8 to the hae ‘The tnftal velocity of each body 1s equal to v5 — 25 t's Noaiging’ tho air dag, nd the dintanco betwen the" bodes # = 1.11. Two porticles move in ¢ uniform gravitational Held with an he init moment the particles were located at fone point and moved with velocitioe 2) 3.0 mis and be = 40 mis lyn oppasite directions. Find the di ween th Particles at he mament when their velocity vectors beeome mut Aly perpendicul TAB. Three points are located at the vertices of an equiate triangle whose side equals a. They all start moving simultaneously with velocity v constant in modulus, with the frst point heading fontigually for the second, the second Yor the thied, and the third for the fist. How soon will the points converge? ‘13. Point A moves uniformly with velocity» thatthe vector ¥ js continually “aimed at point B whieh tn its turn moves rect ly end uniformly with Walocity uo. At the Initial moment of ‘vil and the points are seperated by a distance Z. How s00n will cho -poiats converge? 114. Avteain of length 1 — 380 m stats moving rectilinearly with, constant accoleration w= 8.0-10"" mist; t= 30 # alter the Sart the locomotive headlight is switched on (event 7), and + = 60's after thet ovoat the tail sigaal light is switehed on (event 2). Find the ‘distance Between theap eveats ta tho reference frrbes Bxed to the train and to the Earth. How and at what constant velocity V rl tive to the Earth must s certain tefarene frune K move forthe to (vents to occur la It at the same point? TAS. An elevator car whose Woorto-ceiling distance is equal to 2.1'm starts ascending with constant sczsleratfon 12 mis 2.0-2 ‘after tho starts bolt begins falling fom the eallug of the car. Pad: (a) the bolts fee fall ime: (b) the displacement snd the distanea covered by the bolt during tho freefall In the rofeance frame Bed to the elevator shaft 1.16. Two particles, 7 and 2, move veith constant velocities oy and vy along two mutually perpenileular straight Lines toweed tho Interaction point 0. At the moment 0 the particles were located al the distantes fy and {from the point O. How soon wil the distance botwoon the particles bocome the smallest? What is it equal to? 1:17. From point A located on a highway (Fig. 1.2) one has ¢o get by éar as 00 as possible to potst located inthe fold ata dittonce£ from the highway. Tt is known that the car moves in the field 9 times slower than on the highway. At what distauce from point D. ‘one ms turn off the highway? 1.18. A point travels slong the z axis with a velocity whose pro: Jction v, fs presented asa funtion of time by the plot in Fig. 3 Pig 12. Assuming tho coordinate of aw the approximate tie 148. point te = 100 em duri time interval s = 10.0 s. Caleuats the following quantities figed over that time: "() the meen velocity (o () the modulus of the mean volocity vector |(#)]: {© the modialus ofthe mean vector of the total aceleration |} if the point moved with constant tangent accleratfon, ASDA radius sector of «pail are, wth time ta t= = at (1 — at), whecea is constant vector aad ais « postive factor. Finds {tthe vlatey van the acelrationw ofthe patil a fonctions () the timo interval ¢ taken by the particle to roturn to the in tial points, end the distance» covered ding that Lime "LBL. AI the momont ¢ 0's particle leaves the origia and moves In tv positive direction of the 2 axis. Its velocity varies with tine a5 vim vy (1 — 2), whore ve fo the initial velocity vector whose ‘odulis equals vy = 10.0 emis; x = 5.0.2. Pind (a) the 2 coordinate of the particle at the moments of time 6.0, 10,and 20's) {tb} the moments of time when the particle i tthe distance 10.0 em from the origin: {0th ante «ceed by th pale dung the ist 4.0 ond 80m draw the approximate plot (. 1.22. The velochy of a particle moving Inthe positive dteetion of the = axis varies as = V/ whore a is postive constant. ‘Resuming that at the moment f —'O'the particle was located at the sate 0 va a ‘time dependence of the velocity and the acceleration of the patie {b) tho mean velocity af the particle averaged over the time that tho particle takes to cover the ft + metres of the path. 4.23. 8 point moves rectilinesrly with deceleration whose modulus ponds on the velocity v ofthe particle as w= az, where a is a positive constant. AU tho initial. moment the velDeity of the point Frequal to ry. What disteneo will # traverse before it stope? What time wil it fake to cover that distance? 1.24. A radius vector of a point 4 relative to the origin varies with time Cas att — br, where a and B are positive constants, and L ‘andj ae the unit vectors ofthe 2 and y axes” Find {a} the equation ofthe points trajectory y (2): plot thie function (0) the Lime dependence of the velocity v and aceleration w vee tore a4 woll a8 of the moduli of these quantities Te) the time dependence of tho angle bet won the vectors w and v (G) the mean velocity vector averaged over the bist seconls of motion, ni the modulus of this vector 125. A point moves in the plane zy according to the law z ~ at, y=at (lath, where a and are positive constants, and (1 Hane. Find (a) the equation ofthe points trajectory y (2s plot this functions (B) the velocity © and the acceleration of the point a= functions of time: (¢) the moment fy at which the velocity vector forms an angle w/4 swith the acolertion vector. 4.26. A point moves in the plane sy acronling to th sia wiry = a(t cos of), whore a and a are positive constants Find (a) the distance s travers by the point during the time (0) the angle between the points velocity and acceleration Vectors. 1127..4 particle moves In the plane zy with eoastaat acelerstion w dirceted along the negative y axis. The oquatien of motion of the pevticle ae the form y = az"—" bz where # and b ate positive con ants. Find the volocty of tho pericle at the origin of coordinates. 128. small body is thrown stan angle to the horizontal with the inital velocity vy. Neglecting the air day fn: {a) tho displacement of the body ax function of time x (); {@) the mean velocity vector fv) averaged aver the frst f seconds ‘an over th total time of notion 1.28. \ body is thrown fcom the surface of the Barth at an angle 16 to Ghe horizontal with the initial velocity vy. Astumiag the aie deag te be nowlisible, find: (2) the time of motion: (3) ‘the maximum height of aseent and the horizontal range: at what value ofthe angle = they will be equal to each other (@) te equation of trajectory y (2), where y and are displacements ‘of the body along the vorlieal and the horizontal respecuively: (a) the curvature radi of trajectory at its initial point snd at its peak, 1.30. Using the conditions of the foregoing problem, draw the ap- proximate time dependence of modulo the normal iy and tangent Hceoleration vectors, a8 Well as of the projection of the total accele= tation woctor ws, on the velocity vector direction 1.34." ball sorts falling with wero initial velocity on « stpoath Inclined plane forming an angle @ with the horizontal. Having fall fn the distance y the ball rebounds elastically of te inclined pl ‘At'what distance fom the impact point wll the bell ‘he second time? 32. A cannon and a tergt are 5.40 kim apart and located at the same level. How soon will the shell Iounched with the inital veloetty B40 ros ouch tho target in the absonce of aie drag? 138. A eannon ‘res suecesively two shells with velocity vy — ‘250 mls the frst atthe angle ©, = OO" and the second atthe angie 8, = 45° to the horizontal, the atimuth being the sume. Neglecting ‘le air drag, find the time ioterval between firings Tending Uo the ‘lion of fhe shells, 1.34. A balloon stars rising from the surface of the Barth. The ascinsion rat is constant end equal to. Due to the wind the Dal Toon athers the horizontal velocity component de ~ ay, where a isa constant sad y isthe height of ascent. Find how tho following % (a) the horizontal drift of the balloon = (y; ‘nd normal aoselerations ofthe belloon, the plane y with velocity ¥ = al ba. ‘of the z and y axes, aad a and 6 “eas Toented the point == y (@) the equation of tho partic (8) the curvature radiue of 4.36. A particle A moves in ove direction along a given trajectory with a tangential acceleration wy = at, whore ais a-constant vector {oineiding tn direction with the z exis (Fig. 4.2), and visa unit vector coinciding io direction with the velocity vector at a given point Find tow the velocity of the particle depends on x provided tat its ‘velocity is nogligible at the point x =O. 1.37. A point moves along a cirlo with a velocity v — at, where 20.50 mist Find the total acceleration of the point at the mo- w lrajoctory y (2) ory as 8 function of = ‘mont when it covered the n-th (n = 0.40) frction of the circle beginning of motion. 538, A polat moves with deceleration along the circle of radius sothat at any momeat of time its tangential and normal accelerations Mig, 14, ‘ase equal in moduli At the inital moment ¢ Fei tus Fn "(@) the volo ofthe point a «function of tan end as « function ot th datanc tovered ft (0) the tal aecaoration ofthe point as function af weocity and thn distance covered 1.89. point moves along an arco a citcleof radios. Ite velocity depends on the distance covered an = aV/a, where ait a ovostent, inthe tale Savon he yc af the tl scoiaion and isctor of velocity as function of 1.40. partcle moves along amar of cise of radius sone to the low = asin of; where isthe displacrment from theta Fastion measured along tho are, and sand, are coastate:Asrumn- Toe ="100 sm, @= 0.50" and w ~ 2.00 raul, and a) te magaitade of the total acelecation of the particle at the poimts = O'aad f= ey (©) the minimum va ofthe total assertion img and the cor nine displacement fy rats A point moves (athe plane so that is tgngeaial acsleration fil its normal seeloation iby ~ Bt where a and are anata, and tis tame. At the inoment t=O the pot Atrest Find how the curvature rads of the pats trajectory aad the tail aceleration w depend on the distance covered {.42.'Aparticle moves slong the plane teajetory y (2) with vlo- ity v whore modus iy constant. Find the teslrston ofthe pa tc al the pat 2 ad th carnitine cia of th tects at that point tthe trajectory hes the form (0) fs parabola y © a {0} otal (af) + (/" — 45 and Dare constens here 1:43. A°partle A tmoves ‘along a citele of radius t= 90 tm so that its radius veotorevetative to the point 0 (Bg. 1.8) rotates with Ue" costaat-anguler velocity w 0.10 rade Pind the modulus of the Nlocty f the partie, an the reads ond ditetion ofits the velocity ofthe 1.44. A whoo! rotates around a stationary axit so thatthe rotation nl varice with lime nap = al, whore a ~ 0.20 radist Find the {otal agclerationwof the point A nt the rian atthe moment ¢ 25s IT the linear velocity of tho point 4 at this moment » = 0.65 mis. 145. A shell 500 mis, having i equal to = A inside the barrel with fiom, nd the angle roiailon tthe moment. when the. shell ‘Seapes te barrel TTH6. solid Body rotates about a station ary" tis nccording tothe’ law sree where == 6.0 rails and’ b= 20 radia, Find: (a) the mean values of the angular velo- city and angular acceleration averaged over yy 45 the time interval ‘between t= 0-and the ie complete stop {(h) the angular acclogstion at the moment when the body stops 47. 'A solid body stars rotating about a stationary axis with an sngular acceleration = at, where a= 20:10-* radi How soon after the beginaing of rotation will the total acelertion vector of fag arbitrarg point of the body form an angle a = 0" with its velo ely vector? 1.48. A solid body rolates with deceleration about a stationary vith an angular deceleration BOC Ve, where w ie its angular Selocity. Find the mean engular velocity of the body averaged over the whaje time of rotation fat the initial moment 9f time fs angular ‘velocity was equal 10 oy ‘4.49. A'olid body roteles sbout a stationary axis so that its angu lar weloety depends gn the rotation angle @ a3 = ey ap, where toe ond a nce positive constants. At the moment 0 the angle '. Find te time dependence of (a) the rotation angle (0) the angular veloety. 1.50.'A solid body sters rotating about a stationary exis with an angular aceeloration B = By cos g, where By isa constant vector and is an sogle of fotation fom the initial postion. Find the sngulat velocity of the body ae function of the angle g- Drew the plot of ne {1."A pertele of mass m moves along the Internal smooth sur- ‘ace of « vertieal ylinder of radius R Find the fore with which th particle acts on the eylinder wal if at the initial moment of time [Re velocity equals vy and forms an angle & with the horizontal. 85, Find the magnitude aad dlretion of prcle of masom drifts motion inthe plano = Few z= asin ut, y= 6 cos ot, whore a, b, and ware constant 1.96. Abody of mate m is throwa at an aogle 10 the horizontal with te inital veloity ry. Assuming tho air drag to be negligible, fad {c) the momentum inrement Ap that the body acquires over the feat seconds of tation: (bythe modulus of the momentos increment Ap durog the total time of motion 1.07. At the moment ¢ ~ 0 a stationary particle of mass m expe riences a time-dependent force F'= at (t=) where ais constant ‘vector, isthe time during whieh the given force ets. Find {@) the momentum ofthe particle when the action of the force ise contin () the distanco covered by the particle while the force acted. 74g. At the moment t= Om particle of ass m starts moving due tons force F~ Fain ot, where Fy and o are constants. Find distance covered By the particles function of Draw the approx imate plot of this funetion 800 At th tea to a foree P= Py con ‘0a pagtcle of mass m starts moving de ‘where Fy and «are courtante, How loug will it be moving until it stops for the Best time? Whet distance il Tetraverse during that met What is the maximum velocity of the Pagtcle ever this distanct? 1.100, A motorboat of muss m moves along a lake with veloety 2, Avie moment t=O the engine of the boat i shat down. Assuming the resistance of water to be proportional to the veloeity of the boat For, find (a) How long the motorboat moved with the shutdown engine: (@) the velocity ofthe motorboat asa funetion of red with the shutdown engine, as well as the Total Ail eho complete stop: (6) the mean velocity of the motorbont over the time interval (beginning with the moment #0), during whiet is veloeity de- reas times. 1A04."Hfaving gone through » plank of thickness 4, a bullet changed its Volocty from vy to P- Find the time of motion af the bullet in the plank, assuming the resistance force to he proportionel {0 the square of the velocity. 4.402. 'A'small bar starts sliding down an inclined plane forming fan angle 2 with the horizontal. ‘The fiction eoelRelent depends on {tte distance > covered as F— ar, whore a is'a constant. Find the distance covered by the bar till it stops, and its aaximiia velocity over this distance. 41:408."A Body of mass m rests on a horizontal plane with the fric- tion coeficient k. At the moment f = O'a horizontal force is applied to it, which varies with time as F = af, where alsa constant vector. Find the distance traversed by the body dur the fares action began “L404, A body of mass m ie thrown stright up with velocity vy. Find the velocity v with which the body comes down Ifthe att drag aqua, where ig «constant and vis the velocity of the boty "iOS. A particle of mass m moves ina cerin plane P due to” force F ooo magnitude, is constant and Srhose vector sotates in that plane With eonstant angular velocity 0. Assum- ie the particle. to. be stationary at the moment t= 0, Bnd (a) He velocity asa. function of tune: (s) the distence covered. by the pattlele betwoen two successive stops hd the wean velocity over this time “L406. A small dise A is placed on an inclined plano forming an angie a with the torizontal (Fig. 1.27) and imparted an initial ‘locity vy Find how the velocity of the dise depends on the angle @ Ii the friction coetclent tan @ and at the initial moment Py the first #seonds after 407. A chain of length 2 is placed on a smooth spherical surface rs fixed at the top of the sphere. What ch element ofthe chaln when Its upper ese? Tes azued that th length of th hin 2 4.108, A small body is placed on the top of a smooth spbore of radius A. Then the sphere is imparted a constant acceleration wy in the horizontal direction and the body begins sliding down, Find (ae ect ofthe Body rave ete sphere a the mote Dreak-of at agpantin a i Sea Tift. Asie was aimed atthe vera line onthe target Ioaied Caen eee a outa tion, wil th bulot hit the target. The shot was Bred in the horizontal sea h iaS e SRST cata ite ite Aas i ema es, de ica Scrat Sets witht ra tenag's “Eo perro ela ca rg ees ea pia tects a etn ‘with velocity that is constant relative to Jew ie pntie at te manta data io te rottion ‘he resultant of the inertial forces Fy acting on the particle in the Feference frame Axed to the dise tra into ero. Find: (a) the acesleration u of the particle relative to the dis (0) the dependence of Fn on the distance fromthe rlation axis. 115. sinall body of mage m = 0.30 he stars sliding down from ‘the top ofa smooth sphere of radius R= 400 an The sphere rotates with a constant angular velocity «© — 6.0 radls about a vertical {iis pesing through its centre. Find the ceotrfugal Tere of inertia td the Conolis fore at the moment when the body breaks of the Fuifaceof the aphero inthe reforene fame xed to the sphere TAA train of mass m = S000 tons moves in the latitude 0" Non. Find: (@) the magnitude and direction of the lateral force that the train ‘xerts on the rail I It moves slong a menidian with a velocity ‘4 kan per hour; (0) in what direction and with what velocity the train should move. for the resultant of the inertial fores actiog an the trainin the el- erence frame fixed tothe Earth to bo equal to 280 cll. At the aqustor a stationery (relative to the Earth) body falls down from the height = 500 m. Assuming the aie drag to bo negligible, ind how much off the vertical, and tm what direction, the body wall devine when ft hits the ground Fig, 18 13, LAWS OF CONSERVATION OF ENERGY, MOMENTUM, AND Work ad power of be face a= fra fra rome ay 1 of the Rinse caery of ptt Thonn A rey hr the work pore hy the restant the forces sting o8 the pets energy of patil te geen B= Us (139 4, Relnonshp betwen the ore of» Sil and the potential enetey of a roche tn te eld mleeoe ud sequal wt decease ft ‘a w Feav0, (a) |e the fre is equal to the antgradieat of the potential eater «,focteweut ofthe taal mechani ergy e's prt in ven poten via bad a BoB. mAae (ise) ere Ap, a the alma am of wer perfor by all extremes fore, TENA arth elongag acl te ge Be a + icraect ofthe total tackaniclsoeey of ser Ba Bem hea AT, ast here £ = 7 4. Uy and U isthe inherent potential snargy of the syste, Law of momenta vacation of sje anit (30) ete F sth raltant of al rma foe ' Faontea ef tion of the mysom's canes of ini: aay where is th olan of al extemal fren Kinetic emery of stars as sy rar wi he velocity of he separated (gnned)aubtance relative to the body Low of angler momentum vanation of © system 7 3 ‘thee M i the angular omentum ofthe apse, and Ni the etal momant of cea ular omentum of syste Mm HY Ue os) ors yt angla nent a he mtr the cet of nr a Se RU TT sa Le a 1.148. A particle has sifted slong some trajetory inthe plane =v from point 7 whose redlus vector f, 1-42 to point 2 with the ediud veclor yo" 213}. During’ that time the’ particle. exper fond the action of certain foreas one of which being F = 31-1 Find the work performed by the fore F. (Yeo ry ry and Pare given in Sl unit) 1319. A locomotive of mass m starts moving so that its velecity varies according to the law b= af, where isa constant, and ¢ IEVUES stance covered. Find the total work performed by all to {erees which are acting on the locomotive during the hist f seconds ie cnn oxi Tha0. The kinetic energy of particle moving radius R depends on the distance covered 0s 7 constant, Find the free acing. on the pat- cle as function of &, 1121.4 body of ase m waa slowly hauled sup the full (Pig 1.28) by a force Fw fxch point was directd alons a tangent tothe {Uspctory. Find the work preformed hy. thi forte, ifthe height of the Bill h, the length fits base. and the cofbcient of fetion & 122A dise of mass m= 30 g slides with the"toro initial velocity down an clined plane et at an angle = 90" to the horiontal Erving traversed the distance 1 = 50cm ong the horizontal pa the disc stops. Pind the work performed by the friction fores over {he whole dstance, assuming the frietion coefcient t= 0.19 Tor both inclined and horizontal planes. 1-123: Two bar of tases rand my connected by a non deformed light spring rat on s horisrtal plate. The coefficient of tretion tween the area the aries equal to. Whit minimum constant force has tobe aplied inthe horizontal Uiectin to the ba of mass m, In'onder to shift th othe at? {424 chatn of mass m 0.80 ke and length 1 = 4.5 m rests an a roughaurfaced tae s0 that one of its ends hangs ove th wie ‘The chain sterts sliding ofthe tae all by ial provided the over. hanging part equals n= 413 of te chain lenglt. What wll be the ah ete eit total work performed by the friction foros acting on the chain by the moment it slides completely off the table? 4.125. A body of mass m is thrown at an angle to the horizontal ‘with the inital velocity vy Find the mean power developed by gravity Uverthewholetimmeof motion of the body, andthe Instantaneous power tf gravity as funetion of time ees M426, A particle of mass m moves along a ciclo of radius R wil 1 normal acesleration varying with time a8 th ~ af, where a a ‘Sconstant. Fld the tine dependence of the power developed by all {the forse acting on the particle, and the mean value of this power ‘voragod over the fist seconds afler the beginning of motion ‘127. A stall body of tass m Is located on a horizontal pla the’ polnt 0. The body scquites « horizontal velocity Uy, Find: {() tle mean power developed by the friction force during the ole ting of motion, ifthe eeion tice = 027, = 10%, ind y= 13 mis, “the maximum instantaneous power developed by the friction fofen tt the friction coefficient varies aa — az, wheroa isa constant, tnd ithe stance fom the point 0. ‘428. A’ small hody of mast m = 0.10 kg moves 0 perform dur Foom point to point 2 which are located atthe distances", — 9B ca and 72~ 30 ext from the Totation axis? “149. Avsystem consists of two springs connected in serles and having the sifinss cocthcints hy dy. Pied the minimum work to be performed fo order Lo stretch this sistem by AL 16" "Body of ass hed fom the avis surface by applying a force F varying with the height of ascent yas F = 2 (ay — “Phymg, where a ft postive constant, Pind the work performed by tus fore and the {nerement of the body's potenti netay in {ie gravitational held of the Barth over thelist half of the agent. “51. The potential energy of particle ina certain field has the fora U'— air "bin where a and b are postive constants, ris the distance com the centre of the held. Find: a) te value of ry corresponding to the eguilibsium position ofthe particle; examine Whether thi position is steady {b) the maximum magnitude’ of the attraction force; draw the plata Gy and A, () the projections ofthe force on the radius vec for 8. 1.232. In a certain two-dimensional field of fore the potential ‘encigy of a partice has the form U'~ az* + By, where and B Whose magnitudes ae diferent. Find ox Bela Bente (6) what isthe shape of the equipotential surfacts and also of the surf for whith the magatads f the recor of ores F= ons, 1499. There are two stationary Belds of foes P= ayt and F = 5 whore i and j are the unit vector of the and y axes, hd out whether thw elds ae poston: Hou of mass is pushed with inital eee oy ap.an inclines Plane ant at on angle tote hoon. The fetion Duietnt i el tk, Wat datos wil he body cover befor it Stopestnd shut wotk de th ition ove pertrs ove tie tr sero ‘of 2 anooth bill of et portion (Fig. 1.30), What mst be the height of the horizontal Bion to ensuze the maximum distance + covered hy the dise? What {it equal to? 186. A small body A starts slid from the height dowa, an inclined groove passing into a hall-eiele of radia M2 (Fig. 130 4 4 l, ie f Fig, 1.0, Mig tat, Assuming the fection wo be negligible, find the veloety of the body that pote? What wil ion of the thread at the moment till be passing the horlsental position? 1.438. A horizontal plane supports stationary vertical cylinder it disc A attached to the eslinder by horizontal math fy (Fig. 1-32, top view). An initial velocity 2 of radiug Ban theead 4B of B ig. 2 Pi. 438. ‘is imparted to the dise as shown in the figure. How long will i ‘move slong the plane watil it strikes ageingt the eylinder? The fole~ ton assumed to be absent Hash a 4.439. A smooth rubber cord of longth I whose eoeficiont of elas- fHelty is Is suspended by one end from the point O (Pig. 1-33). ‘The other end is htted with a cateh B.A small sleeve A of mass m stars falling ftom the point O- Neglecting the masees of the thread ‘and the catch, find the maximum elongation ofthe cord. ‘1.100. A small bar A resting on a smooth horiontal plane is at- tached by threadsto a potat P (Fig 1.34) and, by means ofa welghtlese pulley, to a weight 2 possessing the samo’ mass as the bar itself. Pir 136 Fig 435, Besides, the bar is also attached toa point O by moans of alight non deformed spring of length ly ~ 50 em and. sifless % ~ 8 mally where m isthe mass ofthe bar. The thread PA having been bined, the bar starts moving. Find its veoeity at the moment when it 1 Deeaking off the plane HAM A orioatl plane supports a plank wth « bat of mae m= 4.0 kg placed on ft and attached byt light elas on-de- 140 en to point 0 (Fig. 4.88). The cock tet of sion Beton he ar andthe plank eal #0. lonk is slowly shifted to the right nti the bar starts sliding Ti It oceats eu the momeat when the cord deviates from the ‘ertical by an angle 0'= 30" Find the work that hasbeen performed by that moment by the friction force acting on the bar in the rel tence frame fixed tothe plane 1442. A smooth light Horizontal rod AB can rotate about a ver- tical axis passing through ils end. The rod i hited with 9 small loove of mass m attache to the end A by a weightless sping of length Igand stifness. What work must be perforsied to sow!) get this ‘s}stom going and reaching the angular Velocity {.143. A pulley fixed tothe ceiling carries a thread with bodies of masses my and mz attached toils ends. The masses ofthe pulley and the theead are nepligible, fiction ie absent. Find the necoleration Woof the entre of inertia ofthis system, “144. Two interacting particles form a closed system whose centre of inertia is at reat. Py. 1-96 illustrates the positions of both par ticles ata certain moment aud th trajectory ofthe part iele of mst my. Draw the trajectory of the partiele of mass ms if ma = m2. 1.445. A closed chain A of mass m — 0.96 kg is attached to a ver- tical rotating shaft by means ofa Uread (Fig. 1-37), and fotates with constant angul ls. Th th le 8 S05" with the vertical. Find tho distance between the chain's entre of gravity ani the rotation axa, and the tension of the thread. & 1:86. A round cone A of mass m — 3.2 kg and hall: angio a= 10" rolls uniformly and without slipping ‘long s round cones! surface B 20 that its apex O te: Iman snionary. (Fig 8). The Gente a parity of the cone a ivat the same level az the point O snd at a Aistance f= 17 em from 1 The cones axis moves swith angular velocity 0 Pied: (a) the static fiction force io = 10 rails, (hat what values of o the cone A will soll without ‘sliding, I the cootfelent of ‘etion between the surfaces is equal 5. ting on the e000 A. fig 4.96 long the 2 axa, ono of mass my With Velocity vy, and the other of mass m, with Yelocity va Find: (@) the Felocity V of th rferones frame K” in which the cumulative kinetic energy of these partiles is minimum: : () tho cumulative kinetic energy of those particles ia the A’ fra, “148. The reference frame, In which the centre of inertia of a given aysiem of particles is at rest, translates with a velocity V relat of inertia particles in the referonce frame. A: 1.149. Two small dises of masses m, and m, interconnected by weightless spring rest on & smooth horizontal plane. The dises are et in motion with intial velocities % and vg Whose directions are ‘mutually perpondicolar snd lie in s horizontal plane. Find the total gy FE ofthis stom in tho frame of the centre of Inertia 1-80, & stam soit of two ama phe of mane m, and my connected by a weightloss spring, At the moment £0 the sphores ate st in motion with the initial velocities v, and v, after ‘which the syatom starts moving in the Barth's uniform gravitational Feld. Neglecting the alr drag, nd the time dependenea of the total ‘momentum of this system In the process of motion and of the radius ectr of cet of uta slain to he Titel poston of the 1451. Two bars of masoos m and m, connected by 2 weightless spring of aiffaces (Fig. 1.8) rest on a smooth horizontal plane. fs Bar 2 ts sited the floaty of of el i 132."Two bars connected by » weightless spring of sifaess x tnd length (in tho aon deformed sat) fy rest os herizontal plane ‘constant horlconal fore Fars ating on one of he bars a shows fe) equ (G) equal 'to.m, and ms, and the force F is applied to the bar of ‘of two identical cubes, each of mass m, linked together by the compresed wolghtlest spring of stiffness Gig: 141). The cubes aze also connected by a trend ‘which is burned through ata eeriain moment. Find: > Ho) a what valuea of Al. the initial compression of the spring, the lower cube. will bounce up after the thread hes been burned throuzh: Co) to maa ete the cate eat of ths a system will rise ifthe inital compression of the spring a Bra Iagh ne 4.154. Two identical buggies J and 2 with one man PME fp ach mote without tcp dhe 1a Tari slong Darallel tals toward each otter. When the buries get opposite fash other, the mon exchange thelr places by Jumping in the direc- tion perpendicular to the motion direction. AS a conoequence, buggy * {stops and buggy 2 keeps moving in the same direction, with is ve- {cei becom equal tv. Fd the Lita voloctis of the buries Wd ya tho tas ofeach bugay (without a man) equals 3 and Ue mst of each men "Te ideatial buggies move on aftr the ote due to inertia fon) mith thersame velocity We A tan of mass rides {fettear buggy. Ava certs smosnest the Man Jumps into te front ay whe velcy u roletive to hs boy. Rowing thet tho psf acy 0, Sd th voces with which the Dursies will more after that "Sige. Two mon, eash of mass, stand onthe edge of statonar buggy cf mess Mf Asuiiag the eieton to be neti, fd t Nelociy of the buggy after oth men Jump off with the Tonal Yolocty w felauive tothe buggy: (D simultaneously, (2) one Sher the eter in wha case wil the velocity of the buggy e eestor Sat" how many’ umes? {A077 A culo hangs on thread and touches the surface of «table by its lower ends Show that slr the twa has Seen burned trough, ti force exerted onthe tabla by the falling prt of the chain at any Tuoment is esas great seth force of preture exerted by the Part livedy resting om the tao Tig8.'A stl all of mass m = 50 g falls from the height h = = 'ildm on te horizontal aurtoeof a masive sab, Find the camo {iuive momenturn that the bell imparts tothe sia after Dumerous ounces if ovary pact deeeases fhe velocity ofthe ball n= 1.25 tia, A raft of mass Af with oan of massm aboard stag motion. lesson the surface of lake, The mon moses + distance I relative Xo the raft with vocty '() and then ops Assuming the water Testa to be edit, od a) the displscement of the rat 1 relative to the shor: {3} the fetta! component of he foes with which the man ated ny that drag Ce to. a 1160.4 stationary pulley caries a rope whose one end supports a [udder with'n an il the other end the counterweight of me Bf to the ladder , “displacement I of tho centre oF inertia of inlined pl lea tothe horizontal. Alter the canon co¥ cred the distance f, a shot was Tired, the shell leaving the cannon in the horizontal dieetion with « requence, the Eamon stopped. Assiming the mass of the shell to be negligible, “ompared 10 that of the canton, determine the duration of the Shot {:162. A horizontally fying bullet of mass m gots stuck in a body of taass Af suspended by two identical threads of length 1 (Fig. 1.42) ‘Aa rnlt the threads swerve though an angle, Assuming m ma) Find the maximum angle through ‘tien the striking particle mey deviate as result ofthe eolision. "i176. Thrve identical discs 4, and C (Pg. 1.49) rest ou a smooth horizontal plane. The dise ls set in motion with velocity ¥ after ‘of two partici of masses om, to ench ether with velocities which it experiences on elastic callision simultaneously with the Aises B and C. The distance between the ceutres of the lavter dises Dror 1 the colision is thmes greater than the diameter ofeach dise Find the velocity of the dise after the callision. At what value of wil the dise ae 2 cal far he alii? son move on? ¢ QO ‘77. A molecale collides with another, (>) 2, (7 stationary, ofthe same mans. Demonstrate that the angle of divergence ) equals 0" when the eallision is ideally elastic (© dates fom 807 when the collision ee fs inelastic $78. A rocket ejects a steady jet whose velocity is equal to w ‘lative to the rocket. The gas discharge rao equals f.kg's. Demon Strate thatthe roekot motion equation iy this ease takes the form mw = F— pu where m isthe mass of the rocket at given moment, w is its accel= ration, and F (the external force. yt 8 poke mete pth bac a extra fs y eect 4 steady Jot with velocity w constant relative to the took Fibd the velocity y of the rocket at the moment when its mass lp fcqval to m, if atthe initial moment it possesed the Mase my {Hs velocity was equal to ero. Make use of tho forma given in the foregoing problem: TAB. Find the law according to which the mass of the rocket varies with time, wien the rocket moves with constant socelra: tion w, the external forcos aro absent, the gas escapes With stant velocity u relative to the rock, TABI. A spaceahip of mass my moves in the absence of external forces with avconstant velocity ¥,. To ehange the motion direction, ‘Switched on. It saris ejecting a gas jel with velocity istant relative to the spacrship end diveted at ight Sogles to the spaceship motion. The engine is shut down when th ‘ats of the spaceship decreas 20 m. Through what angle & did the ‘wotion dirvction of ‘the spaceship dviate due to the jet engine op- TAS. A car loaded with moves slong a horizontal plane due 0.6 constant force F coinciding in divection with the carts velocity vector. Inthe process, sand spills through a hole ia the bottom with ‘Veonstant velocity jks. Find the acceleration and the velocity of the eart at the moment f, at tho Initial moment f= O the cart ‘with Toaded send tad the tase me and its velocity was equal to sero. The frieion is to be ‘neglected. 4483. A Uatcar of mass my starts moving to the right due to constant Horizontal force F'(Pig. 1.46). Sand spills on the fatear « from a stationary hopper. The velocity of loading is constant, and fqual to p kgs. Find the time dependence of the velocity aad the Sccoleration of the Mater in the process of loading. The rietion is Aegligibly ‘sal i. A chao AB of length 1 is located in a smooth horizontal tube so that its fraction of length h hangs freely and tovehis the surface of the table with its end B (Pig. 447). At a certain moment A i. ig. 17 the ond A of the chain inst free, With what velocity will this end of te chain ap out-of the tube? {.i55, The angular momentum of particle relative to a certain point 6 varies with time as M'= a bt, where-@ and b ant vectors, with a... Find the force moment point acting onthe particle when the angle between nd equals 4 4.486."A all of sass m is thrown at an angle « tothe horizontal with tho Initial locity by Find the time dependence of the mage itude of the all's angular momentum vector relative to the point from wich te Dall is thrown. Find the soyular mor Shee point of the trajectory Sim = 13D. S‘zbimis. The ait drag is to be neglected. {cist A dise of mass m sliding over & smooth horizontal surface i eoety woxpeinee a petty lai lin witha sooth ‘ationary wall at poiot O (Pig. 148). "The agle between the motion divection ofthe dae. 4 Sod the normal of the wall is equal to, Find: a) the points relative to wih the angular amomentum M af the dite temaing constant fn this proces (@y the magnisude of the increment ofthe ‘eetor ofthe dis's angular momentum relative to'the point ” whichis located. in the plane (OF the die's motion at the distance from the point 0. 488. A small ball of mass m suspended Pie 18, from the clling at's point 0. by "a thread of length F moves along’s horizontal circle with a constant sngular ‘olocty w. Helative to which points does the angular momentum Mot tio ball remain constant? Find tho megaitade ofthe Ineremeat of the vactor ofthe balls angular momentum 6 picked up during hella revolution. T-180. A'ball of mase my falls down without initial valocity from ‘a holent f over the Barth's surfoo, Find the increment of the balls ‘angular momentum vector plckod up during the time of falling (rla- tee to the point 0 of the rforeaca frame moving translationally in ‘horizontal direction with a velocity V).The ball starts falling trom he point 0: The tr dee is to be, ntletd T190. K’smooti hooisontal dive rotates with a constant angular ‘velocity about » stationary vertical axis pasting through Its contre, the point 0. At a moment ! 0-8 dive 1 set im motion from that 6e fia nye Fie 18, Fie 1.50 pint with velocity vy. Find the angular momentum A (?) of the Like selative to the poiat O in the reference frame fixed to the se. Mate sur that thir engolor momentum is eauaed by the Corais force ‘Rial. A particle moves slong a cloed trajectory in central fol af fret where tho Particles potential energy ~e* (eis a Positive contri the distanos ofthe porte from the eee O BPine holds Find the tas of the pctiele isis minum ditenca irom the pont O equate, and te vest atthe pat farthest rom © als euymg2.’A small Dall is suspended from a point O by a light thread cot feng f Then the ball dean side st that the thea ev hugh an angle 0 trom Qho vortiat and set im motion iy hore uta distion at right angie tthe voricel plane whieh the Fuad (locate. What is he iil velocty that hast be imperted to'th bell so that it could deviete trough the maximum angle 222 {nthe procs of motion? 14890"Q small ody tf ass m tnd to a nonstretchable thread snoves vee a smooth horianntel plane, Th ote endo tho teed ‘E'Dog drawn Tato'a hole O (Pig. 1-49) ith constant velocity. Find fh {vend nsion asa function ofthe distance r betwen the fou tad the hole f'at r= ry the angular velocity ofthe three fs cual ole Aight non-stetchable thread! i wound on a massive xed Stradius ft A'snall body of mane ia God tothe five nd Bithe thea. AC's moment =O the system i released an starts faoving, Find its angular momentam relative to the polly tale t3 T'tunlion of time a 41.185. A oniform sphere of mass witout lio down a neti plane at a angle fo the hore Zonta. Find tho time dependence of the angular momentum of the {plore relative to the polut of eontact at the initial moment. How will the obtaiged recut change in the case of a perfectly smooth ined plane? 1/406. A certain system of particles possesas «total momontuin p sadam angular momentum M relative to point O. Find its ang ‘momentum M’ relative to point O” whose position with respect to the point O Is determined by the radius vector r Find out whom ‘the tngular momentum of the system of pasticler dove not depend fn the choica ofthe point 0. 1-197. Demonstrate that the angular momentum M of the system of particles relative to 9 polat Of the roference frame K'eaa be Te. printed. as M= M+ trop, where BH is its propor angular momentum (in the reference frame moving translatonally and Ged to the centre of Snertia). re the radius vector of the contra of inortin relative to the point 0p i the fotel momentom af the system of particles in the reference frame. K- 1498. A Dall of mass m moving with velocity vp experiences & Iheai-on clastic collision ‘with one of the spheres of a stationa "gid dumbbell as whown in Fig. 1-80. Tho mass of euch sphere oguals ‘m/z, and tho distance hetweon them i . Dlsogarding the sae of the spheres, find the propor angular momentum Af of the dumbbell after Uke cllision, be, the angular momentum inthe reference frame mov" Ing ‘eapeation "108: Two Yorizontal plone. The discs are interconnected. by’ ight_nom formed speng of longth y and stifass xc At certain mornont one of the disce is sot in motion in a horizontal direction peependicul to the spring with velocity og Pind the maximum elongation of (he Spring ih the process of motion, if It Is known to he considerably Tiss than unity 4.4, UNIVERSAL GRAVITATION 1 Universe ravitation aw eo (1) The squares ofthe peso ofrvoation of any te lente argnd the ‘un Se proportional to he Cates of Un major seminar es (Repl ee wa) patenting of te rastational Bed of « 1 Sth Go Genre ert, =) + ori and cape ote Viti i= VE 4.200, A planet of mass A mo, with velocity 9 = 349 kin/s (lative tothe Frame), Find the peried of revolution of this “{20i. The Jupiter's period of revolution around the Sun is 12 times that of the Barth. Assuming the planctary orbits be circular, faa: {) how many times the distance between the Jupltr and the Sun cexcouds that between the Earth snd the Sum Gh) the velocity and the acceleration of Jupiter in the heliocentric reference frame. T1002, A planet of mass Mf moves around the Sun slong an ellipse so thet ite minimum distance from the Sun i equal to rand the maxt- {rum distance 40 2. Making use of Kepler's laws, find Its period of revolution around ‘dhe Sun i208. A smell body stars falling onto the Sun from a distance ‘equal to the radius of the Earth's orbit. The (itil velocity of tho Dhady ig equal to aero in the holiocaatle reerwnce frame. Making tee of Kepler's Ines, fd how long the body wil be falling “204. Suppose me have mado & mode of the Solar system sealed down in the ratio y but of materials of the same mean density a8 fhe actual materials of the pl periods of revolution of plenetary models change ta this ease? 1.208. A double star a'a systom of two stars moving eround the coniro of nertis of the systom due to gravitation. Find the distance Frtween the components of the Jouble star, its total mass equals 3 fand the period of revolution T. 1.206. Pind the potentiol eaergy of tho gravitational interaction (4) of two mass points of masses m, and m located at distance r from ‘ach other, Fr ”— and eugth fy if taey ore located slong a straight line at a distance & from tock otha; ako fad hn Tore of het stereo. “Lit. A planet of mass m moves along an ellipse around the Sun so that its maximum aad minimum distances from the Sun are equal to ry and ry respectively. Find tho angular momentum Af of this planet roletive to the contre of the Sun. 208, Using the conservation laws, demonstrate that the total smochanieal energy of & planet of mast m moving around the Sun ‘Tong an ellipos depends only on Its semi-major axis @, Find this sengy as. finetion of & "Sia" A'plonct A moves along an elliptical orbit around the Sun Ath tone men 1 wena the dace om the Sa el tity was evel to by and the angle between tho radius vector ry an the velocity vector rq was equal to a, Pind the maximum and tin\- trum distances that will separate this planot from the Sun during Ms opbital" motion. 1.210. A cosmic body A moves to the Sun with velocity vg (when fat irom the Sun) aod aiming paramoter 1 tho arm of the vector vy rolative to th cantre ofthe Sun (Pig. 1.51) Find the minimum di {one by ehh thls body wil et tothe Sun "CBI, A prude of mast mi located outsidoa uniform sphere of co Sie Sa, (2) she potential anergy of grevittioal Inter ‘and the sphere; vey of ih sho ovations fore which th sphere exerts om the particle £212" Benoowinte thatthe qrvietlnel fora sling 08 pare ice inside a uallorm apherlel layer of tater fs equal to seo, ‘fats A parla of moe m was tranatoeed fom the eat of tho dase ofa uillorn hemisphere f mass Mand rods into aba. on of the particle Fig, 458 ‘What work was performed in the proosss by the gravitational force ‘tertad on the particle by the howisphere? ‘244. There isa uniform sphore of mass AF and radius R. Find the strength G and the potential p of the gravitational field of this sphere aaa function of the dstaneo r trom its contre (with <7 Sed'r =. Draw the spprovinate plots of the Tunis 2) aod 00 1,30. inside a waorm sphere of deusity p there iy sprtal cavity whos contre is ata distance [from the contro of the tps, Find the strength G of the gravitational fold Inside the cavity. £216. Aviom sphere haa e mass Mand radios A. Find tho praisure p inside the sphore, caused by gravitatignal compression, iva funcion of the distance r from ite centge, Evaluate p st the ‘centre of the Earth, assuming it 10 be a uniform sphere, ‘L217. Find the proper potential eascgy of gravitational interac- ‘on of mattor forming (@) 2 thin uniform spherical layer of mass m and rading 2 (@) & uniform sphorea! mass m and radius Ft (a to Problem 1-214), 1.219. Caleulate tho ratios of the following accelerations: the accaleration a, dv to the gravitational force on tho Barth’s surface, 6 the accoeration ary due to the contrifugal force of inertia on the Enrth’s oqusior, aad the acceleration wy caused by the Sun to th hodles‘om the Earth. 1.200, At what height over the Barth's polo the fraefall acele- ration dosress by one per oot; by hall? {i2L, On the pole of the Earth » body is imparted velocit ditcctod vertically up. Knowing the redins of tho Earth and the freee {all seesleration on its autface, find tho height vo which the body ‘wil aoeood. The tir drag is be noglocted 282. An artificial satelite i launched foto a eirelar orbit arvund the Earth with veloeity volaive othe reference frame moving Intionelly sad fixed to the Farth's rotation axis. Find from the tatllite to the Earth's surfeco. The radius ofthe Farth and the fove fal acceleration ou its surface are supposed to be known, 288. Caleulate the radios of the circular omit of @ stationary Earths satellite, which remaias motionless with respect (018 sur face, What are fs velocity and accoloration inthe inertial reference frame fixed ata giver moment to the centre of the Earth? 1224. A satellite revolving ine circular equatorial orbit of ra dius A 2.00-10" km from wit to east appoers over a cartain point St the ouetor every T 18.0 hours. Usiag those data, caleulate fhe soace of the Earth. The gravitational coastant is supposed to be knows, 1.295 A satellite rvolves fom ast 1 west in a cizcolar equatorial shit of radius R= 100-40" kan around the Barth. Find tho velocity rnd tho aoculoation of the satellite fn the roferonce frame fixed to the Barth, "208. A satelite must move ia the equatorial plane of the Barth close to its surfas either inthe Earth's rotation direction or against i Find how many times the kinetio energy of tho satellite in the Initereage exceeds that in the former cas (inthe reference f orbit whose process of motion the anellito experiences « sight resistance due to Eosmic dust. Assuming the resistance foree to depend on the velocity ofthe satellite as P = at, where a isa constant find hov long the Satellce will stay ia orbit'until i falls onto the Moon's surface. {Td Calealate tho orbital and astape velocities for the Moo Compare the results obtained with the corresponding velocities for the Earth 1.22. A spaceship appeosches tho Noon, along a parabolic tory which is almost tangent to the Moon’s surface, At the moment of the meximom approset the brake rocket was fired for a short time Interval, and the epaceship was tansforrod into » circular orbit of Moon setellite, Fuad ow the spaceship velocity modulus increased fn the process of bral 11230." spaceship is launched into a circular orbit close to the Earth's ue, What don] wloty ha to be imparted he ‘Spuesip to overcome the gravitational pul? :2B1" At what distance fom the canta the Mogo is thepoint, at whith the strength of the raatont of the, Earth’ and Moon's avittionel Geldet equal to sao? The Earth's mas seemed to fery 2s times thet ofthe Mooa, ad the distance botwoen the oa. feet of thse planets n = 60 thmes greater than the radius of the Earth z 1.282, What is the minimum work that hes to be pecformed to brings spaceship of mass m = 20-10" hg from the surface of the Earth tothe ‘Moon? 1-258. Pind approximately the third cosmic velocity v9, jo. tho ‘minimum velocity that has fo be impartod to a body relative to the Earth's surface to drive it ont of the Solar tystom. The rotation af the Earth about its own axis ls 19 be neglected. '» Bauetin of dynes of wld Body vtatingebot a stationary ais = Ne 33) ‘whey i the alec um ofthe moments of extra ois relative tothe TOD According to Sta Ta tet me 3b) 1 Kineleeneey of «slid body rotating about « salary ais: (ase seat Na nme by extemal fre using the atin af «ald ay an sn a9. (1.54 owes of ti body nae mtn: rote ae i # Reta sts alas momento begun ofa and te moteat N ip betwee the sogula wlecity a of aymacape preenin, ie eee frees! wl] = x (sh 1.284. A thin uniform rod AB of mans m = 1.0 kg moves transla- {onally with aecelerston w--2.0 m/s due vo two aatiparalel forces Fyand F, (Fig. 1.52). The distance hetwoon the points at which these forces aré applied is equal to a = 20cm. Besides, it is known that Fy = 30 N. Pind the length of the rod. 1.285. A force F = Al's Bis applied to « point whose radius vector relative to the origin of coordinates O is oqual to F ~ al Fbj, whore a, 2, A, Bago constants, andl J ae the ualt vectors of 5 the 2 and y exes, Find the moment N and the arm J of the force F Felative tothe point 0. 1.236. A foren Fy = Al is appliod to a point whoso radius vector ty aly while» fotce Fy = BI is applied to the polot whose radius ior ty = Bj. Both radius vactors are determined raativo to the ‘origin of coordinates O, 1 and j are the unit vectors of the x and y “I % le Pig. 1.52 Mig 48 2, A, B are constants, Find the arm 1 of the resultant force to the point 0. 1237, Three forces are applied. to a square plate as shown in Fig) 153 Fd the ods, diction, tad tho poo aplication of te ravultant foes, if this pot is taken on the side B 1238. Find the moment of inertia (@) ofa thin ualform rod relative tothe axis which i perpendicular to the rod end passes through is end, if the mass of the rod Is m and ita Teogth f (8704 hin orm rectangle late lative to he axe ping perpendicular to the plane of the plate through one of its vertices, db, and its mass fs of inertia ‘oiform dae relative tothe symmetry axis porpon- isa to the plane of the disc, if is thickness equal tob=-20 mm find its radios to. = 100 mn; ‘(h) of a uniform tolid cone relative to ts symmetry axis, if the smasy of the cone is equal to m and the radius of ts base tof “£240. Domonstroe that tthe case of a thin ple of arbiteary nape there ie the following relationship betwee the moments of nena: [y+ 1a = Iq, whore subindies {, 2, and 2 define three tmu- {ally perpondiealar 230s passing through one point, with exes 1 and 2'iving ithe plane of the plete. Using tis relationship, find the moment of inertia of a thin uniform round dive of radius # ead mass im relative to the exis coinciding with one ofits diameter. {241."A ‘tlform dise of radius M = 20 cm has a round cut as shown fa Pig: 1.9% The mas of the remaining (shaded) portion of the oquals m~ 7.3 kg. Find the fe to the axis passing through Heular to tho plane of tho disc 1242. Using the formula for the momoot of inertia of a uniform aphore, ind the moment of inertia of @ thin spheriel layer of mast im and radius ft relative to the axis passing through its entce ‘243. Tight thread with a body of mass m Led to Itsend fs wound ‘on 8 uniform solid einder of mass MT and radius A (Pug 1.35). At Rimoment f= 0 the system is set in_ motion ‘Assuming the friction in the axle of the cylin: der to be negligible, ind the time dependence Fj (@) the angular velocity. of the cylinder; {hy the kinetic energy of the whole system. Lbe4e The ends of thin threads tightly wound on the stle of radiua r of the Mexwel aise are attached to a horizontal bas, Whe the ise unwinds, the bet is tnised to Keep the ise atte same height. ‘The mass of th dise With the azle is equal (o\m, the moment of Ihartia ofthe arrangement relative to is axis. Find the tension of ‘tel theead and the aceclertion of the hae 1/245. A thin horioatal uniform rod Al of mass m ond length £ can rotate freely aboot a vertical axis passing through its wad ‘AU certain moment the end H stars etperiencing & constant force qf Fir 155 Pig 8, centre of Inertia and perpone F which i always perpendicular to the original position of the sta- tionary rod and directed ia'¢ horizontal plane. Pind the angular ve locity of the rod asa fanction of ts rotation angle @ counted relative 1 the initial position, 1.248. 'In the azangoment shown in Fig. 4.56 the mass of the v form sold eylinder of radius Mis equal to m and the masces of two Bodies are eu fo and ms, The thread tipping an the fiton inthe aul of th eyliader are supposed to be abents Find the angular acceleration of the eylinder and the ratio of tensions Ty/Ty of the Vertical sections of the thread (nthe process of moto 1.247. In the system shown in Fig, 4.57 the masses of the bodies are knowa to be my and me, the cocligiet of Friction between the body fry and the horirontel plane is esl &, and a pulley of mass m FRfassumed to be a uniform disc "The thread does not sip over the pulley. At the moment # —"O the body m, starts deseanding. Assum- fig the mass of the thread and the irleton in the exte of te pulley to be negligible, ind the work pesformed hy the friction forces ecting ‘om the body my over the fst F seeoads afte tho boginning of motion. "1.248. A uniform cylinder of radius Fs spianed about Its axis (0 the angular velocity ty and then placed fato a corner (Pig. 1.58), Fig 451. ig 188 “The eoeicient of Ietion between the corner walls and th eslinder Freda to How many turns wil the eyliner accmpish before i Dope 15338" a uoitorm dise of din is spines tothe angular vlosty sad then ‘carfolly placed on a horivotsl surface How lng wl fhe dige be otong on the surface f the fiction eoeificlent i equal, {O'i hy prose ext by the dss tha sorfoce cane regeded olor 1250," A fywhel withthe inti dit tothe tres whose moment tlative to tho axis Is proportional {othe square fot of is angular velocity” Find the ten ang Nay Of the ete avenged over the total decalerat Past, ‘A unifoem eyinder of radian W and mae MY en rte ly about 2 setionny horontal axis O (Pig 459). Un cos [Rogth 1 apd mass m fs wow on ho eye fv wing layer Fad thcangolaraecloration of the cylndar as fonction ofthe length Por the hanging pas of the card. The wound par othe cord i sup: jam to have ie sate of gravy onthe eylinder a ‘258, A uniform sphere-of met mand radius Moll without supine dain pon sot aoe tn herent. 1c) the magnitudes of the friction cotctent at which te Sheets * Byte ii ony fe sae ¢ wna afer he oping ot oton FSA tng eae of mu 2c igs 160) starts desconding at 4 moment 1 izovity: Nogleceng the ma of the thread, hd so gular velocity i doccerates ipping 0 ig and radius ‘ive to (0) the tonsion of each thread and the angular acceleration of the cylinder, {d) the time dependence ofthe instantaneous power developed b tat gravitational ore. ne 1.254. Thin thrends are tightly wound on the ons of «uniform sol-eylinderof tava. The fos nds of the Aineads ar attached 0 {hn ceiling of alert cr. Th casas goa up wth mn acl ton wy ind the acclerationw’ of the eylinder oltive to tho ear and the fore F exerted by the eslinderon thecaling (hrowgh the threads). Sond wound ot faced on sailing tages tte kana nf nd hth wall os dona ls Pgs LOM Ibe mas i pci scant ae tte ee dot Blas tertile tile sound teed gee = SO, Find the acceleration of the spool axi ed ° 128: As sl eter af as me ton two horizontal puana'a‘thoad is wound ou the eye’ ie hevaty nd te thread is pulled vertically down with s constant forge P (Fig. 1.62) Find the maximum megnitude of tho force. which still does aot bring about any sliding of the cylinder, if the coefficient of feetion Between the eylinder and the planks is equal to #. What ls the nc- erleration wes of the axis of the eylinder rolling down the inclined plane? 1257. A spool with thread wound on it, of mase m. reas on a rough horizontal surface, Its momeat of Inrtia relative to ite own axis is ‘equal to 7 ~ ymdi® where is & numeral factor, and ty the out ‘ile radios ofthe spool, The vais of the wound Uhread layer is ogul Pg. 8s tor. The spool i pulled without siding by the thread with a constant force F directed aan angle to the horizontal (Pig. 1-63). Find (@) the projection ofthe aceelerstion vector of the spool Axis oa the (Gh) the work performed by the force F during the fist 1 seconds af tor the bopinaing of motion. ‘258. ‘The arrangement shown in Fig. 1.64 consists of two identical unliorm solid cylinders, each of mass m, on which two light threads Pi, 1.04 1 wavind symmetrically. Find the tension of each threat inthe pro- ‘ese of motion. The fiction in the axle of the upper cylinder Is ac- ‘Some to be absont 1.50, tn te arent shows tn Fig. 1059 weight prose mace ma pulley B possesses mase AF- Also known are the moment of inertia of the pulley elative to its axis and the radi ofthe pulley and 2R. Tho mass of the thrwads is negligible. Find the eccelera- ton of the weight 4 aftr the syston is st feo “200, A unforn solid eylindor sof mast m; caa freely rotate about a horizontal ass fixed to. Mount B of mase m (Pig. 1.00). cone it horizontal ToreeP is applied to theend A of» light thread tight iy wound on the cylinder ton between the mount and the sup poring borioatal plano is assumed fo be tb tem f'teconds after the beginning of “L261. A plank of mass my with ioe sir of mash my placed ob Pie 1.0 laze. A constant horizontal fores Fis applied to the plank. With what Sccelerations will tie planke and the centro of the sphere move peo- ided there is no sitdiug botwoon the plank and the spore? 1:262."A uniform solld eylindar of mass m and radius Mis sot in rotstion about its axis wih an angular velocity ty, then Lowered wilh {ta atara surface onto a horizoaal plone and raloased. Tho coolh- alent of Teton tatwen the clacton the plone i eral toe {s) how long the eylinder will move with sliding: (b) the sotal work performed by tho sliding friction fore acting on, the cylinder. 1.263. A uniform ball of radius rolls withoot slipping down from the top ofa sphere of radine R Find the nogular veloity of the ball ‘atthe moment ft hreoks off the sphere. ‘The initial volocity of the Balls negli. 1.266." A uniform solid cylinder of radius R= 15 ex rolls over a horizonial plane passing into an inclined plane forming an angle ig. Lr Pre. 88 = 0" with the horizontal (Fig. 4.67) Find the maximum valve of the velocity, which till parmits the cylinder troll onto the inclined Plano section without a jump. The sliding ls assumed to be absent. s 1.285, A yall body 4 is xed tothe inside ofa thin seid hoop o radius ft and mass eq to that ofthe body. The hop tlle witiose slfpping over horizontal plane: at the stosioate when the hou 4 sts ino the lower position, the coma ofthe hoop mova wih vloeiy (Pi 1.8), At what valor wl the hop move withoat bone 4.256. Determine the kinetic enengy of «tractor crawler belt of tmass if the tractor moven with weloiy 6 (Fig 1.09), ee tak 8 A aon spa ofa nag lt nto de {Fig Oy. Inthe prota he conte ot the: TN tHe 4 sphere moves with voloity slong ile of radios Find. the Kinetic energy of the ie Fs ty of th 1268. Demonstrate that in the reference frame rotating with \s constant angular Yelocity w about 9 Mationnry ais » Eady DT mass m experiences the esltent Go centrifugal force of inertia, Fay — =imortte where Re ie the radius yoetor of i's 'onte of ea eoive to (2) Coriolis foree P.., ~ 2m {vea], where et the velocity of Hebaiys Conne stg Inet inthe rotning een tame 209.8 midpoint ofa thin uniform rod AB of tose m and longth {is rigily fined to 9 rotation tle 00" as shown tn Fig. Pk the fod isso Into rotation with r constant angular velocity Find the resultant moment ofthe centeifugl forces afters felotive ta the Point 'in ‘the reference treme hited to the sxle 00" and to the ro 4270, A conical pendulum, « thin vaorm rod of length and ina: ote nirmy sot» weticl nae vith ngaat velosity sg tela ho ts i ee ‘uniform cube wih edge «restson horizontal plane whose feito cotton cyan ePhecae sat ip motion at ee al ‘eloity, travels some distance over the plane and comes to's stand still, Explain the diseppetrance of the angular momentum of the ‘ube relative to the axis ying 9 the pl ight angles to the ‘ube’s motion direction, Pind the distance hatween the resultaats of fravitational forces aud the reaction forces exerted by the support- ing’ plane, #222. A smooth aniform sod AB of mass Mf and length 1 rotates trosly with an aagulat velocity o, in horizontal plane about a tonssy vertical axis passing through ste end A. A small seo fnass starts aiding along the rod from the point 4. Find the veloc ofthe sisevefelative to the rod atthe moment st reaches ite fond 1278. A ‘niform rod of mass m = 5.0 kg and length 1 = 90 rests on a smooth horizontal surface. One ofthe ends ofthe ro is struck With the impulse = 3.0 N-+ in a horizontal direction perendiculer to {he rod. As'a result, the rod abteins the momentum p=8.0.N-s. Find the force with which one half of the rod will act on the ather in the proets ofnot on {v4 A thin uaiform squere plate with side f and mass M can rotete freely about « stationary vertical axis coineing with one of Hecides A small ball of masa m ying with velocity v at right angles to the plate strikes elastically the contre of it. Find: {a} the velocity of the ball v' after the impact {5} the horizontal componeot ofthe resultant Tones which the axis will exert.on the plate after the impact. 1.275. A vertically oriented wniform rod of moss Mf end lopsth 1 cat rotate about its upper end. A horizontally fying bullet of ass im strikes the Tower end of th tod and gots stack init; a8 2 reslt the fod swings through an angle a. Assuming that m<< Af, finds 2) the velocity of the Nying ballet {0} the “momentaan increment in the system “bulle-rod™ dusing the impact; wat cousts the ehange of that moment, {c) at what distonce +from the upper end of tue rod tho bullot mast stclke for the momentim of the system ullet-rd to remal con ‘ant during the Impact {976s A lorisutally oriented uniform dice of mass M and radius 1 rotates frvely about a sationary vertical axis passing through its ‘oatze. ‘The dige has a radial guide song whieh can aide without rion a smal oy of maa A Lit thea rung down hod the hollow aale of the dise (tied to the body. Loitslly the body twas located at the edge of the die and tho whole system mutated sith lin angular velocity wy. Then by means of a foree F applied to the lower end of the thread the body was slowly pulled to the rotation axis Find a) the angular velocity ofthe systom in its final state: (0) the ‘work performed by the forea 1.277. A'man of mata my sands on the edge of « horizontal uni {orm dise of mass m, and radia R which i capable of rotatiug Teely about @ stationary vertical axis passing through its centce, At a cer tain moment the man starts moving along the edge of the dise: he shifts overs angle rlative to tha dise sn then stops In the pro fics of motion the velocity of the man varies with time as 2°), ‘Accumlug the dlmoosions of the man to be negligible, ‘ind {@) the angie through which the dise had tirned by the moment the rman. stopped: (@) the force moment (relative to the rotation axis) with which the an acted om the dae nth pros of ation 4.278. Two horizontal dises rotate freely about a vertical axis pass- Ing through their contees. The moments of inertia of the dice eelatiy tothis axis aro equal to f, ead Z,, and the angular vlocities and w,. Whon tho uppor dse fell dn the lower one, both dises begat otallng, alter some time, a4 single whole (due io fiction}, Finds {a) the stoady-stato angular rotation velocity of tho dist (3) the work performed by the frition forees ia this procs, 279. A'smalldise and a thin uniform rod of leagth J, whove tase imes grostr than the mass of the dse. le on a smoath horizon lane. The dis is st in motion, 1» horizontal direction and per- Pendicular to the rod, with velocity v, after which it elastically {ilides with tho ond ofthe sad. Find the velocity of the dise and the angy- a la velocity of the rod aftor tho cll Sion. At what value of will the elocity of the aise after the col Blo he equal to sear rovers ih ie 1.280, A stationary platform P wiih can rotate fosly tout a vor Tica axis (Fig-1-72) supports motor A Tr a a and @ balance ‘weight Ns Taw mo imeot* oft inertia" af the platform trth the moto an the Balance weight Me Felative to this axis equal oA Tight fame it aed tothe motors aft witha uniform sphere A sotat- ing rely with en angutor velocity Wp boot n sft BPP coined. Ing with the axis OO" The momest of Inet of the sper reltivo {o'the rotation axis ie equa to Ty. Find ‘ performed by tht motor in turning the shaft BE? thtough 9 thou 18% (hth oa of extra oe which mais the axis ofthe ip the vores! postion afer the motor tare the shalt Hi farough 0 * a ‘2st. Av horiontally oriented uniform rod AB of mass m 1:40 kg and length 1, — 100 om woates freely about stati vertial axis 00" pesing throws ia end. The points located atthe middle of the axis 00" whose length sequel to = 59 ce A what sngala velocity of the rod the horizontal component of ths force aeting onthe lowes ond ofthe atin OD" is equal to rea? Wat is in this case the horizontal component of the foros acting on the Sppe.end of the ak 0 ‘The middle ofa uniform rod of mass m and tenth {1s vig {aly xed to. aerial ania OO" so that tha ngle botwenn the rod tnd the axis is oqual to 0 Geo Fig. 1-71). The ends of the axis 00" are frovided with buecings. The system rotates without fection with Eigular velocity @. Find {E) the magnitaie and direction of the rod Mrelativeto te point C, a wel its angulor the rotation aa “o) how much the modulus of the vector M relative to the point Snereass during a halfturn: (3) the momont of esterual forces W’ acting on the axle 00" in roca of rotation. "B.A top of mass m = 0.50 kg, whose az silted by an angle 6 ~"90" to the vertical, processes dus to gravity. The moment of fnogtin of the top Tolative to its symmetry axis is equal to 7 — ‘2°32 gam the augur velocity of rotation about that axis equal io o 350‘ the distance om the pat of rst to the conte of inertia ofthe top is! = 10 em. Pind: ia) th angular velosity of tho top's precession {3} the mapnitole and direstione¢ the horzontl component of the linn rn netigom the lop a the pnt of ets {Bei A gyroscope, uniform fis of edigs 5.0 em ot cot of rout ongth i cng. 17, owed onthe or Stan elevator cor going ap with 9 constant ace tration = 20m The ather ent of the Tod {S"hinged atthe point O- The gyroscope prec fern nith an angular velocity, nm —08 tps Négustog the iietion snd tho mas ofthe rod, find the proper anguler velocity of the dis. 1.283."A top of mass m= C0 kg nd. moment of inertia rative toate own asin T= G0gem™ ng 4 Spiny with an gnyular velocity Gm P'5f0 nnd. Tts point of rest is located on a Dlock which ie shifted {ns horiaonial direction witha conatont aceration e = 1.0 mis ‘The distance between the point of rest and the contre of inertia of the op equals = i0'cm. Find the magnitude aud divection ofthe an. {Euler velocity of precession 1285 'A oniform aphere of mass m — 5.0 kg and radius Ft — With an angular velocity @ = 1290 radls about le passing throug its cent ad fixed om tho mount ing base by sean of bearings. The distance hetwoen the bearings gusts ? — 15 cm, The bases aet in rotation about a vertical ais ith an angulay velocity o” - 50 radi. Find the modulus and dt- Feetion of the gyroscope forces, 1287. A exlindrienl disc of gyroscope of mass m — 15 ke sadiue + = 510 cm sping with am angolar velocity 0 = S80 Falls angular momentum mentunrelative ot ‘60 cm ‘ hortzon ” ‘The distance between the earings in which the axle of the dise is routed is equal to I= 15 em. The axle is forced to osellate about ‘@horieontal axis with period 7. ‘.0's and amplitude oo Find the maximum value of the gyroscopic forces exerted Dy tho axle fn the bearags. 1.268.'A ship moves with velocity » — 96 km par hour along en srcofaeirle of radius #1~ 200m Find the moment of te gysosenp- fe forces exerted on the bearings by the shaft with 2 Dywhtol whose ‘moment of inertia relative to the rotation axis equals 1 5,810" kgem® and whose rotation velocity m= 300 eps. The tation axis is oriated along the length of the ship. 1.280. & locomotive is propelled by a turbine winose axle is parl- lol to the axes of wheels. The tubine's rotation disetion colneldes with that of wheels, The moment of inertia of the turbine rtoe rel ative to its own axis is equal to J 240 kg'm*. Find the adaitionsl fore exered by the gsrosepic forces on thy rll mhen the lomo Use moves alge # lo of radio 250 m with velocity '50 tem por hour. The gouge is equal to velocity of the turbine equals n= 1500 7pm, 13m. "Tho snes 1.8, ELASTIC DEFORMATIONS OF 4 SOLID poy 4 elation botnon tena (omprsive stain «and azn 0 ss) hee 8 Younes mod. ss llaion tnt lates compreive (ian) etan e” and longitu nat sane (compre) ai 1 (10) bce is Pointn’s ati, “sllatio betmen soa tein y and tango WG, 8) here 6 i shear modal, ‘ Compretiy 15) (4 1.200, What pressure Inder to keop It feagth constant on raisiug it temperate by 100 °C? 4.291. What internal presure (inthe absence of an external pres- sue) cn bo sustained (a) by’ glass tube; (b) by a glase spherical Dose iff both cases the wall thicknoss fs equal to Ar = 1.0'mm and the radius of the tubel and the flask equals» = 25 mm? 1.292. A"horlzonally oriented copper rod of Yength 1 = 1.0 m fs rotated about a vertical axis passing through its middle, What [s tho number of ps at which this rod raptures? 4.295, A ring of radiga r= 25 can made of lead wire is rotated bout a stationary vertical axis passing through its coutre ant per pendioular to the plane of the ring. What Is the aumber of rpe at Which the ring ruptures? 41.304. A lee! wire of diameter d = 4.0 may is stretched horizon tally between two clamps Tocated at the distance 1 ach other. A weight of mass m = 0.25 kis waspondd from the mi point O of the mira, What will tho reslting descent of the poiat Feta centimetres? 1.295. A uniform elastic plank moves over a smooth horizontal, ave dae to-a constant force Fy distbuted uniformly over the end ce. Tho surface of the end faces equal to S, aud Young's modulus of the material to £, Find the compresive strain of the plank fn the Gieeetion ofthe acting force, 1.206, A thin uniform coppor rod of longth ! end mass m rototes uniformly with an angular velocity @ ta a haeigontal plane about a vertical axis pasing through ouo of its ends. Determine the tension in the rod ¢ function ofthe distance rfrom the rotation axis, ind the elongation of the tod 1,297.-\ sold copper cylinder of length f= 65 om is placed on a horizontal gurfaco and. subjected toa vertical compreasive fares = 1000 N directed downward and distributed uaiforaly Over tho fd face. What will bo the resulting change of the wolume of the jligdee ia cubic millimetres? 1.298. A coppor rod of length Its suspended from the calling by one of ite ends. Plo (a) the slongation Al of the od due to ite own weight (0) the rolativa ineroment ofits volume AVIV. 4.200," A"bae mou of material whose Young's modulus is equal to E and Poisson's rato to pit subjected to the hydrostatic pressure p. Find: a) the fractional deccement ofits volume; {) tho relationship between the comprossbility Band the elastic constants ‘Band ‘Show that Poiss's ratio w cannot oxcaed 1/2 1.300. Ono end of a sel Fectangular girder is embedded into a wali (Fig. 474). Doe to grevity 1 segs slightly. Find the radius of ccarvature of the neuteal layer fo the dotted Tine la the fguro) in ‘the vicinity of the point 0 ifthe length of tho protruding section of Fig, 14 {hfe out = 60m and th hee of ho gir equals TL. The hending of an elastic sod is deseribod by the clase curcepissing through centres of gravity of rods erossectons. AL Salt Bendige the squetion of this carve takes the form Neem et SE, ‘whore 1 (2) i the Bending moment of the elastic foros in the eros- Section corresponding to the 2 coordinate, is Young's modulus, Tis the moment of inertia ofthe cross section relative to tho axis pase” ing though the total Ise | #48, Fig 179 Suppose one end of a seo! rod of square crosasetion with ide «js embodded tuto a wal, the protruding section boing of Tength 2 i$ Leer 0. 2 4 Pip 1. Pig 8 (ig. 1.70). Assuming the mass of the rod to be negligible, find th ‘tthe elastic eusve and the deletion ofthe rod, itis end A moment of the couple Vj (ha fee F eran ong the pau, . A stool girder of length reste frooly on two supports (Fig. £77). The moment of geri of its cross-section is equal to T {see the foregoing problem). Noplecting the mess of the girder and ‘assuming the sagging to bo eight, ind the dection} duo tothe force F applied to the middle of the girder, 1'30s.The thickness of rectangular steel girder equals. Using the equation of Problem 1-301, find the defection } caused bythe ‘wolght of the girder in two eases (a) one ead of the girder Is embodded into a wall with tho length of the proteading section being equal to T (Fig- 178), (b) ute ginder of length 2 rests fvaly on two supports (Fig. 4.780), o 4.304, A steel plat of thiclines has the shape of a square whose side ontalsf with hee 1. Tho plato is rigidly fixed to » vertical axle eS (00 which is rotated with a constant angal ion (Fig. 1.79). Find the defection iy assuming the sagging to be small 1305. Deteraine the relationship between the torque Nand the torsion angle for (a) the tube'whosa wall thickness Ar is considerably less then the tube radius, (i) for the solid rod of circular cross-section. Their longth {, ra- dius 7) nd shear modulus @ ate supposed to be know! re =" o —— * ol 0 Fig. 178 Fig 7. 1,306. Gslealate tho torque W twisting a stl tubo of = 40 m theoagh an angle p = 20° about its axis if the Outside “lametr of the tb are equ t= 30 mm and = 1307: Find the maximum power which can be transmitted by sia ofa el aloft rotating abo ana wih an angler vlocty (= 120 tals, if its length f= 200 cfm, radive ™ the permissible torsion angle g ~ 2.5 1308. A uniform ring of moss m, with the outside radius ra is fitted tightly'on 1 shaft of sadiae ry The shaft it rotated about te fazis with a constant angular acecloration f. Find the moment of ‘asic Torees in the ring 48 4 fusetion of the distance ¥ fom the o- tation axis 1.300, Find the elastic daformation energy of a stel rod of mass rm 3.1 kg stretched toa tenale strain = 10-10" 1.310, A'stelevlindcial rod of longth and radius r i suspended by its end from the ceiling (a) Find the elastic deformation energy 1 of the rod. {@) Detino U in terms of tensile steain Al! of the rod 1.311. What work has to be performed to make a hoop ont of a bend of length f= 2.0 my width h = 6.0-em, and thickness 2.0 tum? The proces i asshined to proceed within the elasticity ange of the materia TIS12, Find the slestic delormation energy of a steel rod whose ne end i fxed und the othr is twisted through an angle @ — 6.0", ‘The length ofthe rod is equal to = 1.0 tm, and the radius to ‘0am, 1318, Tind how the volume density of the elastic deformation gy is distribuced in steel rod depending on the distance 1 from its aula. The Tength of the rod fs equel to 2 the torsion angle to 4314, Find the volume density ofthe clastic deformation energy in eeah water atthe depth of h = 1000 m - HYDRODYNAMICS ‘The fundamental oa mics of isl uld (Eulerian cunt the Dud deat fs the valame density of mam forces (f eet, Spleie ye aint” : SE spate cnt sng my son, Rol nor ing Sn fow pase af vice Re= pot, (120) ‘whee 11s» characters lath, mathe Bald vito. fy Patull’s Lew. The volume of liquid owing tracgh a ccaar tl in oho ™ ‘ sR pry i eT oe) ‘whee and ae the tte ras ad length, Shee betwenn te ideo the fae Sake ow Te tin fe on th spor of is 4 8 he preace der ough fw 29) 1.315. Ion fuid flows along 1 fat tubo of constant erot-ection, leat in aThoiontal plane aid beat an shown i Fig 1.60 ( viow). The fw is steady” Are the precsures and Yelocities ofthe Bul ‘equal at points 7 and 2! What is the shape of the strenslines? 1316, Two manometric tubee are mounted on. horizontal pipe of Varying cross-section at the soctions Sand S, (Fig. 1.81). Find the volume of water owing across the pipe's section per unit time HW the difergoe in water columns Is eguel to Dh. 1317. A Pitt tube (Pig. 1.52) (¢ mounted along tho axis of a gas pipeline whose eros-soetional area is equal to S. Assuming the vi Easy to be negligible, find the volume of ger flowing scroes the Le Fig. 18 Fig 1 section ofthe pipo per vnit time, ifthe diference n the liquid col inne i equal Aky andthe desatiee ofthe liquid andthe ges are snd p respectively. P(Ste" A wide vescel with stall hole in the bottom is lod with water ond Kerosene. Neglecting the viscosity, Gad the velo- iy of the water fom, if ther thickness of the Seales layer to equal 13 hy =—30-em and that of The eronene layer to hy 20 em 4.210, A wide eplindeicalSeswel 50 em in eight is filled. with water and rests. on (abl ‘Assuming the viscosity to be negligible, ind. a ‘what height fom th Botton of the sessed amel Tole shouldbe perforated for the water jel com~ Inout of 1 to hit the surface of the table at the ‘maxima distance “Iyyy from the. Yes Find ta TSS20.7A bent tobe is lowered into a water stream as shown in £89. The velocity of the steam relative tothe tube is oqual (0 b= 2.5 c/s. The closed upper end of the tube located t the eight yo i2 em has a small orice. To what height h wil the water Jet Pig 182 ‘mounted, whose radios Ry > My (Fl the eylinder andthe bottom of te v sity te p. Find the static pressure of the Suld in the clearance os function of the distanesr from the axis of the orifice (and the c¥ fen) tthe height af the uid is equal to A. “i322. What work shonld be done in order to aqusore all water fro horiauatlly nated epinde (ie. 185) dung the tne ¢ by means ofa constant foree geting on the piston? The volume of wa- ter In the cylinder is equal to V, tho crosesectional roa of the or o fice to #, with ¢ being considerably less than the piston ares. The frietion and. visesity-aro negligibly smal 1,828. A cylindrical vessel of height h and bere atoa $ is file with water. An orice of area s <8 Is opened in the bottom of the Wessel. Noglocting tho viscosity “of wi ter, determine how soon all the water sll poor abt of the es, 1334, A horizontally oriented tube AB of length rotates witht constant Singular velocity about a stationary Wertcal axis 00” passing through the end A (Fig. 188), The tobe lod with an ‘eal fini, The end A of the tube is open, the closed end B has a very small orihes Find the velocity of the uid relative 10 the tube ae a fonction of the colomin “height”. 25, Demonstrate that inthe case of d steady flow of an ideal fd Bq {2a} foran into Beret evation. 328. On the opposite sides of a wide vertical vessel filed with crater two Ideal bole are opened, nth having te evar anton Fig 4.8. area S = 0.50 em. The height diference between them is equal to ‘Bk St em, Find the resllant fores of rnetion of the water low ing out ofthe vessal 1.827. The slide wail of a wide vertical cylindrical vessel of height 475 em has a narrow vertical sit runing allthe way down the bottom of the vessel The length ofthe slits T= 30'em and the Width b= 4.0 man. With the slit closed, tho vesol is led with ‘water. Find the recnltantforea of reaction of the water owing oat of The vessel immediately after the silt is opened. 1.328, Water ows out of w big tank along a Lube bent aright an ales the inside radios ofthe tube is equal to” — 0.50 em (Pig. 1.87). The tenth of the horizontal scion ofthe (be eal to f= 22 en. The water ow rate is 0 — 0.50 ites per second. Find the moment ‘of eacton forecs of Qowing Water, acting on the tube's Wale, relative tothe point 0 1.329, A side wall of a wide open tank is provided with « narrow: singe (ig, £85 rough which water ow st. The coastal area of the tube decreases from $= 3.0 em to # ‘water level in the tank is = 48 am higher than q [Neglecting the visosity of the water, find the horizontal component ‘of the force tending to pull the tube out of the tank, Fig. tar, Fig 485, 1.830, A eylindrical vessol with water is rotated sbout its ver- tical axis with 4 constant angular velocity. Find: (a) the dhape of the fre Surlac of the water; (@) the water pressure distribution over the bottom of the vessel along its adios provided the pressure atthe central point Is oquel to P5331, A thin horizontal die of radius = 10 cm ie located withe tn's splits cvlty fled wth ol wo stealy 9 008 B {Gig Pah cloraace etc tele sd the hore plans of the cavity is equal toh ~ {.0 mm. Find the power developed by ‘the viscous fores acting onthe dise whoa Tt rototes with the angular ‘velocity w U0 radia. Tho end effcte are to be neglected. 1.882, A long eylinder of radius Ay i displaced along its axis with t constant velocity vy inside a stationary co-axial cylinder of Tadius fy The space between the cylinders filled with viscous 1ig- ‘ld. Find the seloety of the liquid a8.¢ function of the distance r Irony the ais of the cylinders. The flow is laminar 1889. A fuld. with viseosity 1, fils the spaco botweon two long covetial cylinders of radii ty and R,, with y= Hy The inner esl fader is stationary eile the outer one is Totaled ‘with a constant Angular telocity The Ovid flow (laminar. Taking into account that the frition foree acting on a untt area of a cylindrical surface tf radius is debued by the formble 9 = m2 (2a/dr, find (G) the angular velocity of the Totating Muld as 4 function of ius 1b)" the moment af the friction forces acting on «unit ength of the covter eylinder 11894. A tube of length [and radius M cervies 2 steady fow of fivid whose density isp and viscosity n- Tho uld flow velocity de- Pendson the distance r from the sxisaf the tube as = By (t — 77), Find a) the volume of the fluid fowing across the section of the tubo per unit tie 1D) the kinetic energy of the Muid within the tube’s volume; {6} the feition foree exerted on the tube by the did (4) the pressure diference at the ends of the tube. {3a5. fathe errangement shown in Fig. 1.90 1 viscous liquid wise density is p= 10 glem® lowe along # tbe out of s wide tank A. Find the velocity of the liquid ow, ify hd hy = 30 cm. All the distoncet Care equal 1.336, The eros-wetional radi ofa pipeline decroaos gradually as = remy where a 0.50 04,3 He the distance from tho pipe Kine inet. Find the ratio of Reynolds numbors for two ero-sections saparated, by Az ~ 3.2m, H'397, Wien a sphere of radius r, = 1.2 mm moves in glycerin, the laminar flow is observed if the velocity of the sphere aes nol (exceed Uy 23 emis. At what minimum velocity v of a sphere of radios ry = 5.8 cm will the dow in’ water become Turbulent? The 0 cm, y= 20.0, viscosities of glycerin and water are equal to my ~ 13.9 P an att eepete Couemenee aan Te A cadet tnt siaking in lyin whose viaity intgul ges OT, het ie patina Sasa tar ey MTN tow around tat sphere sil masta asus Ine at in rain the eat no eens Haye ‘be the sphere iismoter ath takes fe 13 te al of dinmeer d = 3.0 mm start soking with stoi oly nse a hap a SOU Hae from the steady-state velocity by n = 1.0%? ¥ home 4.8, RELATIVISTIC MECHANICS 1 oreate contention of ent tte shee I othe proper logth and 6 the proper tne of the moving clock, ‘ont trnaaraton™ owing af 3 moving ick oa ra potest. 7 viewer UF C) fe terval eam ayaa: feet whore the i itaval betwen event {and 2 i the dntane betwens Buh st sbi Gh sven cur = "> Teescloration of (136) Ty Toa sn) + Ratmiviie mate and altvinie moment se) wah ‘whee isthe relativistic momento af the patil, "otal ad Hale enero of lative Band amttt, Paina aoe. 130) The feign fram Ke tcumed to move with fe dame Te FR nme Ae locity Vin the pose saa eaieng od sant sothy Beton he ers a uments of allt pa Bi peteendet, pon VETTE: (oa 1s Whes considering the cllsous of partclo i help to uae the fllow- ing ibanant gam Bhp, as) whee B ad por the total ewer aad momentum oft eae prot to the coli Jn isthe ros ims oft the rater farm oF the rod in this framo bo n= 0.3% lass tha its proper length {Suita e tiengle the prope: length ofeach side equels a, Find ‘he periotar ofthis langle tthe reference frame moving relative toil with a constant velocity V along one of its (2) bisector () sides oveatigato the results obtained at V par = . 259. The velocity distribution of molecules in a bean coming ‘out ot sole n'a val is described by theConcionF()—tee-m Sthore 7s the tomporoture of he ga in ho vente. Find the most Probable values of tho velocity of the meee obtained with the most probable velocity of the maeculoe ia the wee {the kinetic energy of the molecules inthe beam Boo.” Andes ge tonsisting of moocalo of mace m with concen: teston » han temperature 7Usiag tho Maxwell distribution fe: Xoo, Bnd she number of molocuou roaching unit aren of wall fe Sogeseewen 6 and te oral per nit Ue 2404, from the conditions of th foregoing problem ad tho m hero alcnag reaching unit oan f all with iw veloc Inthe interval from 6 to 0 dp per unit tne 2.402" ind the force exerted on particle hy a uniform Geld if th concentrations of these particles at two levels separated by the distance, ak'=3.0\em (along the Held) der by = 20 times ‘Tie temperature of the system is ogual to. 728) K. 2403. When examining the suspended genboge droplets under a microscooy tit averege numbers in the layer sperated by the distance 8 40 um wore found to difler hyn = 20 times, The en fonmental iempersure i equal to 7 — 200 K. The diameter of {ho roplets isd = 0.30 pan and their density oxen that of the ssmounding Ould by Ap 0.20 qien® Plad’Avogadra's number ftom the data heen FiO4e Suppose that isthe ratio of the molecular concentration of hydrogen to thot of nitrogen atthe Earth's stace, while la {he Eoresponding ratio at the height = A0D%Im. Find the ratio whe ‘assuming that the tempersture ont of the height. compan of fo kinds of molecu tnd. ith ma > my Tho eonntralons Of these moteculen sth bottom of the veel av aqua tomy ad my ch molecule is The value obtained with the reeipeoeal of the tively, with nym. Assuming the tomporature T and the ‘sczaloration tobe indepondent of theheight, nd the height at which the concentrations of these kinds of molecules sre eq 20106. Avery tall vertical cylinder contains carbon dioxide 4 cortain temperature T- Astiming the gravitation held to be wn fora, find how the gas pressure on the bottom of the vessel will change wien the gas temperature increases 1 times. 2407. A vory tall vertical cylinder contelas ¢ gas ato tompore tuge 7 Assuming the gravitational field to be uniform. fad the mean valve of the poteotia! energy of the gas molecules Does this valve {epend on whather the ges tonsists of one kind af molecules or of Several kinds? ‘24108. A'horizontl tubo of length 1 ~ 100 em closed from both ‘ends is displaced lengthwise with # eopstant acosleration . The tube contains argon ata temperature T= 330K. At what value of i will the angon_concantrations at the tube's ends difler by n= 1.0%? 2100. Pind the mass of a mole of colloid particles if during their ‘enirifuging with an angular velocity @ about a vertical axis the con fentration of the particls atthe distance m rom the rotation 2x8 le 4h times greater Man that at the distance 1, (am the same horizontal lane). The densities of he particles andthe solvent and to py respectively. 2.140. A’uorizontal tube with closed ends is rotated with come- ‘about a vertieal axis passing through one of Hts ena’ The tube contains carbon diosige as temperature T= 300 K. The Tength of the tube {of 400 em. Find tho value at which the ratio of moleoular concentrations ni the opposite ends Of the tube is equal to = 20. 2411. The potential snengy of gas molecules in » certain central field depends on the distence’ from the feld's contre as U7) — ar, where alas puslive constant. The ges temperature is 7. the eoncen” {ration of molecules at the contre of tho fld me. lod! {@) the number of ‘molecules located at the distencas. hetwoen and 7} fom the contre of the Bel 4) the most probable distenco separa contre of the fl (@) tho fraction of molecules located in the spheres! layer between and rb dr, {@) how many tins the concentration of molecules ia the contre ‘of the feld will chango if the tomperstare. deecoass tits ‘2.412, Prom the conditions of te foregoing problem find (@) the number of molecules whose potential enerey lies within the interval trom U"to + dU: (0) the most probable value ofthe potential energy of « molecule comparo this velue with tho potential encrzy af a molecule located SUH most probable distance rom the centre af the Beld ee? vented, es mat ne te By erent : hers 7, and Te ave th tperatons of the hot an eld Boies eesti. > lnusainnguity 0 eo, et here 0 i the elemactary amount of bet trustee to the wt (80 isa Tisai gusanty Sat vt ot 3 f ean «Feta not Semen ras >a pat eo lation betwoon the tony an the satis weight Oe thermo dyna proba " mee tue, ea whee ks the Sots constant 2.113. In which cace will the efficionoy of » Carnot eycle be higher: when the hot hody temperature fs increased by A, or when the cold Body temper ‘2.114. Hydrogen fused lua Carnot cycle asa Working sub Find the eldcioney of th cycle, far result of an adiabatic expansion (a) the ges volume increases n = 20 times; {B) the [pressure decreases = 2.0 times 2418. AThoet engine employing « Carnot eyclo with an efficiency of y — 10% is used a releigeraing machine, tho thermal M=er¥0irS Doing the same, Find ite refrigerating effeleney EUi6. An ideal gas goes Tisough 4 eyele consisting of alternate lspttermal and adiabatic curves (Mig. 22). The isothermal process proceed at the {emperatures Ty, Ty: and 7. Find the efielency of Buch a cyela, ifn each isothermal expansion the gas volome increases in the same proportion. 2.417. Find the elficieney of @ cycle consisting of two isochorie and two adiabatic ins, if the volume of the ideal gas changes m= 40'times within the eyelo. ‘The working eubstancs is nitrogen. o* 2.118, Pind tho olficioney ofa eyele consisting of two isobaric and toeo ediabatic lines, if the pressure changes times within the eyele ‘The working substance Ison ideal gas whose adishatie exponent i equal ty. ‘110. An ideal gas whose adiabatic oxponont oqualey goes through a cycle consisting of two isochorie and two iobarie les, Find the {EWheloney of such eycl, ithe absolute teeperetare of the gos rises ta times both in the tnchorie heating adi the iubarle expansion, y ‘2.120. An ideal as goes through eycle consisting of 5 (i) ocborle, adiabatic, and isothermal Hines, 1h) lobaric, adiabatic, and isothermal Hines, Bi with’ Uhe isothermal process proceeding at tho minimum tomporature of the whole eycle tae abso Fig 22 Ite temporsture varios fold within the cycle 2.121. The conditions are the some as in ihe foregoing problem ‘with tho excoption that the tothermel procese proceeds at themaz num temperature of the whole excl. ‘2122, A ideal ges goee through 8 cyte consisting of isothermal, polytropic, and ndiabetic Lines, with the Isothormal proces proceed ial the mazimum temperate of the whole cycle: Pind the efhe~ cha eyee i the absolute temporatre varios fold within feacy of the evel 2.483 "An Leal ga wit the adibae exponent yous thoush «a diroot(lockoriae) eyele consisting of sari, and isocho- fie Tins: Find the eiieneyof te eyle i inthe adlabtie proto ‘Me vote of the leat gas os mol (decreases mel cla tseftetency of a cyle conseting of tothermal, isobar, sn ivehors lines, io the aothermel process the volume of the Wal gas withthe adiabatic exponest 7 (@) ipeeasos rods (b) docronss nfold, £25" Find te etitioney of oyele consisting of two isochoric and two ioibermal Hine ithe vlume varies 010 ond the aboolute Wampereture fold within the eyele. Tho working substance Isa {deal gee with the adiabatic esponcst 7. idk. Find the elciency of eyelecoftting of to tobare ana two tothermal Tins if the pressfe varies fol and the absolute lamperatore fold within he eyle, The working substance tan IGS eh he alata epatot . E12. an lien gus with the niabate exponent gous through a eyete (igs 2) whi, whieh the abuse tempersire vars ‘Hold Find'the ecieney of this eye ide. Making use of Ue Clauss inequality, demonstrate that all cycles having the same maximum temperature Toe and the fame minimum temperature Ty are less eficient compared to the Carnot eyele with the same Pines ond Trin 429. Making use of the Crrmot ticorem, show that in tho caso ‘of « physically uniform substance whose tate is defined by the part ‘meters 7 and ¥ (ULV) y = T (pl0P Ve ‘where U (7, ¥) is the foteroal energy of the substance. Instruction, Consider the fainitestmel Carnot cyete in the variables pov 130. Find the entcopy snerement of one mole of carbon dioxide when ts absolute temperature increases n'~ 20 tines i the process Br heating i (a) isochoric: (b) isobaric. ‘The gas isto be rogarded as ideal. 243k, The entropy of ¥ ~ 4.0 moles of an ideal gas increases by 45°= 48 17K" due to the otherms! expenaion. How mony times Should the Volume 'y —4.0" moles of the. ber increased? a 2.132. Two moles of an ideal ges are epoled isochorically and then expanded ‘sobarially to ower the ga temperature buck to the inital vel vue. Find the “entropy increment of the gas iin this process the gas prossure changed "=3.3 tines, 2-189, Helium of mass m=1.7 gis expmnied —_|// adlabatically 2.0 times and then compressed isobarically down to" the initial. volume. 0 v Find the entropy increment of the gas in this ig, races, 234, Pind the entropy increment of v = 2.0 moles cf an ideal gas whose adiabatic exponent y = 1.30 if, ‘result of cortaln. process, the gas. volume increased —'2.0 times while the pressore dropped. B= 30 times 2 moles af gaseous helium Js" volumes VylVy = c= 2.0, and tho ratio of ello in thom Ty, = p= 13. "find the diflrence af gas entropies in the absolute temp ‘Aseuming the ato be idea these vessels, Sa — She 2.196. One mole of ar ideal gas with the adiabatic exponent goes shrowgh a polytropie process as a result of which the ebsolute tom- perature of the pas inereases fold. The polytropie constant equals Find the entropy increment of the gas in this procass ‘2437. The expansion process of v —~ 2.0 moles of angon prococds so that the gas pressure dnereases in direct proportion ta its Wolume. Find the entropy increment of the gas in this process provided ite volume increases = 2.0 times, 2.198. An ideal ges with the adiabatic exponeat y goes through f.fraen 7 py ma wee py tnd ro position constants find Vis the volume, At what volume will the gas eutropy have the axlguin value? 2.199. One mole of an ideal gas zoos through 2 procass in which the entropy of the gas changes with tomperatice Tat 5 = a7 + Gy In Jy whore a is + positive constant, Cy is the molar heat capacity of this gos at constant volume, Find the volume dependence ofthe gas temperature tn this process if T= Ty at V = Vy 2.440. Pind the entropy increment of on mole af» Vau der’ Waals gas due to the isothermal variation of volume from V, to Vy. The Van ler Waals soetectine aro assumed to. be known, Zatdt. One mole ofa Van der Waals gas which had initially the ithe temperature 7, was transterred 0 tho stato with ature Py Find the comesponting frentent of the gas, assuming its molar heat capacity Gi to, be. known, 142. At very low temperatures the heat ‘equal to C = a7, where asa constant, Find theo iva function of temperature. in this’ temperature Interval. ‘2:43. Find the entropy ineroment of an aluminum bar of mass rt the tomperature 7y = 00 Kup re interval the speefc eat capac Juminum varies as ¢-— at 67, where a= 0.77 Iig-K), a (8K). 2.464, In some process the temperature of a substance depends on its ontropy Sas F'—~ a8, whero a and m are constants. Pind the ‘correspoading heat capacity €'of the substance ae a function of $ ‘Atwhat condition iC < 0? ‘2.145, Pind the temperature 7 as a function of the entropy 3 of e substance for a polytropte procass in whieh the heat capacity of the substance equals The entropy of the sbetance is known to be foqual to Sy at tho lomperaturo Ty. Draw the epproximate plots 73) for C0 and C= 0. 2.146. One mole ofan idesi gas with heat capsclty Cy goes trough ‘process in which is entropy S depends on T a8 8 =P, whore is'a constant. The gos temperature varies from Ty to 74. Find ) the molar heat capacity af the ges 8 8 function of fs tempe {) the amount of hoat transforrd to the ga (6) the work performed by the ges. through 2 cycle within which 1s showa 45 (a) Fig- 24a: (H) Fig. 24b, whore 7's the absolute temperature, and $'the entropy. Find the cifiency of cach yale 2.AK8. One of the two thermally ingulated vocals interconnected by a tube with a valve contains v ~22 moles of an ideal gas. TRE oles eal is Gvacaned. The valve baving eon opwned. the ae Inca a volume m= 8.0 tines, Find the etopy ierement of the as 2.499. A woightles piston divides « thermelly insulated cylinder {nto two ogual parts. One part contains one mole of an ideal gas ‘with adiabatic exponent, the other ix evscuated. The inital gas Temperature is Ty, Th piston is released and the gas lls the whi dlaplaced back to vi fe neon of he tera oery a Ue entropy of the gay resulting fom theso two procs. 2480. Rn ideal gas wos expended trom the inital steto to the volume V without suy heat exchange withthe surounding Bodin Will the final gas presare be the same inthe case Of (0) fast and in the case of (b) a very slow expansion process? 245104 thermally dovelsted ved fe pa so that the volume of one partis = 20'Umes greater than that af the other The stale part contsins yy ~ OAD male of aitrogen, and the greater one vz == 070 mole of oxygen. The temperature of the fact tthe samo. A bole is Punctuned Im the partion an the gases Ste mised, Find the corresponding increment of the system's entTpY, Ssuming the ges 0 ea 0 wait saa Pisco of copper of mass my wlth initial too perature f, 297° tn placed into a calorimeter in'which th water St mast my = 100 gis ate temporatize fy 7 °C. Find the entropy {ment ofthe afstem by the moment the famperatares equ ‘he hoot capaciey of the calorimeter itil is neplgibly. small 88, Two tential thermally insulted verde intereonnected by a tubo with a valve contain one mole of the seme Idoal gus each. Te caress in one westel is equal to 7; and tn the other, Ty. ‘molar es capaci a ho ge a ont vue oils Cr ing Been opened the ges comes to new equim isa e'Ctopyteromont 48 of tho gu. Domes tat aS > 0. E454, 27 stoms of gaseous helium are enclosed in 8 cubic vessel of volume 1.0 em at room temperature. Find: {a) the probability of atoms gathering in one half of the vessel {B) the approsimate numerieal valu of ensuring the ozcurrence ‘of ths event within the time interval #10 years (Ihe age of the Universe) 2.155. Pind the statistical woight ofthe most probable distribution of H~ {0 ideatical molecales over two halves of the eylinder's Volume, Find also the probability af such a distribution ‘2196, Avesel contains V molecules of an Ideal pes Dividing rmontally the yess! into two halves A aad 2 find the probability that the half contains x molecules. Consider the cases when N= 5 and n=O, 4, 23.4, 3 2437. A yoasel of volume Vy contains IV molecules of an ideal agar Find the probability of m molecules getting ato w cortatesopar {Gf part of the vessel of volume ¥. Examine, in particular, the case vv, is under standard conditions. Find the diame- ter of the sphere within whose volume the relative Quctuation ofthe umber of moloctles is oqual to = 10-40". What 1s the average Duimber of molecules Inside suche sphoro? ‘2190. One mole of an ideal gas consisting of monatomic molecules, fg enclosed int vessel at a temperature Ty — 300 K. How many ines and tn what way wil the sets) weight of ays (gas) vary if itis heated Usochoricaly by AT = 1.0 XK 25, LIQUIDS, CAPILLARY EFFECTS Aina plas pomee aa Noel mde a ahr sce (talieas ormel tree (ot aan wens othe aurae tain af nen au ihe tanya of te stage lier of a igi oral es where Se the ama increment of the sulate apr, te mont of beat equa fo form ssn rw of he Lig arae layer tui fhe Intbermal nooo of sure a ese 2.100, Pind the capillary pressure (a) in mercury droplets of diameter d= 1.5 yan {b} inside soap Dubble of diameter d'~ 3,0 min if the surface tension of the soup water solution fe @ = 49 mN/m. ‘.IGK. Tn the bottom of a vessel with mercury there is @ round hole of diameter 270 ym. AU what maximum thicknese of the rmoresry layer will the liguld still not fow out through this hole? 2.462. A vessel Hlled with air under pressure py contains 909 bubble of diameter d. The sir pressure having been reduced isother™ increased wold. Find the surace tension of the soap water so 23463. Find the pressure in ana locsted ia water at's depth h— has the standard value py 2.164. The dismoterof& gas bubble formed atthe bottom of «pond is d'= 4.0'hen. When the bubble rae to the soroce ts diamet Ipcenesm Sit mee Find how dupe the goed at hat sot ‘imozpharc prasure i stondand, the gos expansion fs assum to be taothermal.” Zee ‘2465. Find tho diforonco tm helght of mercury columns in two commusteating "vertical capillaries, whose. diameters are. d= 10.50 mm and dy = 1.00 mm, ifthe contact angle 0 — 158 2.106, A vertical capillry ‘with inside. diameter 0.50 mm i submerged into water so that the length of its part protrudiag over the water surface Is equal to = 2hman. Find the curvature radios of the meniscus 26h. A gle capillary of length 7 bubble of diameter d = 4.0 pm, .0-m. The atmospheric prosure 10 mm and inside alae Tong's haste capilary tobe uborged to mae te wae evls Inside and ovtalde the capillery.cninlde? 2.468, When a vertical eapillay of fength £ with the saleé upper ‘end’ was brought Jo contet eth thesurfoce at lid, the level ‘of this Liguld rose to tho height A. The Tiguid density is, the ins ‘inmotor ofthe eapiary tad, the contest angle ts, the stmospherie rai pg Find the pric tenon of 21468, A giass rod of diameter @ — 13 mm ts Inserted sym ametrealy into s glass capiltry with iside dlameter dy = 2.0 mm ‘Thon the whole areengoment is vertically oriented’ and brought tn contact with the surface of water To wha height will the water ria {nthe capiiry? ‘E470. Two vertical plates submerged parilly ina wetting liquid form. a wedge with a very small angie 69. The edge ofthis wedge is vertical” The donsty of the liquid fs, is surface tension ia the fontart angie is 8. Find the helght 0 which the liquid tse, a2 fanetion of the distance + trom tho edge. ZA. 4 vertical water jet Hows out of 8 round hole. One ofthe horizontal sections of the jet has the diameter d-—2.0 min ‘while the other sotionToeatad = 20 tm lower has the diometer which ie'n=.3 times loss Find the volume of the water fowing from the hole exch second 2A72. A'wvator Gop fall im aie with » uniform velocity. Pind ‘he ‘ifrence between the curvature radit of the drops surface at {upper an Tower pont of th drop aepratee hy the eaense A mercury drop shaped a= a round tablet of radius R and thicknos his leeted between two horizontal glass plato. Accum: ing that h A, fina the masem of weight which has to be placed oto diminish the distance between the plates n-times, 8. Calculate mit R = 2.0em,2~ 0.38 m0, Fl the ettreon fore between two parallel las plates scparted by acdbtance b= 40 mae a weer Gop of must PEWS og'vas intedueed beiveen thew. The wong @ esd 1 Fe cnmpite aire las discs of radius = 5.0. om were welled with water td fu together fs he thchnes of wate lope be ats falar fet tan Autti eting toe colet, find the foree that has io be applied at right augls tothe plates in Shen's gull aap Lin Bo veri prallel glass plates ar partly sbrenged in gute, Te diane aimed the pisos 8 O40" mms tad thateliuh iol 212 cms Assuming! tt the water belwec the pistes doeot reac heuppar cognate porn the the vetng Es Staplsie tad the fos of fei totaal aitstn 27a the Mun of «sop bale of tales coneced vi he atnoatier tnegh's pny of Ing 1 and se Tells the'sttae lesion lathe lta onic of the oe Hal a vetcl explllary is brought in contact withthe water soriace: Woat auiunt of Kia (liberated wile the water sss it tapas the ating ie une a cots, the etna ely 3B Bnet een of the syage layer of {o's motery opie St dlanaur dE a 1B a soap buble of lametr d= 6.0 min If haute tension ot\the SP water anaion equal to a= 65 maa 280d the iment of ibe ice energy at The osc layer wien toe ental meter relee cach af amet de tsa Serre maker Shad e"yor to be perfomed in ondr to Blow a sup bubble of rednn A te otic air psn i out py nad Use eae tason of th Sonp wate afaion eal to 5.188 A soap buble of ratios Fined whan een. ‘Tues: pre te often te snp ee Bilisioe wteridl in dutaee betncen the motte hea epucty Bt i fos dat ating nde bbe api ek Sag ath go er oan rer 2 Chstering th Cast eptoas apie 0 fgld stow tha a ta olheroal pres the seta af hat cole fot oration ets wal axe af ihe ses ayer a ogo veal. re dali? temporada ae ace tea 2.484, Tae surace of « sep fl was incrsmd sothermally by ag as wagers T. Rovbiog the ras tenon atthe at) sro solsion 24nd the omperstare omfcent dala, nd the Beeneet ) of the ontropy of the Bla’ aac lager 18 Ot the Intra “ooergy of the race Tagen. 25, PHASE TRANSFORMATIONS slasons tates a5 cinch Se os lsc 4 Watle constants an the paratas of the a - ee 2185. A satoratod water vapour is contained in « epindrcal esl under a weightless piston at a tomperstare # == (00 "C. At Ses of dow ittedueson ofthe plson sso faction ofthe Tepour Am’ 07d g gets condensed” What amount af wore Was Perlormed over the gar? The vapour is assumed to be ideal, the ‘Volume of the lguid to be neseced 2406. A vescl of volume ¥-=" 6.0 contains water together with pour 50 Uk Under these conditions, The total mast of the system water-vapour bauale m= 5.0 kg- Find the mass and the voluue of the vapour NR” The saturated water vapour hezeone ma einer ender 1 piston snd oceuples a volume V, 5.01 at the temperature ¢ 00 °C. Find the mass of the liquid phase formed after tho volume Under the piston decreased ienthermally to Vm 1.8 1 The saturated ‘apour is assumed to be ideal 2188. A'volume occupied by” a saturated vapour is reduced iso- thermally n-fold. Find what fraction othe final voluane is oeeupied by te asd phase i he gi ume ofthe eat vapour and the liguld phisso differ by times (N= n). Solve the same problem under the condition that the final volume of the substance Eorresponds te the midpoint of horizontal potion of the Isothermal Tine inthe diagram p,¥- "2180. An amount of water of mass m ~ 1.00 kg, boiling at stan dard stmospherie prestur, uses completely into saturated. vapour. ‘Assuming the saturated vapour to be an idesl gas find the increment ‘of entropy and internal energy of che syste. 2.190, Water of mats m—20'¢ is enclosed in a thermally insulat- fd cylinder at the tomporature of O°C under a weightless piston ‘whose area is S210 em®- The outside pressure is. equal to Standard atmospheric preseure, ‘To what fing will the piston rit hon tho water ehsprbe Q ~ 20.0 kJ of heat? 1. One gram of saturated water vapour is enclosed in a therm= ally insulated cylinder under a weightloss piston. The outside pros Sure being standard, m'— 10 g of wator is introduced Into the tl. inder at a temperature fy = 22°C. Neglecting the host capacity of he cylinder and the friction of tho piston against the eyliader's walls, find the work pecformed by the foree of the atmospheric pres Sure during tho lowering” of the piston, 2192. If an additional prossure ‘Ap of 9 saturated vapour over « convex spherical surfac of a liquid fs enosidersbly leet then the ‘vapour pressure over a plane surface, then Ap == (oy) 22ir. where ond py are the densities of the vapour andthe liquid. isthe sur- eo tension, andr Is tho radius of turvetare of the surface. Using this formula, find the diameter of water droplets at whieh the eat rated vapour pressure cxeoeds the Vapour prostate aver the plane Surfaco By y= 10% at a temperature ¢ 27". The vapour is ‘sumed tobe an ideal gos 2.198, Find the mass of ali molecules leaving one square centi- metre of water surface per second into a saturated water vapoti above i at a temperature f= 100"C. Tt ia assumed that m= 3.0% of all water vapour molecule felling on the water sutface are retained Jn the Tiguid phase 2.196, Pind the pressure of saturated tungsten vapour at a tem- perature 7“ 2000'K ifs tongeten filament i= know! to lose amass 2.10-% piis-em?)" from a unit ree per ‘unit time. when ing into high vacaim at this temperate, 195. By what maguituce would tho pressure exerted hy water op the walls of tho verse have Inceeased ifthe Intermolecnlar atese~ tion forces had vanished? 2.196. Pind the itor pressure p, of liguid if its density 1p and specite latent heat of vaporization g are known. The heat 17 is asm to be equal to the wark performed against the forces St the intemal prossure, and the liquid obeys the Van Ger Waals fauation. Calculate py in water. ‘2197. Demonstrate that qs. (26a) and (2.6b)_ are valid for « substance, “obaying the Vander Waals equation, in critical state Tnstruction. Make uso of the fact thatthe eritical state corresponds to the: poiat of inflection in the isothermal curve’ (V). 2.498. Calealate the Van der Waals constants for carbon dioxide Per = ADK K and critical pressuny poy = 2.109. Find the spose volume of beneone (Cs) inertial st sie ideal temperature Pop 902 Rend cea ream por = i tm ‘2200, Write the Van der Waals equation via the reduced pare motors x.y, and 1, having taken the eorresponding critical values forthe salts of pressure, volume, and temperavre, Using the equa- ton bind dh any tie he ga temperate ernest tritial temperature Hf the eve ponsue ie 12 ie as high as ere frossur, ad the volume of geri equal to halt the ert volome ‘2301. Knowing the Van fer Waals constants, Bad {G)tie'marimum volume which water of mess m= 1.0 kg ean ‘ecupy’ in Tiguid states {@) the, makimom, pressure of the saturated water vapour 2202" calculate the tempersture and density of eatbon dioxide in ciical state, assuming the ges to be a Ven der Waals one, 2203. What tration of the volume of veel must liguié ether ‘oceupy at room trsperaine in onder toass into eitical state when sriiel tompersture se reached? Bier fa Tec itt Ky pay 385 atm, 2 Moi gimol ‘2204, Demonstrate that the straight lint 7 conenpondng fhe sition cute [and TT are equal. (Fi ho, “Wnat fenetion of wa percooled down. (0. the tompe R30 Cader standard peste {urns ito ioe when the system pases Into the equilibrium stata? AL what temperature ofthe supereoled water fle it turn into ir completely? 06. ® increment ofthe Sn melting, temporeture inthe vicinity of 0° when the pressure is ineressed by Ap = 1.00 atm. ‘The speelic volume of fon exceed that of water by'AV’ 0,00 cag. 3.207. Pind the specie volume of saturated water vapour under standard pressure if desroae of posure by ap = 3.2 Ke is known io'decrease the water boiling temperature by AT = 09 K. 2b Ansuming the satareted water vapour to be i ing preseure atthe tempereture 1011 °C 2.Boo. Acmall enwont of water and its caturled vapour are ene ‘lose tn a esel ava temperature = 100°C. How much {in percent) Sei'he nase of the sutured vapour incease if the tomperature of aby AP = 13°K? Assume thatthe vapour is an rus and the spetie volume ot water lv oglgible a8 compared fo"that of vapour 2210, Pind the presture of saturated vapour as a fonction of temperature p (2) Hat « temperature 7 its presure equals Pe Fig. 25. * Assume that: the specific latent heat of vaporization gis independent of 7, tho speciie volume of Higuid is negligible as compared to that ot vapor, Started vapour obeys te equation of sat for on eal fnvestigats under what conditions these assaplions are permis- Es Hing the entropy increment of the system. Tae = 90°C wat placed ins calorimeter in which ice of mast 0 g as at a temperature —3°C. Find the entropy increment of the piece of copper by the moment the thermal equilibria is reached. 2216. A chunk of lee of mass my = = °C'Waa placed ins eal = 100g was i) was poured into eslorigtor packed witha large amount of ie ata tompeatare ty ~ O°C. Find the x ‘py iserenent of the aysiem lealtee by the moment the thersal Squisbriom is reached. The apectic latest heat of melting of esd i ual oy ~ 225 Fig ond Weopeciie heat capasity i equal o.€ = 0498 Vien), 2N8, Aveter vapour Bling the space under the piston ofa eylin det‘ compressed for expan) that St romeine setae al {he time, being just on the verge of condensation: Pind the tolar fea capacity €of the vapour fn chia roca as funtion of tem Yerstare 7, taming the vapour tobe ln ideal ens and neglecting Uhe'specthe volume of water tn companion with that of vapout iain © at a tmperatue = 100° 2.210, One mole of water boing in equilibrium with a negligible amount of its saturated vapour at s tomperature 7, was completely Converted iato saturated vapour ats temperatore 7 Find the eat ropy increment af the system. The vapour is assumed to be an ideal iis; the specific volume ofthe liguid is negligible in comparison with that of the vapour. Relay Visas numbar of pa moles tovering the distance without cl. iy & where Ae tbe masa foe pb te ee path of gn molecule = ‘Vind’ oe ihre di the allactive damoter of = ealecle, and m i the oucber af mole "2 Cuan of ese D,vnosty m nd ost conductivity of gan: Dadun andtidn x=diien ate) regu deni, and cy ase Beat capeciy st coostant vlan. ‘Prlcton force acting om & unit reo plates doing it mation parallel te oak atari highly trod gos Fe Lomein al emu) sehr at ya th oaciin of the plates Sebo a'e hero tnned rtoen two wll by My tedomina tn arm tes fad Fam tempt he ale 2.220, Caleulete what fraction of gas molecules (@) traverses without collisions the distances exeaeding the mean tee Da a (b) has the free path values Iying within the interval from 2, to 2.921. A. narrow molecular beam makes its way into a vesel filled with gus onder low pressure. Find the mean five path of wole ules if tho boa inteneity decrees n-fold over the istence AL 21392, Let aat be the probability of # ges molecule experioncing 1 collision during the time interval df; is a constant. Find: {@) the probability of « molecule experiencing no collisions during the time doterval {(b) the mean time {aterval between successive collision tnd the ee ee path nde ean ime interval be toeeon successive collisions of gaseous nitrogen molecules (@) under standard conditions 10 (b) at temperature ¢ = 0°C and pressure p = 1.0 mPa (ouch 2 pros- ure can be reached by means of contemporary vacuum pumps). ‘2.224, How many times doos the mean free path of nitrogen mole- cules exoeed the mean distance between the molecules under stan- ‘ard conditions? 2.985, Find the mean free path of gas molecules under standard conditions ifthe Von der Waals constant of this gus is equal to “40 ano : 2.226. An soouttie wave, propagates through nitrogen under stan ord conditions, At what feeqvoncy will the wavelength be equal forthe mean. free path of the gos molecules? 2.207, Oxygen ie enclosed at the temperature O°C in a vessel with the characteristic dimension f — 10m (this 1s tho linear ‘ilmension determining the character of 2 physical proooss in ques tion). Find a) the gos pressure below which the mean free path of the mole- ee 1G) the. corresponding molecul distane® between. the molocules. ‘228, Por the ease of nitrogen under standard conditions find: (@) the mean number of collisions experionoad by each wolecale per second {b) the tots] numberof collisions occurring betweon the molecules within f tm? of mitrogen por second 2.229, How does the mesa free path 2 and the numberof collisions ofeach molecule per unit time » depend on the absolute tomporature ‘ideal concentration end the mean () an isobaric. process? 2.390, As’ sult of tome process tho pressure of an ideal gas fnereases nfold. How many times have the mean free path ?. and the number of calliont-of each molecule per unit time v changed fand how, if the process Is (a) dsvehorics (B) isothormal? 2st. An ideal gas consisting of rigid diatomic molecules goes through an adiabatic process. How do the mean fre path } and the hhumber of collisims of each molecule per second v depend in this process on {the volume V; (b) the pressure p (2) tho temperature 7? Sroaz" au ideat gos goes through a polytropie proozws with ex: ponent n. Find the meen free path hand the number of collisions of fch molecule per second v as a function of i ho olue. (hha prem 7 (9) the temperature. 1233. Determine the molar Beat eapecity of a polytrople procass ‘through which an {deal gas consisting of rigid diatomic molecules oes and'In which the number of collisions between the molecules constant ‘olume; () in the total valume of the gos. 408 { aod 3 by's thin heatinsulating’ partition with two Holes."One the gos i kept ata temgerataron ites higher than thot of fort cient 1 of an ideal gas change if its volume increases n times: 2200." dal gus goes through a palytropic proces: Find the {yo long eousialeylingers, the mean radi of the cylinders fs equal it El = 6 mm? a f we of adit fy snd Ry, With Fy = My The obter oyinger rotates with 2.246, Two identical parallel dises have a common axis and are located at wdistanco from cach other. The radius of each dise ts fsa To, with a > he One dice rotated with slow angular velo fio tolsive to the other stationary, dize. Find. the: moment of feletion fore acting on the mationary aise if the Viscosity coat lent of the gos between the dises is equal {0 Z219. Solve the foregoing problem, assuming that the discs are located a an vltrareretied gasof molar mass My at temperature 7 fan under pressure. 2.206. Making use of Poisuilo's equation (17d), find the mass soi gos flowing fer wit tne through the pip of length Land adios {iSconstant presures py and py ant maintatned at is ends. ERG"Ogtnd ol anid thermally naltiogseah, {is kept al a temperstare T, while the other, at Py The 10d is com poved ot two sectios whow lengths ace fy and fy and heat condvet Ty coflciats my and. n, Find the temperature of the terface, 3.218, Two rods whone length ar land fy and heat conductivity confcnts vy and xy are placed end to end, Find the heat conduetivity outiclent of a uniform rod of length Ql, whove conductivity {Stho sate ts that ofthe sytem of hss to rods, The latval sustacs ‘ft rode axe assed (oo thermally insulated 21060. od of longt Twth thermally lasulatod Tatra surace consist of material whose heat conduetivitycoettileat varies vith fmperature as 4.-> Ts whore 3 fs 4 constant, The end ofthe rod Sve hopt a temperataces Fy and Ty, Pind the fanction T (2), whore F'ie"The ‘distance from the ead? whowe temperature is Fy. and ti heat dow density 3.250: Two chou of sta with heat expactios C, and Cy ate Anteronnected by a rod of longth {and croseseottona arcu Sand {Misly ow heat condvetivty The whole system Is thermally ast- Toted from te environmeat, Ata moment f="0 the lomperatore Alorencr between the two chonks of metal oquals (AT),. Assuming tho het capacity of te rod to be aeligible fd tho temperature Aiferonce botwoon the chunks av a function of time 251. Find the temperature distribution ina tubstance_placod between two parallel plates kept-at temperatures 7) and Ts. The plate separation is equal tof, the heat conductivity cofhiibat of the substance x 2° T. "2.232, the space botwoon two lange bosizontal plates is filled with bein, The plate separation equals 1 — 50 tm. The lower Plat is kept at a temperature T 200 K, the uppery at 7, “Sho K. Pind he hat flow density il the’ gas pressure Is close to, standart 25358, The space between two large parallel plates separated by a dstenoe 5.0 smn ie ed with heli under a pressure p= “H10'Pa. Ono plate Is kept at's temperature fy — 17 and the perature fy =37 "C. Find the mean fre path ofhetivm the heat Hox density to {vo coaxial cylinders of rat yaad Zybiled with Sanit eat aie Sai oe sry ee ac eres mae Aloady-state temperature ate surface ie equal to Be PART THREE ELECTRODYNAMICS f Stongth and potential ofthe feld of a plat char « 1 # Nolation betowen felt strength and pote Be v5, an) ‘fal strength i gust the aie ‘Shunt that and crcl ‘he Sector ron Gesmoim §eeno ate Pond sgh fhe fo ont dil wih ci me oy ome gt we say ten 9h ne wen he ea WN ot foray sting 0 the dipole = etre Ged, andthe moment ie a1 Force acting ona dipole, and is projeton Fy whe an ‘Rhea the daivatie ofthe vector wth rept othe diple retin, ‘PET ili gratin of He feos By 8.1. Calenlate the ratio of the electrostatic to gravitational inter action forces between two electrons, between two protons. At what ‘ale of the specie charge gfm of particle would these Torees be ome equal (in thelr absoluto values) tn the caso of interaction of ‘Meatical particles? 3:2. What would be the interaction force between two, copper spheres, each of mass 1 g, separated by the distance 4m, ifthe otal ‘electronic charge in them dillered from the foal charge ofthe auclel by one per cent? ‘3:3. Two stall equally charged spheres, each of mass m, axe suspended (rom the same point by silk Useads of length 1. The distance hetwren the spheres z-<- Find the rate dg with whieh ‘the change leaks off each sphere if their approach velocity varies as bo all’ a, where is a constant 3.4. Two postive charges q, and, are located at the polats with radius vectors, and ty Pin. & nogave charge gy and a radius vector fy of the point at which It has to be placed forthe foren acting on ‘heh of the three charges to be equal to 22 ‘3.5. A thin wire ing’of redivs r'has an electric charge q. What will be the increment ofthe foroe stretching the wie 2 point change {eis placed at the ring's centre? 6. A positive polat charge 30 uC is located in the plane zy at the polnt with adios vector t= 204 9), where Land} ate iho unit vectors of the 7 andy ates: Find the Yeetor of the electric field strength E and ite Imagaitude. at the point. with radius vector ‘Sj. Here ry and are exprecced in Point changes g and —g aro located at the vottiees of 4 aquare with diagonals 21 as shown in Fig. 24. Find the magaitade of tho oleetele field strength. at » point located symmetrically ‘with respect to the vertices of the square at & Aiatanco from Its centee. ‘38. A thin falling of radius J — 20 em is uniformly charged with a total charge = 0.70 nC. Find the magaitude ofthe electric Feld strength at the corvature centre of this half-ring. ‘3.9, A thin wire eng of radius rearros a charge q. Find the magni turd of the elestete eld stength on the axis of the ring as a function ff distance U fom its centre. Investigate the obtained funetion at Vor Fina the maximum strength magnitude and tho correspond- tng distance 1° Drath sporriaate lt of the tanction 3 0. A point charge q is located at the ceatre of a thin ring of radius Ft with uaitormly distributed charge —q. Find the magnitude St the electric field strength vector atthe potnt Tying om the axis ing at a distance from its contre, tz > 2 Asti consis of thi haat we ng of radi and a very long uniforaly charged thread oriented along the axis ot the ring, with one of fe ends coinciding with the contre of the “rhe total charge of the ring Is equal to q. The charge of the vd (per ust length) is equal to. Find the lateraction force bee tween the ring andthe thread, "312. A thin nonesadyeting ring of radius has a Linear charge density A = hy cos @, where hy is a constant, is the aimuthal tangle. Pind tho magaitode of the electric field strength fa) at the eeatre of the rag: (6) om the axis of the riag asa function of the distance 2 from its cout Tametgne the obtained anetion at 2 > TE M13, A thin steaigie rod of length 2a sarrying a baiformly dist booted charge 7 loeatod In vacuum. Flad the magnitude of the 106 electric fold strength as function of the distance + from the ros entre along the straight line (@) perpendicular tthe rod. and passing through its centes {b) foineiding with the ro’s direction {ot the polats ving outside tne roa ete ig Tnvomignte the obtained sxprstions at ro, SA Avery long. sraiht oformly charged thread carries 1 charge 2 per unit length. Fiod the magoitude sad direction et tue ect Reinga» ont iis na a dlsance 7 fom the thread and lis on the perpendicular passing through one of the tread nd a A read carrying a uniform change 2, per unit length has the configurations shown in Fg. 32 a and b, Adsuming' curvature Fig 3.2 Figg. radia M to he considerably lost than the length of the thread nitude of the elecieie Sel stengt atthe point (x A spre of radios» castes surface change of deasity ¢ = ‘ar, whore a isa constant vootor; and ia the radius vector of 4 point of the sphere relative to 8 centr. Pind the eecsic held Strength vector at tre sentey of the spe. 55.45, Suppose thesuface charge donsty Over 4 phere of cada depends om n polar angle 0 as & ~0y co 0, wher oy fs 0 postive estat. Show that such change distbution cam he iepretnted as syiegl of smal lina two ually stage ale St tadius Pt whove charges are quel In-magnitode ana apposite in Sign. Resorting fo this representation, had the rece Bld strength ‘eetor Inside the given sphere 38, Pid the oles fed srongth vector at the cetee of ball of radius with volume change density p= ar where tea conlant ‘eclor and ris radius vector drawn from the all's entre, ‘340, Avery long uniformly charged thread oriented slong the agi face of rn reno cnt wih one of th el ne charge ofthe bread pet uoit length i equal th, Pind the ux Of ie tion Ears the cee ren 1.20, Two point charges g'and —-g are separated by the distance 21g, 33) Find the Mex ofthe electric eld streng vector across 2 Sale of radius EB. A ll of radius 2 Us uniformly charged with the volume density p. Find the Nox of th slectete Sed strength Vester actos the bal’s section formod by the plane located at a distance ry < J? from the. contre of the Dall. "3.22. Each of the two, long parallel threads carries 2 uniform ‘charge h por unit length The threads sre separated by a distence Find the maximom magttude of the electric field strength fn the ymmetry plane of this system located between, the threads. ‘ZEN An’ infinitely loog eylindical siface of circolae. eros section is uniformly. charged I oa, cos q, where pis the polar angle ofthe cylindrical coordinate System whose axis coincides with the axis of the given surface Find the magnitude and direction of the eleelei field strength vector fom the # ou. 2, dete eld seas depends only on he and coe dinates according to the Jaw E =a (al ya" (where a {s'n constants Land j are the unit vectors of the 2 and\y axes, Find ve fof the vector B through sphere of radius U with is cenee tthe origin of coordinates. 3:25."A"ball of radius M caries positive charge whose volume density depends only on a separation r from the ball’seeatre a8 papett wilt), whore py ist constant. Assuming the permittivities Se the bull and the environment to be equal to unity, hud (a) tie magnitude of the electric eld strengths «function of the distance r both inside and ovtade the ball (bythe maximum intensity Eye, and the corresponding distance rw. Sibne‘ATeystem consists ofa Dall of radios eareying spherially symmotiio charge and the surrounding space flled with a charge of Nolume density po lr where «in a constant, is the distance from the cvntre of the ball Find the all's charge t which the ma nitude ofthe eleetric eld strength vector is independent of r outsige the ball: How high Is this strength? The. permittivities of the ball find the sorrounding space are assumed to be equal to on '22T- A'space is fled up witha. charge with Volume density pga where pg and 2 are postive constants, ris the distance from ‘the entre of this system. Find the magnitude of the electric field strength vector asa function ofr Investigate the obtained exprex son for te small sod large values ofr a- at a <1 and arf “28: Inside a ball charged vaiformty with ‘volume density p ise phocical cavity: Phe come of the eatin dspced with co the contre the ball by a distance a. Find the ld strength Einside the cavity, assuming the peraitivity equal to onity. 4028. Iosde an fadately long sienlareylinder charged unitoraly with volume density p there v4 a circular cylindrial cavity. The Aistance between the axes of the cylinder and the cavity is equal {044 Pind hy lati Geld srg Eni the cavity. The permit tHeity iassamed to be equal to unity. ‘3:0. "here are two hin wire rings, ech of radius fy whose axes ince, The carges of the rings are g and —y. Pind the poteatial Aliference between the centres of the rags separsted by m distance a 108 wthwise with the surface dexsity 3.31, There is on infinitely long straight thread carrying « charge with linear density b= 0.40 wCim. Caleulate the potential diference betwen points 7 and 2 if point 2 is removed m= 20 times farther from the thread. than polbt Z "382. Pind the elocele Geld potoatial and strength at the eantre fs bemiphere of radi Sharpe uniosmly wit the sac lenaity "385. A'very thin round plate of radius & earrying a uniform sur- face change density o it loeated in vacuum. Find the electric Held Potential and strength along the plates axis as function of a dis: Etace ! from ite contre. Investigate the oblalaed expression at [0 and >A ‘SS, Find the potential g at the edge of a thin disc of radius carrying’ the uniformly” distributed change with srlace densi= we 3.35. Find the electric feld strongth vector if the potential of thie eld has the form @ ar, where a Iva constaat vector, and F [s the radius vestor of a point of the field. 3.36. Determine the eleetsie eld strength vector ifthe pote cof this field depends on =, y coordinates 42 ag =a a @) om where @ is a conttant. Draw th slag lines of foree (la the x, y plane) ‘3.37. The potential of « certain electrostatic field has the forn pate") + Oct whero a and 6 are constants. Find the mag~ ‘itude and direction of the lect feld strength vector, What shape have tho equipotential surfaces inthe following cases: ()a > Op >O; (by a 0, bc OP S38, charge g 19 utformly dsuibuted over the volume. of 4 spieze of radius fl, Assuming the permittivity to be @jual to-unity throughout, fad” the povential {a) at the centre of the sphere: {() fnsde the sphere ass Function ofthe distance r from its conte. 3.39. Demonstrate thet the potential of the fold. generated. by 4 dipoie with the electric moment p (Fig- 3.4) may be represented = pridxey?, where tis the. radius vector. sing this expression, fad the magnitede of the sloctrie eld strength vector asa function of zy ~ and 0. 1 ‘340, A point dipole with an electsic moment p12 ‘ivection ofthe Tocated at the origin of coord a Projections Ey and £ ofthe feeloe (on the plane perpendicular to th raen re point (see Fig. 3-4). At which points is perpendicular to p? "BAL. A point electric dipole with a moment p is placed in the external woiform electric feld "whose strength equals By, with 08 ptt Bp: In this caso one of the equipotential surfaces enclosing the Sifole"torms a sphere, Find the radius of this sphere. 'Sa2. Two thin parael threads carry a uniform charge with linear densities hand —h. Phe distance between the threads is equal to ‘nd the potential ol the electric Beld an the magnitude of tastrength tector athe dstonce rsp Lat the angle Oto the veetor 1 (Pig. 3.5) ‘5.43. ‘Two coaxial riage, each of radius f2, made of thin wire are separated by a small distance 1 (Fe R) and carry the charges q and se Pind tho electric field potential and strength at the axis of the : a i Ke = system as a function of the z coordinate (Fig. 3.0) Show in the same ‘Hrawing the approximate plots of the functions obtained. Tnvestigate hese functions at [2 > R ‘hid, Two infinite planes separated by a distance cary a uniform surface charge of densities ovand —c (Fig. 3.1), The planes have found costal holes ot radios with Ue 2, "aking the origin ‘Gand the z coordinate atis as shown in the igre, nd the poteotial Of the electric eld and the projection ofits strength vector Z, on the (dees of the system as functions of the # coordinate. Draw the approx mate plot 9 (2) "JA6, An tleciric capacitor consists of thin round parallel plates, each of radius Tt, separated by a distance J (Ure. 1) and uniformly ‘charged with surface densities 9 and —o, Pind the potential of the five Held and tne magnitade of ils szength vector at the axes apacitor as functions of a distanca = from the plates if > Investigate the obtained expressions at z > ft. 'SH6. A pole with an electrie moment p is located ata distance ‘from a long thread charged uniformly With a linear density 1. Fin the foree Fractiog on the dipole if tho vector p is oriented (@) along the thread; (i) slong, the radius vector r: {0 at right Sq. Find. the int separsiod by 4 distance b= 10 nm itheir electric moments are Srlented along the same staight ine, The moment of each molecale foquals p= 062-10" Com. 1343, "tind the potential (x, ) of an clectostatic held B= ali + ai), where a is a constant, {and J are the unit vestors of the andy axes. 249. Pod top potata (9) of an slconaie eld = abet als yf wate totais ta a the vegort at the a aly ase Bg oe bt ww td bare contol 5.3 The Bld potential i tren of pace depends on ate x corinate wy = "eh wes «a9 ucts Fig debt th apace are pC) 5h" nfermlyitiuted space came sup the space be- yt i ee a plate epee Te poten A ithe bneern pte gut fo At wat a fis af soot What wit hen be he all wot at waa, To ad I TAe eld poteata snide a charyd Bll depends only she dane fom cei ang oe hee etn rh cane tthe Pid the apes charge dstation ‘p'7)T hhal 442, coNDUCToRS AND prEnrcrarcs CAN ELBCERIC. FIELD + Hlectie Ged sueap near the srace ofa enductor in vacuums B= alte om Flux of pelaitaton Pacrns case sara: ‘hve inthe lesrae sum of ound charges enclosed By thi src, seh Gow, as where gs the alge sum of etrencos charge inside» cloned surface. > Relations ot tho Boundary betnea te dlc Pam Pinm—0's DexDie% ExmExs 2) pce” and o ry the sich deniin of found and extant charges, and ‘Beit vector wa the normal rrted rar medium ft madi 2 wih letepie dete Poxet, Dock, eo 1b 20 tet wilorm iectie ng wp alte epee B= Bye. aan 3.54. A small ball i sospended over an infinite horizontal con ducting plane by means of an insulating elastic thread of stfiese ‘AS sion as the Dall was charged, It lescenfed by 3 em and is saps ‘aula ftom the plane became qual to Find the charge of te sexu hc 2.5, A point change i Tocated a a distance I from te ifate conducting’ plane, What mount cf work has to be pevlrmed in Sriee to slowly reuove the charge very fart Ue plane SS. Tova point charges, g and, ore separted by Sista f both wing inated at's distance (2 trom the iat conducting Plan. Fina Ma) the modulus ofthe vector of the electric foes seting on each charae “G) the magaitade of the electric eld srongth vector atthe mide volt Hw the shang | ape Aint charge q located betwen two mutually perpendt- cular conducting hal-flanes Tes distance from each’ Salbplane {S equal tof Pind the modulus of the vector of the force acting ‘the chang. "AS8, A"polat dipole with an electric moment p is located at «distance from. inate conducting plane. Fad the modules St the vector of the Tore acing on the pele f the vector pis perpenclealay to tbe pane Sh. A point change gi located at distance I fom an infinite conducting plane, Detertuine the sustace density of changer indveed nthe planes function of sparstn oa he base of tne perpen Teulada tothe play fram the charge. ‘S60. X'thindogaitly long thread earring a charg & per unt length ie oriented parallel to the infinite conducting pate. The Alstance betwen to thread andthe plane is qual fo" Fin! is) she modulus ofthe vector ofthe force soUng on awit length ot he thowndy (b) the distrbation of srtce change density a (2) over the pane, shore isthe distance stom the plane perpendicsla To the condoling Sirface and’ passing through the thread ‘R01, Avery long straight threads oriented at right angles to sn infinite conducting plane; is end is separated. fom: he pane By aigtance sho thfend aris a unifoan charge of linear sky 1, Suppose the point Q is the race af the thread on the plane ita ihe nitface domly of the saduced charge on the plane is) at the point {5} & a tation ofa distance + fom the point 0. ‘342A thin wit ring of rade caries a care g- The cing is crleuiedparalel toa tniait conducting plane nd separated By a distance trom it, Find che surfs charge dani atthe potat ofthe plane symmeteea! with expect tothe ring (0) the sng and tho potential of tho elated atthe contre at the rng {16, Fisd the potential g ofan uncharged conducting sphere 0 side-of whieh" "poi charge qs ocatel at Aistanee fom the sphore's onto 2 3.64. A point change gis located ata distance r from the centre 0 ofan unchanged conducting spherical layer whose inside and outside adit are equal to Mand Mt respectively. Pind the potential at fhe poine O it re fy 3.65. A ‘system consists of two concentric conducting, spheres, with the inade sphere of radius o carrying. positive change What charge , has to be deposited on the outatde sphere of cation b to rodoce the potential of the tne sphere to tee? How” docs the potential y depend in this caw on a distance r from the centre of the system? Draw the approaimate plot of this depeudonee 3.66. Four large metal plates are focted ata smal distanc from one another as shown Ia Fig, 38. Tho extreme plates are Inter — connected by raeans of a conductor witle « potential difference ‘Sp is applied to fateral platen Find {a) he values of the slic eld strength between neighbouring plates {O) the total charge per unit aren of enc plate 4.67. Two Tofinite conducting plates 7 and. are seperated by ‘distance 1A’ point change qi Wcated Between the plates at d tance 1 om plate £. Fisd the charges induced on each plate 3.68. Find. the elecisic force experienced by a change redsced tov unit area ofan arbitrary condor ifthe surface deuity of the sarge equals 0; "360. A metal Dall of radios = 1.5 cm has a charge g = 10 yC. Find the movil he voctora te estan force acting om a che Togsted om one all of the ball 5.0. When an unchanged conducting ball of tadius 1 is placed fn an external Uniform eletee Beld, » siface change density o = cos Ts tndueed onthe ball'ssaraco hare oy In 8 constant, 6 i & polar angle). Find the magnitude of the resultant eletee force fcting on an Induced charge of the same sign ‘SL. An lestrie Geld of strength E 1.0 kViem produces polar ‘ation in water equivalent fo the corectoPiontation of only one out Of molecsles. Find WV. The electric moment of a water molecule foquals p ~ 0.62-10°% Com ‘372. A non-polar molecule with polarizability is loeated at 4 great distance I from a polar molecule with electric moment p. Find the mayaltude of the interaction Loree between the molecules {Ue wet is oriented along stright ine pesog through both molecales 7, nonpolar mogul oad at te asf hi ufos charged ning of radius /tAt what dstanes from the cing’ cere GAUES mptitude of the force Factiog on the given molecule (sheqel to new, 0) maximo? the approsimate plot fy @)- "SFA. A point chargeg is vated at the conte of ball attra ontop detotle with permittivity e~ Find the polar wetion P as'a function of te radive vector r relative to the cane TE ake" system as well ag. the charge inside 8 sphere whose fadjus is les than the radius of the ball S75, Demonstrate that ata disletric-conductor intertace the surface density ofthe dielectrics hogad charge 0” — a (e~ Ae, ‘Mere els the permittivity, ls the surface density of the charge the inde oe ees ‘37h, Aeondusior of arbitrary shape, carrying a change 4 8 surrounded with uniform dielectric of permittivity © (Pig. 39) Fin he tral bound charges a hefner and oater sacs of he te SI gail totape dlc shaped ws phil Iyer with radii a and b, Draw the approximate plots ot the ie a Teor fe dfzitehas'a cvtain positive extraneous change {ioctl etm: teh eral sofce of the layer: (ove the volume of cate 5.28" the point A (Pg 340g the boundary bbw sua ind ue set al ttong a ecnin a Sf'Vnn”ue nie Bawoon the tector Ey an ih gma tot tie bundy Hebel equt ay = 30 Find eel strength Edie Uhe'snite diy of the Bound charges nthe pots A: 0 Nettle eae aracs ot 4 unfor botupe dinette witty Ete dette ll seat i vogum ie ea wana fonming an angle wt he amma of fee Weert eit suming th tow oor bth te {nate thn een ia iy the fot of te vert though a sphere of radio with colle eed tthe sac of ton dilecnes Phe ereuain of the vector D around, the ned path of length 1 (4s Fig- 3-1) whose plane i perpendicular to the wirlace St the dielectric and parallel to the vector Ey Pie 344, ‘280, An infinite plane of uniform dielectie with permittivity ¢ te uovformiy” change with extraneous charge of apecr deslyp. ‘Toe thchnes of the Plate Gs equal to Bl Find: (a) the maznitade of the elacse Bld strength andthe potential as fnetions of distance {fom the middle post ofthe plate (where the potential is stumed tbe equal to tro: Having chosen the £Pondinate axis perpendicular inthe plat, draw ihe approsimate Hiote ofthe projection 2) ofthe vector Hand the potential (es (0) the srfate and space densities of the bound charge. Sa"tatcatesas charges ave unliomlydltebuled with space danaity p 50 over a ball of radius trade of uniform iatopic ‘ieee with permittivity Find {o) the magaite of the secre fold strength as function of Aistance r from the contee of tho ball, draw the approximate plots "68 he Space and surtace desis of the bound te space and surlace densities ofthe bound charges Sihe"A found deicric die of radius 1 and thokoet dis sat ‘cally polarized so that it geins the oailonm polatzaton Pr with the votior P ying ithe plane of the ae. Pind the strength Eat ihe electric Held" the Genre of the tise i'd fe 3.83. Under certain conditions the polation of an inte snchnrged delete pate takes the frat P= By (1-5), where eclor perpeadiesla to the plate, «iste distance fom the late, di its haltthiekoeas: Find the lrvngth oF the electric eld asi the plate and ental diference between = ‘384. Initially the space hetween the plates of the capacitor te fled. with air, nd th fold strength fa the gap ts equal to, Then half the gap is filed vith Lntfonm isotropic dielectee with permittivity ¢ ae shown ia Fig. 9.42. Find the moduli of the vectors’ E end D in both parts of the gap (Fiend 2) Ifthe introduction of the disleteie () does not change the voltage across the plates; {B) Teaves the changes at the plates constant ‘185. Solve the foregoing problem for he ease when balf the gop is Bled with the dislctele tu the way showm in Pig. 3.13. 7 CET, 2.86. Half the space between two concentric electrodes ofa sper ical capacitor is fled, as showa ia Pig, 8.14, with uniform isotropic dilectrie with permittivity #- The charge’ of the capacitor Is 7. Find the magnitude of the eleettie Geld siroagth between the elee- trodes aan function ‘of distance r trom the curvature centre of the lectrodes. ‘57, Two small identical Dalls carrying the charges of the same ign are suspended from the same polat by Insulating threads of ‘equal lengthy When the surrounding space was filled with kerosene the ivengence aagle betwaen the (hreads remained constant. What is'the density of the material of which the balls are made? B.SB. A uniform electric old of strength =» 100 Vim is genee- ‘all made nf uniform isotropic dielectric with ity # 5.00. The radius of the ball is A — 3.0 cm. Phi tmaximom sveface density of the bownd charges and the total bound hallepace. The permittivity of th dielectric egualse. Find (a) the surface deasityof the bound charges asa function of distance + from the point ebarge 4: analyse the obtained result at T+ 0; (@) the tolal bound charge on the surface of the dislecttic. 3.90. ‘Making se ofthe formulation and the solution ofthe foe going problem, find the magnitade of the force exerted by the charges Bound’ onthe surlace of the dielectrle on the point change ‘3.91. A point charge 7 is located on the plane dividing vacwom ‘snd infinite uniform ioteopic dielectric with permittivity e. Fld the moduli of the vectors D-and E a well asthe potential @ as fune- tos of distance v fom the charge ‘3.92. A small condcting ball careying a charge is located in uniform isotropic dislecteic with permitivity tat a distance T from an infinite Boundary plane hetween the dielectric and vacuum. Find the surface density of the bound euanges on the boundary plano function of distance 7 fom the hall, Analyse the obtained reste 6 3.93. A ball-space filled with vniform isotrople dielectrie. with tivity e has the conducting Douadary plane. Inside the diele (va distance Crom this plane, there isa small metal ball pos- sesiag a change g. Find the surface density of the bound changes at the boundary plane a4 a function of aistagce 7 from the bal. “34 A plate of thickness A made of uniform slttally polarized Aieleteie ie placed Inside a capscltor whose parallel plates ae Inter ‘connected by a conductor, The polarization of the disectee I equal ZA ZZ Fig 8, to P (Pig. 2.45). The separation between th capacitor plats is Find She'strongih and induction vectors forthe elastrie eld both Inside and outed the plates 85.14 lang rovnd electric cylindor is polarted s0 shat the vector Pars whore a is positive constant and tit the stance itom the axis Pind the spcedonslyp of Bound cargesana fonction Siidatnee © tom the a 206. A jlectrc ball i polaszed uniformly and. statically, ta polation equals P.Taking nto acount that ball polarized. tn ‘is uy nay te foprsented sel of al shit of sl ponitve Charges of the dielectric relate to (ay Sod the slectse eld sreng {3} camoostate tht the Sold outlde the allt logated atthe centre of the ball the potential of that fe qual to'g = parlnes whore py isthe sestete moment of the ball shdvr te the Qistanc from (6 contre ‘1: Using the slut of the foro problem, find the ee tric it stent in spherical cut i an nts sacl polria- ‘lunorm sear if he dilerticspolarauion bP, and fa om the cavty Oe field seogth "398, A uniform dilecne ball i placed sa a uniform, electee ‘eld of stongth Ee. Under these conditions the dieesive besa Polarited uniformly Pinte clectio eld strength inside the ball fd the polarization Po the dsiecase whowe permittivity equals Mako se’ of the recut obtained tm Probl 598 M398. Ac infnitly long round. dices cylinder polarized silty and statically the polarization belag.perpendiclar {othe saisoltheeylindor: ind tho elects faldarength Eiside the Slates, 400A tong round cylinder made of uniform diletrc is placed ina Mims lil ast The in he Cylinder ls perpendicular to vector Eye Une these conditions {He delcticecomes polarized waiforely. Making ase ofthe rst obtained in the foregoing problem, find the electric field strougth E in the cylinder and the polarization P of the dieectele whose por imittivity is equal to &, 3.3. ELECTRIC CAPACITANCE. ENERGY OF AN ELECTRIC FIRLD = cCapatane of» parle late capacitor: = weld 6.80 Intuctonsouey of + apt of poit cares wat Sean oa + Tote cnergy of» apt with continuous shige dations wtf apa aa ‘+ Toa lace once of to card odie and Wait ee W esa et, Wood rae she altenecen of he hi, ad i he intetion conn. Bong of « charg capt: Waa a 9 4 olune deny of te tld ery a on 3.101. Pind the capacitance of an isolated ball-shaped conductor ot radia 1, surrounded by an adjacent concentric layer of dieleette ‘vith parmittivity e and ovtside radius ey 3.102, Two pacalle-plate alr eapacitots, each of capacitance C, were connected! in series to a battery with’ erat 8. Then one of the apacitors was lle up with uniform disletsie with permititvly # How many. times did! the electric field strength in that capacitor eres What amount of charge Howe thug he balers? 3 he space batween the plates of a paraliel-plate capacitor is filed consecutively. with two" dielectric layers 7 and 2 having the thicknowes d and’ dy and the permittivities ¢ and ey apectively ‘The area of each plate Is oqual to S, Pind: (a) the capacitanen of the ‘capacitor, () the density o* of the bound changes on the boundary pi Is ¥ and the electric Bel rates Lagan in the direction perpndicslar tothe plates ‘The area ofeach plate equals 8, the seperation betweca the" plates ‘is equal to d. Find: : (s)he capacitance of the capacitor; (0) the space density of the bound changes az a fonction of ithe change of the capacitor isg and the field Kin iti directed towand the rowing « values, 5.4 Pld th capacitance of» sphere capacitor whose le tori have radii Hy and 11, > M, and waich is filled with isotropic ‘lelectewe mtooe peril tivity varie aye — al, whegea ie a constant. find rie the distance from the centre of the capacitor. ‘34106, A cylindrical capacitor fs flled with two cyliadrical layers of electeic with permittivities ey and ¢q. The inside radit of the Tayers ate equal to My aad Il, >, he maximum, permis aloes of electric Beld strength are eal to Eye and Bay for these Aielecteleas AL what relationship between Hy end 2 will the ‘oltege increase roll in the ted strength teaching the Breakdown Value for both dielectrics simultaneously? ‘S07. There it a doublelayer eylindrical capacitor whose para seters areshown in Fig. 2.16. The breakilown field strength values for these disletris aro equal to Ey and. Ere spectively. What is the breakdown voltage of this capacitor Mf eVMEy< tall? ‘S08, Two long stenigh(. wires with equal rosesectional radia are located. parallel to each bother in ait. The distance between th fsquals b. Find the mutval.eapacitance of tbe Wires pet walt length under the condition B > o ‘3.199. Along straight wire is Toeated parallal to an infinite conducting plate. ‘The ite evs tonal radius ie equal to, the the axis of the wire and the pl pacitance of this system per uait length of the wine Under the eandi- tion ae “SMO. Find the capacitance of « system of two identical metal bulls of radius of the distance between their centres fs equal to, with 3a. The system is located in a uniform dielectric with permittivity "ALK. Determine the capacitance of a systom consisting of metal ball of radius @ and an intnite condveting plane separated rom the cantse of the ball by the distance Tif 1 a 3.112, Find the capacitance of a system of identical capacitors Detwooa poiats and shovea in (a) Fig 3.8765 (by Fig. 3.17. 8.115. Pour identical metal plates are located in air at equal distances d fom one another. ‘The ea of eaoh plate is equal {0 S. Find the eapsoltance of the aystem between polnts A and # I the plates are interconnected as shows, (a) in Fig, 3.484; (b) ba Fig. 3.18. = B.104. A capacitor of capacitance C, = 1.0 pF withstands the maximim voltage V,— 6.0 kV while ‘a capactior of capacitance 2. ah, the maximum voltage Vy — 40 kV. What voltage ‘wht the system of these two capacitors Withstand if they” are con ‘ecto in serie? "A115. Pind. the potential difernce between points A and B of the systom shown in Fig. 310 If the emf fs equal to € = 110 V tnd the eapacitanee tation CyC, — n= 2.0. petition of the same link consisting of two ideutical capacitors, tach with capacitance C (Pig. 3.20). Fig 20, Pig. 3.2, 3.117. A circuit bas a section AB shown in Fig. 3.21. The emf of the source equals # — 10 V, the capacitor capacitances are equal fo Cy = 10 ph and C, — 2.0 HF, and the potential difereace Gq — wip = 5.0 V. Find” the voltage across each capacitor. S116. Tn a cioait shown in Pig. 3.22 Und the potential diference between the lot and right plates of each capacitor, 120 1h eapacitor in the cieait shown in 3.119, Find the ebarge of Fig. 3.22. ch GS 8.120, Determine tho potential diference ya. — 5 between points A aiid B of the cireat shown in Fig. 3.23. Under What condition is it equal to 2aro? R121, A capacitor of capacitance Cy — 1.0, uP charged up to a rolinge omc HOW anne itu the tomas St a edrenit consisting of two ‘uncharged. capacitors connected in Series and posessing the capacitances Cy — 2.0 pF and C, ~ 3.0 ‘What ‘charge will ow through the counecting vires? "3122, What charges will flow after the shorting of the switch Sarin the circuit ifuseated in Fig. 3.24 theough sectious 1 and 2 inthe directions indicated by the arrows? : eer = att feet TET ‘ 4 woth Tt ag re te a eh ty ‘8124. Find the poteatial tilference py — gy between potnts Avand B of the titeait shown in Fig. £2 it ™ aA ye Late CE Gee eit shown in at the point O to be equal to zero, 8.125, Determine the potential at point 2 of the ig. 3.27, assuming tho poten Using the symmetry of the formula obtained, write theexpressions for the potentials. at poists 2 and 3 "3.126. Find the eapteitance of the circuit shown in Fig. 3.28, Fig 328, 3.127, Determine tho intoration energy of the potot charges lo- cated at the comers of a square with the side-¢ inthe clzeaits shown In Fig. 3.29, 3.128, There is ap infinite straight chain of alternating changes 4 and “~g. The distance betwen the neighbouring changes is egal fora Flea’ the interaction ‘energy of ach charge with all the there Tnstrution. Make use of the expansion of ln (1 +a) in @ power series Ine ‘3129, A point charge q is located st s distance fom an iafoite tonducting plane. Find the interaction energy of tha change with ‘hose inden onthe ple "i190, Calculate the Interaction energy of two balls whose changes 4 and ay ate apherieally symnmetcical ‘The distance between: the Ssatres the Balls is equal to natruction. Stare with nding tho teraction energy of a ball and this spheres!" Tape. SA." capucitor of capacitance Cy ~ 1.0 pF carrying titally a Voltage ¥'= 300 Vis connected io parallel with an ackanged Eapacitar of capacitance Cy 20 uf. od the increment of the tlectrc eerzy af thi system by the momeat equilibrium is Teached. Explain the result obtaived. 3.182, What amount of heat will be generated inthe cient shown {In Fig. 3.30 after the switeh Sw lashifted trom position Ito position 2 och 7 e Hig, 3.0. Fig 834. 8.198. What amovt of heat wile generated fn the eleatt shown in fig. 2.3 after the switeh, Sw i shifted from position 7 to posi ton 2 “S134. A system consists of two thin concentric metal shells of adit Hy and My with corespoodiag charges, and gy Find the alt= luce WV, and. W of each shell, the interaction energy of ud the fotal lectre energy of the sys rae q is distributed uniformly over the volume of ius #. Assuming the permittivity to be equal to unity, {a the electrostatic selLenongy of the ball: (8) the ratio of the energy W, stored im the bell to the energy Ws pervading the surrounding space ‘136° point change y= 3.0 ui located at the centre of «sper orm sotropie dielectric with permittivity «== 80. ‘The inside radiuy of the layer is qual to ¢ =~ 250 tam, the outside radius is 0 500 em, Fd the clecrosatic enecgy teside the Sleleeeie layer SA1J7. A spherical shell of radius Ry with uniform charge ¢ is expanded to'n radite My, Pind the work porformed by the electric forces fo this proces, "3198. A spherical shell of radius A, with a uoiform charge g has ‘point charge g tits caatre, Find the mork performed by the elee= tele forces dosing. the shell expansion from radias Mh to $139.8 charged with erie shell ts usitormly 7A surface derity &. Using the (4 EA feaergy conservation law, find the magoitude ff the electric foree acting on a uoit area of the shel 3.40, A, posat change, Lsaad he cenize 0 of ¢ spherical uncharged conduct fnyor provided wi w small orice (ig. 332) The fside and outside rai of the layer ate equal tog and b respectively. What amount of ‘work has to be performed to slowly transfer the change g fom the Point O through the orice and ito iataity? 3.141, Bach plete of a pavalleplate sir capacitor has an area S. ‘What amount of work has to be performed to slowly increase tho Alistance batween the plates from 7, 10-4 if {e) the capacitance of the enpaitor, whichis equal to, or () 1 voltage scrote the eapncitor, which fe equal to’, is kept constant [the proces? ‘3.442! Inside a pacallel-plate capacitor there is » plate perailel to the euler plates; whoes thiekne ia agual to n = 0.80 of the gap ‘width, When the plate is absaot the capacitor capacitance equals ‘P= 20 nF First, the capacitor was connected in parallel to a cons tant voltage souree producing V = 200 V, thea it was disconnected fom Wafer whe the plate was alonly remove from the ee Find the work performed Baring the removal, Ifthe plate Is (a) made (@) made of lass. S15. palpate capaiior wos Tweed fa water yor iaontal position, with water filling up the gap between the plates 2 tam wide. Theo a constant voltage Y= S00 V was applied tothe capacitor, Find. the water pressure iacrement in the a 44, A parallel-plate capacitor is Ioeated horlzontelly #0 that cone of lig plates ia submerged into liquid while the other le over its firtace (Fig. 3.33). The permitivity of the Tiguld is equal toe its density t= equal to p. Fo what height will the level of the liquid fn the enpacitor rise ater it platas get a charge of surface density 0? wr Mig, 338 Mig. 336 3.165, A cylindrical layer of dicletric with permittivity is fagerted into a eylindrical capacitor to fil up all the space between the electrodes, The mean radlus of the electrodes equals R, the gap between them is equal to d) with dc. The constant voltage V is applied serost the eletzodes of the capacitor. Find the magnitude sf the electric Toree pulling the dielectric into the capacitor 3.446. A capacitor eopsists of two stationary plates shaped as fa sonpeirele ol ratiug A and a movable plate made of dielectic trith permittivity © and capable of rotating about an axis O between the statioonry plates (Fig. 3.34). ‘The thicknese of the movable. plate [equal tod whichis practically the separation between the station fry plates, A poteatig! diflerence "ts applied to the capacitor. Fina’ the magattude of the moeat of fores relative to the axis O fcting of the movable. plate in the position shown in the ‘gue 234, ELECTRIC GURRENT 1 Obs law for an inbemagenous segrent ofa lc 8 Be. where gis te voltage drop 708 the segment ‘> Dilrnsiat fora of Ohm's Ine de aie my aan) shew B® is the seneth of «Bald yrodvol by exirancout fret, Tae (or an lode ea Baa, Dake Dee aw 1 Power P of curt and theaal power (mvt Fd O=aP 4s) “+ Spossepomer Py of coront and pai thermal poe 0, Pepe ELE Op 0F (0) caren dont bn tal: — aan wate isthe avera vlosty of casors 1 "Noor of tone eeomning por anit vale of pa pr anit tims ane ie) hore + 6 tha roeombizatlon esate, au 3.107. A long eylinder with uniformly charged surface snd eros sectional radius @ = 1.0 om moves with a constast velocity S10 mis along its ati, Ao slecteie eld strength at the surface of the cylinder ieequal to B = 0.9 kViem, Find the resulting conve toa eareent, that Is, the current cansed by mechanical transfer of charge 8.148. An air eylindrical capacitor with a de voltage V = 200 V aappited across it is being submerged serticaly nto ¢ vesel Bled sath fy 0 = 5.0 ms. The electrode of the capacitor fre separated by a distance d—2.0 mm, the mean curvatoreradias ff the electrodes is equal to r = S0 mama, Find the current lowing fn. this case "along lead wines, if dr. . 7 8.190, AU tha tamperatura 0° the electric — rslatance of conductor 2 is q times that of | 2 Conductor 1. Thoir temperature coeficients of ‘elstance are equal to ay and 2, respectively. Find the temperature conficlont of resistance Is ‘f'n ciruit segment consisting of these two fonductors when they” are connected ’ (pa sees) in 4 190. Find the rsatanco ofa wiee frome TE S3% stapod oe a cube (Fig. 38) when measured betwees.polats (a) EF; th) 18, (13, ‘Tho resistance of eech edge of the fame is H 3.451. At what volue of the resistance Hy in the circuit shown in Fig. 9.30 will the total resistance between points A-and 5’ be Independent of the oumber of eels aaa ‘PEE 3.152, Fig. 3.37 shows an inGinite cizeuit formed by the repetition of tive sue liok, consatiog of resistance 8, = 4.08 and fy = 3.00. Find the resistance of ths cireuit between points 4 and B. pot liye lee ly Fig ast. 8.158, Theve is on jainte wire grid with square cals (Fig. 3.28), ‘The resistanee of each wire between neighbouring Joint connections is equal to Ry Find the tesistance 7 of the ‘hole grid beiween poiots, and B. Tnatructions Make tse of principles of Sem 28d Sor homogenents poorly conducting ile imetitum of resistivity p Alls up the space al Tretween two. thio coaxial ideally conduct- Ing "cplinders. ‘The radil ofthe eylioders tte equal to a and b, with av<:?, the length teadh eylinder is E- Nepleting the edge fects, find the resistance of the mediom Fig. 3.98. Dotan the eyliodes ‘S155, A'mgtal ball of radius ais surrounded Dy a thin concentric rmotal sell of radius b. The space between these electrodes is fled Up witha poorly conducting homogeneout medium of sitivity 9. Flod the tesistance of the interelectrode. gap. Analyse the obtained folution at be, ‘3.108, The space between two conducting concentric spheres of adit and (a'=) fled up with homogeneous poorly conducting ‘dium, The capacttance of wich a system equals C. Find the resistive Ty of the medium if the potential diference between the splieres, ‘Shen they are. disconnected from an external voltage, decreases vlold during the time interval Af. 3.457, Two motal balls of the same radius a are jocated in s homo- gencous poorly vonductiog medium with resistivity p. Flad the sistance of the medium between the balls provided thatthe separ tom between them is mich greator than’ the radius of the ba ‘158. A metal ball of radius aie located at distance ! {rom ah tate aly Codtig lnm, The apace oc te Ball led ‘sith « homogeneous poorly conductiog medium with resistivity. In the cave of o <0 had (a) the current Gensty at the conducting plane as function of Aistanee'r from the ball the potential difeence between the ball fd the plane is equal to: (the electeie msatance of the medium between the Dall and the plave ‘3.150, wo long parallel wites are located in a poorly conducting sodium vith resistivity p. The distance betweet the axes of the ‘siren ft egal to, the crssection radius of each wice equals, Ta the case @ 2.1 Bnd: (a) the enoreat density atthe poiat equally removed from the axes ot the wires by a lstaace rif the potential diflerence between the ‘vires ig equal to V5 (b) the electee Tesietance of the modium per unit Tength of the ‘S160, The gap Betwsen the plates of « parallel-plato capacitor ts filed with glase of resistivity p= 100 Oem. The capacitance (ol the capacitor equals C = 4.0 af. Find the leakage currot of the Sapacitor when a voltage V~ 2.0 XV i applied to i Biel, Two conductors of arbitrary shape are embeded toto an {nftts” homogeneous poorly conducting. medium with resistivity rand permittivity tind the value of product Yor this system, ‘nhere Ais the teistance of the medium between the conductor, ‘mediom M162, A conductor with resistivity p hounds om a dleostse with permittivity At certala point at the conductor's surlaee the Electric displacement equals D, the vector D beiug directed awsy from the conductor and forming an angle with the Bormal of the surface, Find the su of charges on the eondvetor at the point a ind the cusrent density inthe conductor ip the Sleloity of the same point. ‘S163. The gop betwoen the plates of parallel-plate capacitor fs filled up with an inhomogeneous poorly conducting medium whose fonductivity var In the direction. perpendicular to. the plates from oy, ‘2 pSimns Each plate has an {rea § ~230'em", end the separation between the plates ig d 3.0 com. Pind the current Bowing through the capacitor due to @ voltage V = 300°V. 3.184 Demonstrate that the law of refeaction of direct current tines at the boundary hetwoen two conducting media has the form tan ay'tan a — oylow, where oy and oy ate the conductivities of the media and ay are the anal botween the Curent Ines and the Dorma of the bondary ‘suriae ‘S65. Two cyliadries| conductors with equal cross-sections and siete eesintvitiespy and pg ate put end tend. Fad the charge atthe bonadary of the conductors ita curent 7 lows from conductor Two conductor 2 ‘3.106, The gap between the plates ofa parallel-plate capacitor is Ale up wth wo dilctrie ayers 1 and Bvt iekonte dyad lp permittivities e, andy, and resistivities py and py. A de voltage Viisepplied to the capaitS, with etic fad desta fom layer to layer 2 Find 9, the surface density of extraneous tharge atthe Doundary between’ the dislectic layers, snd the condition under wich 9 ~ 0 ‘3.167, An ishomogeneous poorly conducting. medium fills up the space between plates 7 aad of a parallelplate capestor. 1 Deraitivity nnd rasstivity vary. frm valuen ty py at plate Z to alos ty. fy at plate 2A de voltage is applied fo the eapacitor through ‘wich « steady current I flows from plate 1 to plate: Pind the total extraneous change in the given medium, ‘$168. The space between the ploten of paral 4s led up ith inhomogeneous poorly cont Fesistivty wares Ilneary ia th direction perpendislar $0 the plates ‘The ratio of the maximum value of vesatvity to the migimam ‘one is equal o.n, The gap width wquals€. Find the volume density ‘of the change in the gap ita voltage ¥ is applled to the capacitor. ‘Tis ausmed to be Leverphere ‘S160. A long round conductor of eosescctiona atea 8 is made of matecial whose vesstivity depen only on a distance frm the ‘xi of the conductor asp — ar, where a is « constant. Find {G) the esstance por unit length of seh n conductor {8 tn elect ld strength tthe eet dont whic on rent I fows tThtongh 'u70, A enpacitor with capacitance C= 400 pF is connected via'a resitanee A= 030 © to a source of constant vollage Vo How soon will the voltage developed across the capacitor reach & value TO. Vee SAMI. A capacitor Slo with dielectric of prmittivity «= 24 lowes half the Shange aclsed during stim (terval t 3.0 mi ‘counting the chang to Teak only rang the dees ile, ote is reat Su172Acecilconsists af soueeoofw constant emf # and a resist tance Ml ana capacitor with capacitance € connected i series The {otoroalroisinee of the source i boeligble. Ata moxaeat f = 0 the apacttance of he capacitor i abroply dssrosed fold. Kod the current Nowing through the elcutt ata function of time ¢ ‘173. An ammotor and voltmeter are connected in series oa bat: tery with an emt BO. When a certain resistance is connected 18 in parallel with the voltmeter, the readings of the later decrease 1230 ties, whereas the readings of the anametor increase. the Thame number of times. Find the voltmeter readings after the coo Beetion of the resistance. "RATA. Find a potential diflrenco @ — gy betweon points and 2 of te eirult shown in Pig. 3.39 iC )—40 0, 200, 8 1D Ve and By = 20 V. The internal resist- ances of the estrent tees ate. negligible ae ‘S175. Two sourecs of cumoat of aval emi Connected in series ani have dierent |. Internal resistances and Ry” (fi > fp Find'the exter resitenos 2st which the gt Pateotildiference scons the feraioals of one Tithe sures Gehish one tn paricter?) be. Pit 8.29. Comin equal fo foo But76N source of current with differnt ems are connected asthowa In Pig 240, The ems of the soures ae propor 4 th Pig 840, Pie 2 their internal resistances, Le. 8 = alt, where «is aa assigned con- stant, The lead wire resistance is negligible. Find (the arent i He cal (0) the potential difesnce between points 4 and B dividin the elroule Inn and Nn links * S.177. In the citeuit shown in Fig. 3.41 the sources have emf's i) V and. 8, — 2.9 V and the resistances have Iho velues = 10.9 and Ry = 20. The interual resistances of the sources netligble. Fiad potential difereace qa pg between the Thies and 1 of the capacitor Cn “© 5.178. In the circuit shows in Fig. S42 the emt ofthe source is eaual to #—'5.0'V and the resistances are equal to Mf, 60 @ tnd 2 ~ 0.0 2. The Interoal resistance of the souree equals ft = "26 @. Fiod the esronts owing through the Tetons My 3.175. Fig, 3.43 illusteate a potontiometsie cireult by means of which we can vary a voltage V applied to a certain device possessing UT resitance A The potentiometer has a Tength T and. fsistance Fy. and soltage is appiod to its terminala. Find the voltage ¥ fed ortho device a8 a function of distnmes +. Analyse separately the case 3 By 3.180, Find the emf and the Internal cesistanes of source which {s couivalent to two balteries connected in parallel whose emt's fare equal to 2, and &, and internal resistances to. Randy. ‘S181. Fiod the magoitude and direction of the current flowing through the resistance ZC inthe cirult shown in Figs S44 MT the fy 4s oT te Te ipaereal a lt pat 2 Hig 546 Pies cemf's of the sources are equal to 8) = 1.5 V and 8 = 3.7 V and the raistancen ame etal to fo, 2p 20. n= 5.0°2, Tnteraal resistances of the sources, are, negligible 5. te cot shown in Fag 2.8 the sources B= 15 ¥,. 28 Vand the Sua to Ry 180, fy S04, By = 902. The intra ertanes of the sources are, negligible. Tod: (a) the ‘corrent Sowing. through the reistance = {bt potential diference 44 — ge Detseen the points A and B. a rae ‘3.183, Find the cungent Dowing through the resitance fia the elreult shown in Fig. 3.46. The ioteraal resistances of the i 36 batteries are opligibl ‘8%. Find potential difference 4 — py between the plates for C in tho circuit shown in Pig, 8:47 ifthe sources have =". V and §;= 10 V and the resistances are equal tof, =109, Ry = 200,'and Ht, = 30.0. The internal resistances of the sources aré negligible, 99 3.485, iad the current owing through the resistance fy of the ge gh i ie tthe tne ae eu 510 '0'Q) and y= 302, and the potentials of points, 2 and 3 fre equal tot, BV, and gy = 3 Mie sar, e848 3.186, A constant voltage Y= 25 V is melotained between points a and Bi of the Circa (Fig. $49). Find the magoitude and Hetek direction ofthe eurreot flowing through the segment CD if the rsit- ances are equal to) ='4.0G, 2.02 fy = 3.00, and = f'o. ‘3.187. Find the resistance between points A and # of the circuit shown in Fig. 380, ‘S188, Find how the voltage across the capacitor C varies with £4 2.190 What aust of hat wil be ened inci frei: ance # dus to. charge passing through i the curent in the coil a) deeeases domo ta foro sniformiy during 8 time iatoral 8 {6} decreases down to ero halving its valve every At second Skto0, Ade source with, iotrnal resbtance Hy ls loaded with thee Hiatal nistantes interconnected ns shown io Fig. 352. Al what value of twill the thermal power generated in this circuit ibe the highest? "3194, Make sure that the curent distribution over (vo resistances Ryand R, connected in parallel eoresponds to the misimurn thermal power generated. in this circuit. 3.192 A storage altery with emf £ — 2.6 V loaded with an ‘exiernal resistance produogs a crrant T= 1.0 A. Ta this ease the potential difereace Between ‘the terminals of the storage battery ‘quale Y= 2.0 V. Fiad the thermal power generated in the battery fd the power developed in it by slectee Taree. ‘3103, A'voltage Vs applied to ade electric motor. The armature winding resistance is equal to. At what value of cureat Bowing ‘rough the winding wil the ssafal power ofthe motor he the highest? Whattis it equal to? What. is tke motor eliciency in this case? ‘3.194, How much (in por cent) has lament diameter decreased due to evaporation if tho maintenance of the provious temperature eaquited an ioorease of the voltage by 1) = 1.0%? The amouat of Heat transfered from the hilsment into eurrounding space Is assumed to he proportional to the Slament sufaco aro, ‘S95. A conductor has a temperature independent resistance ft and a total heat capacity C. At the moment f =O it fs connected toa de voltage P Find the time dependence of conductor's tompe= ature Tastimiog the thermal power dasipated dato surround space to vary asf —k(D.~ Ty) whore & is'a constant, To 19 the favironmental temperature (equal to. the conductor's temperature at the initial monet) ‘3.196. A eimuit shown in Fig. 3.59 has resistanoes Ry = 208 and fiz 300. At what value ofthe resistance M, will the thermal o 4 6 al E all (le Ue Hig, 58 Pig. 354 or goneratd in tbe practically independent of sal vatitions Bra anotence! The veltage between ta points 4 and Bie supe posed (0 be soratant in thir ase Stor, tm aceielt shown in Fig. 9.56, resntances Ry and Ry are hnvvin_ as well as ints By and €y- The internal Tesstances SF euros are oolgile, At what value of the retance Zt Shit the’ theenal power generated init both highest? What i ft 18. series parallel combi number = 500 of identical ce ion battery consisting of « large ‘ach with en interual resistance 1~ 0.5 @, is loaded with an external cesistance = 10 2. Find the oumber n of parallel groups coasisting of an oqual number of falls connected in series, at which the exleraal resistance generates the highest ermal power ‘3.109. A. capacitor of capacitance © = 5.00 uP is connected to ' sowroe of constant emt # — 200 V (Fig. 353)- Then the soitch ‘Su wae throwa over ftom contact Z to contact. 2, Find the amovot ‘Of heat generated io a resistance 1, — 900 0 if Ry 390-0. S.200. Hetween the ‘plates of parallel plate eapacitor there i 1 metalic plate whose thickness tales up m)~ 0.60 of the capacitor ig 85. Pe. 330, wap_ When that plate is absent the capacitor has a capacity C ‘S%b0 ni ‘The capacitor is connected ton de, voltage soureeV = f00'V. The matalfe plate ts slowly extracted from the gap. Find: () the energy increment of the capacitor, the mucha! work ptomaed a the pines of plate exten “3201. A glass plate totally fills up the gap between the electrodes ‘ofa parallel-plate capacitor whoue capscitaaca in the absence of that glass plate equal to C= 20 aF The capacitor is connected to's de voltage source V = 100 V. The plate ie slovey, ad il ‘ction, extracted from the gap. Find the capacitor energy increment ‘andthe mechanical work performed ia the process of plate extrac tion, ‘3.02. A eylindvical capacitor connected to a de voltagesource ¥ touches the sneface of water with its end (ig. 3.58), The separation ete he capaci cron sfsaailly les fa th ‘enn radive Pi ight h to which the water Tevel ia the gap ‘wil rise, ‘The capillary effects are to be neglected ‘3.208. The radii of spherical capacitor electrodes are equal to ‘and 5, with a-< 2, The interelectrode space is filed with homoge ets satan of permittivity «and rentty, Tiny the Expacitor is not charged. At the momeat f= O te ierael electro wets a change gy. Find: (@) the thine variation of the charge on the interns lectrod () the amount of heat generated during the sprasdiag of the charge. 3.20, The lcs of capacity cary opposite charges p, = 10D me Sounectd through a fsistance 5.0. MG. Find {a} the change Rowing through that resistance during » time iater- vale S200 D} the amosat of heat generated fo the resistence during the same interval "Sah Tn a cect shown in Fig. 3.57 the capacitance of each capacitor equal fo Cand the resistance, to One of the cpaitors swim connected toa. voltage Ve and. then at the Toment (0 wes shorted by means of the switch 6 Soe Find fa) a current J in the clecuit as a function of fer ot “the amount of gemerated best provided « dence f(s kown 2 coi eral 25 mound ofa tia copper wire of length = 00m rotates with 40 2eite ueocty = 400 rads about its axis. The coll is conngct- Sitio ballistic gulvonometer by means of sliding contacts, The Seva! ssatace ofthe ivelt is equal to t= 21 2, Flad the specie Share’ of current caries copper If a sudden. stoppage of the eer gah a charge g'— 10m Bow through the galvano- S37. Find the totel momentum of eletrons in a straight wie ot ogi = {000 90 comeing a eurent 70 A. SHEA copper wire caries a curentof density j= 1.0 A/mm Ading’that"oue ree electron corresponds to each copper atom, ‘WSluntetahe distance which wil bw covered by an electron duriag fe dpagemeat f= 10m along the we 0 ‘5.20.4 straight copper wire of Long sand cron sectional aren S10 tia! caries ac A Assn Taian fiee lection corresponds to each copper atou, find uy he tie’ takes an eaeton to dxplace fom one snd of the write co the other hy the soma electric forces acting on all fee electons in the sien we S215.'% homogeneous proton boar accelerated by a poteatial dietnce 7 2G EV ae round towesetion of falas r = are eet Find the lets ald strength on the surface ofthe Bean fade potential iferense butwoon the surface and the axis of the Beant tthe beam urent ts equal tof — 50 A. Bite argo pase plese located im vacium, One of tua ares asa eatlodn, a source of electrons whose initial velocity seeligiie An eletoon Sow drceted toward the opposite plate rod veaee pace chong causing the Potente gap Stren the Histeseevary as fms where isn positive torstanty and = 1 the" ficinee vom the cathode. Find {0,8 volume deosity of the space charge as 9 function ie caret, dont Baie The air between (yo parallel plates separsted by a distance 420 wm fs ionized by X-ray radiation. Each plate has an ar ‘$= 800 em*. Find the concentration of positive ions if at a voltage 100 V's current T= 3.0 uA ows Detween the plates, which {swell below the saturation cuttont. The air fon mobilities are up S37 emm(ves) and uy = 191 cm!/(V). ° Fe fonized in the immediate vicinity of the surlnca of plane electrode 1 (Fig. 3.98) separated from elecirode 2 hy a dis tance [An alveroatiog vollage varvlag with time tas Y= Vy sia at Ievapplied. tothe electrodes. On Secreasiog the roqueney «it was observed thet the galvanos meter @ indicates a carent only at aon ‘fi | shor oy a ata cabo eqns. Fil |} the mobility of fons reaching electrode 2 under {|} {ins eonditons . ! e ‘3214, The air between two closely located Ly. plates is vaiformly ionized by ultraviolet radia- on, The air volume betwoea the plates is equal to ¥ ~ S00 em, the observed saturation current {sequal to Tyr = 0.48 HAL. Find: tl the mumbo on pals produced fo « wit volume per wnt (@) the equilibrium concentration of fon pairs if the recombination confacient for ait oo is equal to r= 107-10 cals "3.215. Having been operated long evovgh, the toner producing us 25-10 cn of fon pair per wat volume of alt par nit tine was switched of, Assuring Unt the oaly process tending To teeth nuates coe int hi erwin wth cot: eas, find how soom after the fooizer'sswitbing Of th oe coooratction deren = 2.0 ime * 3 Darallecplate it tapacitor whose plas are separated bya distance d= 3.0 mm is het charged (0a potenti) diflereace V'= 90 Vand then disconoected from» de voltage source, Find the time jterval during which the voltage atote the capactiar de creases by = 1.0%, taking into account that the average number of fon pars formed iar under standard conditions per unit volume unit Kime is egal to y= 5,0 cms" and thatthe given volt- 2 coeds tthe stration caren 217. The gap between two plane plates of « capacitor egual to ished with'egas, One of te plated emits vy eestrons per scons Which, moving in an electric hel, ionize gas molecule this way ‘ach elestron prodaces new elocrons (and fos) along a unit length ‘oft path. Find the eectroie eureat atthe opposite plate, nepeet- ing the loalzation of ges molecules by formed ins hig 38 B.218. The gas between the capacitor plates separated by a dit ‘anced is uniformly ionized by uteaviolt radiation so that my lac ons per uall volume per second ace formed. These electrons moving In thotlctrie held of the capacitor ionize gas molecales, exch electron Dproduclog st new electrons (and Tons) per uait leogth of is path, Replectin the fonization by Tons, nd the elestronic.curent dea sity at the plate possessing’ a higher poteotial. 35, CONSTANT MAGNETIC FIELD. Scenes Magne Sl «pst ches ¢ moving wih unit rei fein. om Dissolve anne ay, ennapt BL om 4 caclaton of yc Bad Ga’ ther ot Guamnde grasa. sy 7 . F = gk + olvBl. (asa) ae oy = (gpley, dP = 1 (dl, Bl (3.59) Yow ud manent ove ming on» magne ple y= 5: Pana, Nien es te ol te dai ox ctr 8 wth opt 9 ioe deen eae a aoa ax fue where 1 he ota mols euro var and seats ue Bad Geen asm ‘where 1 eth algehaie nu of nerpcope coments, % 1 fintont a tbe boondarsbetwesn. bro magnate: Boe Bams Hoxie sn For tho cape of maguatics in which J= a Be pad, pote as 3.219. A curceat = {000A circulates in a round thinewire Joop of fading = 100 1am, Pind the magnetic induction (3) at the oeatze of the loop: (0) at the point ying on the axis of the loop at a distance = = ~ 0 fos ene ' : ; ‘3.2000 A current T flows along & thin wire shaped as a regular polygon with msides which can be inseribed intaclrle of eadius R Fen the machete inaction at the cate ofthe ple. Atle the abtained expresioa at "> 0 Bat. Find the magnetic induetion at the centre of a rectangulae i equal to d= 16cm athe" he entrant. Sowing fa ‘the Teame, egy "3.222 ACcuryent 15.0 Afows along a thin wire shaped as shown in Fig: 3.59, The rdiasof curved part of the wire is equal to ft-— £20 mim, the angle 2¢'= 90" Find the magnetic induction of the eld at the point 0. 8.225, Fiod the magnetic induction of the field at the point of a loop with current 1, whose shape is ilustrated Sh iz eth all an Bae wal ot gl @ a8 (Gliia Fig. 3.602, the radius @ and the side b are know. Slat. A\Current f ows along a lengthy thin-walled the of radive ‘with longtaioal sie of width h. ind the indueion of the mag. Belle ed ios the tobe under the condition he 5.205. A curent J fows in ong straght ire with cose setion tring fora) of nthe hela of raion Pig. Bt. Pind atts i 3.226, Find the moguetic induction of the field at the point it a cnerent-carrying wire has the shape shown in Fig. 3.02 ay by ‘Tho radius of the curved port ol the wite is the linear par sssnmed to be very long 7 3.227. A very long wire carrying « current T= 5.0 A ts bent at right oagles. Find the magnetiesndetion ata potat lying ox a per Dendicular to the wite, dewwn through the point of bending, at f distanes 1 35 cm fom it ‘3.288. Pind the magnetic induction at tho point O if the wire car rying a current 1 8.0 A has the shape shown 10 Pig. 3.83 4, b,c 4 2 a [a] Tain| [ey , @ o Mig. 383. “a ein he crved prt ofthe wir i = 400 mm, th see pans he it ae ver avg “a, Fal te magna td diction ofthe meget indtion vesiorB i ‘ao ax insite plane carping cutest of near dnaty tua eto ti the se a ll plo oe plo ‘'af i pare fninite pans cary creat of nar dan sides ant 24he veces and tan unten al oto iiecerenponding"plaes, SN aalla! caret of density J fows inside an tofnite pinta ekg Spoil otra. fgd ea agte fd Ti induced by tesoeot asa fone Ui aeasce 2 at the” ean plane of the pate The magnet peril somamed BEN cunt out ohne and ote the pie, BB's dret_cuest 1 fows slong a Tenthy aig wie Prom," pat ae AME cots. esas ally at ¢ sre an intate eautusueg plane perpetions uy. “sag iatie he ete Hod he mageede daetin Me 88 BM unt of pen 5.2 "R Semen Tow along a round loop. Find the integral { Bealong tears ofthe lop within he rnp mn o> 9. Esplin the eat obisied °EBG, Mae cn of dnt flows slog rsd utr stnight wie wih crsecon adie Fd the agua oduct Set ot eurent a he pst fom puting eltve tthe a lie wire fs defined by aradius vectors. The taguetic permeability See eee se any tegen af te spate 3.254, Inside a long straight uniform wiry of round cross-section there is a long round eylindrcal cavity. whose axis ie parallel to the axis of the wire and displaced from the latter by a distance I ‘Attest curent of density} Hows slong the wit. Find tho magnetic induction Inside the evity. Consider, in particular, the eave 1 ‘3.289. Find the current denalty af @ fanetion of distanes F {fom the axis of atadially symmetrical pavallel stream of electrons ifthe Magoetic induction ideide the stfeam varies ax = br, where Diand a are positive constants, 3.286, Asinglelayer coll (olenoid) has length Z and cross-section radius ®. A sumer of taras per uait length is equal to.m. Find the ei induction at the centre of the cll when a corent 1 Nows Thott S87. A very long stesght solenoid has a crosesetion radius land ta prs ee, dt cart fv thug ie Solenoid, Suppose that Ta the distance fom the end of the soley Teasured loa i axis. Fi (Ee gnats butane the ais aston 0 deo ao dpprotimate plot of 8 vs the satio aft {oy the distance z to the point on the oxi at which the value of % fom’ that inthe middle ection of the sle- "3388. A thio conducting aeip of width b = 2.0 em is tightly sot Shape of very log ol with ene ttn radian = 72'5 cm to make 4 sloglelayer sight soleoid. A direct cureat T= 3.0°A' tows through the strip. Find. the magnetic Induetion foaide aid outside the solenoids funtion of the distance from ite wea “S389, N = 2.5-40" wie turns ace uniforaly wound on a wooden ua eo ery wal eos: eet flow trvgh the wire, Find the ratio of the magnetic induction taside the core or chat at the centre of tetova ‘8200, direct eureat I = 40 A ows in long seat round conductor. Find the magnetic fux through a Hell of wires cost Setion per one taste of is tot SEBUA ‘cery long straight solonoidcarros rose sectional area of the solenoid Is equal 10 furae per unit leogth is equal to m Fiod the Dox of the veetor B through the en ph ‘3.212."Fig. 3.05 shows ‘poid whose cros-section fs rectangula Find the magoetie ux through this roseaetion the curent through the Winding equals 7 f.7 A. the tal Fig, 3.65 umber of turns f= 4000, the rato ‘ot the outside diameter to the foside one is = 1.6, and the fight ts equal toh 5.0 om, current 1. The the ‘mimbor of 3.249, Find the magnetic moment of a thin round loop with eur rent ifthe radine of the loop 1a equal to ® = {00 mm and the mag: Delic induction at its centre te equal to B= 6.0 ul. ‘S2U Calculate the magnetic moment ofa thin wire’ with acu reat 0.84, wound tightly of half a tore (Fig 3.60) The diameter ‘Of the creavsoetion of the tore fs equal to d = 5.0 em, the aumber of turns 1s 37 — 500, i. 366 Pig 307 4.245, thin fontated wir fray plage pin of — 100 waa an nya cunaat 1 28 wh The sO of fat and ites Tr ig Bt equ toe = 50 ta a 9 — 10 mem Fin 1 the magn ntin a th cnr fi {3 ie neti owen the ape ih’ gee current Dae meee eauctng thin dine of adie cated uaoraly oven sei erie Unity tates bouts aia wth ‘stage lets ‘ai fueyecte Induction at the eats of th ds: 2) Sete moment of the. dae Saat caSecooducting sphere of ran = 50 mm charged unligtiy’ with auroee tently = 400 nCia® rotate with ale lly etoile ast the an pang theo fe Tae aguliinueton the conte of the spare is etre gs unfraly diced oe in wll a vif ball ofind'n snd ds seh rotates with ap neue Selden tho sas yusg tough entre Find the epee Teena mement ea rete to te mechan moment, aR loag lls elias of radian atte poled soa aa hs nts on polation te eal to = a here Sat eatne whet, snd re the itn tom the orb The Spline slat ation sb isnt withon agua nolo Fe anginal att cote oft elder Sos00 te polos ove pale teach thar with an equal velocity 0 St Kae Pind the aio of fore of aetic ad Tesi arco tthe protons “0 90 an ‘3.251, Find the magnitude and direction of «force vector acting fon a unit length of n thin wire, eazrying’« curreat = 8.0.4, at point 0, ifthe ite is bont as shown in a) Fig. $68, with curvature redius ft = 10 em (©) Fig. 3.68%) the distance between the long parallel segments of the. ire belog equal tof ~ 20. cm. 3252." Acol cntryine a curmeat J = 10'mA fs placed ip wolform sagostic fald so that its axis colockdes with tho field. direction. ‘The single-layer winding of the coll made of copper wire with Fg. 308. diameter d= 0.40 mm, radius of turns {¢ equal to 2 = 30 mm. ‘AU what value of the induction of the exteroal magnetic feld can ‘the “call winding be ruptured? 1.258, A copper wire with crosectionsl area S = 2.5 mm? beni to'make thre sides of a square can turn about « horizontal ‘xis 00" (i. 3.69), The wie fe located ia uaifora vertical magnetic feld, Fiod the magaetie induction if on passing a current T= 16 A. through the wire the latir defects by 4p angle 0 — 20% ‘3254. A small coil € with = 200 tums te mounted on ono ‘end of a balance beam and introduced betwoon the poles of an elect ‘agus! as shown i Fig, 3.70, ‘The croseseetional area of the coil Fig 3.70 fs 5 = 1.0 em’, the Jength of the arm 04 of the balance beam is 1230 om, When theo Is no eurent in the coil the Dalaace is in ‘quilibsium. On passing a cnmment 7 22 mA through the coil the fuilibrinim is retool by putting the additional counterweight of sass Aim = 60 mg on the balance pan. Find the magnetic induetion tthe spot whore the coil is located. ‘3.255. A square frame carrying & current J = 0.90 A is located the same plane as a long straight wire carrying e current fy = 5.0 A. the frame side hase length « = 8.0 em. The azis of the fame paising through the migpoints of opposite sides Is parallel to ‘the wire and is seperated from it by the distance which is = 1.5, mer greater than the side of the frame. Find. {a) Ampere fore acting on the frame; () the mechanical work to Le petformed in order to turn the frame through 180" about its axis, with the currents melnteined Constant. "3.256, Two long parallel wires of negligible resistance are con nected at one ond to's teelstance Mt and atthe other end toa de volt ge source. The dstanee between th Breater than the srase-sectional ff resistance Jt doce the teal ‘Sires turn Inio zero? 5.257. A direct current 7 ows in a long str ‘ernection has the form ofa thin hall-ring of radive H. The same ‘ureent Dow in the opposite direction along thin conductor located fon the “exis” of the frst conductor (point O in Fig. 2.61). Find the ‘magnetic Interaction force between the given con fvetors reduced to a unit of thle length 3.258, Two Tong thin parallel conductors of the shape shown in Fig. S71 carry direct currents ‘and Ty. The soparation between the conductors is a, I the wath of the right hand conductor is equal to BZ; | ‘With both conductors Tying’ In one plane, find the ‘magnetic fteraction force: Between" them reduced I to's unit of thelr Teogth ‘3259, A system consists of two parallel planes carrying currents” preducing’ a uniform magnetic is: 37 Feld of tnduetion B between the planes. Outside this epace thors ie no magnetic Bel. Find the magnetic force acting per walt area of each plane "3.200. A'condueting current-carrying plane is placed in an external ‘uniform magnatic Beld. Asa result, the maguetfe induction becomes os 4 a he @ o © Pig, 372, force of interaction between the a ATE equal to £1) on one side of the plane and to fy, on the other. Find lle magnetic force acting per unit area of the plane in the cases iustrated in Fig. 3.72, Determine the direction of the current ip the plane in each case B61. In an electromagnetic pump designed for transferring mol ton molalsa pipe eetlon with metals located ln-a uniform maguotic Held of induction 2 (Fig. 8.73). A current T'is made to How across this pipe section 4 {nthe direction perpendicular oth to the vector Brand to the exis of the pipe. 1s ee! peamure. produced hy ‘the pump He = O40, 1100 A, and a =2.0 em 4.262. Ncurment J ows ine long” thin walled éylindor of radiue H. Whet pressure fo, the walls of the cylinder experience? '3.268."What pressure” doce the Tateral scrfnee of a Tong. straight solenoid with turns per unit length experience when a curzent J ows through it? ‘deh. A current 7 Mowe ine long single layer solenoid With oes fectionsl radius f."The nomber of turns per unit length of the sle- noid equals n. Find the limiting current which the winding mey rupture ifthe teusle strength of the ite ie equal Uo Fei ‘3.265. A. patollel-plato capacitor with aree cf each plato equal to '§ and the seperation between them to is pue Into a stream uf con Acting liguld with resistivity p. The liquid moves parallel tothe Dlates with constant velocity v. ‘The whole system is Tocated in Uniform magnetic feld of induction B, vector B being parallel to the plates snd porpendicutar to the stream direction. The capacitor plates ore interconnected by means of an external resistance wi rd jn that Tsistance? At what jghest? Whet is this highest 1 Be carries a current 1-80 A. Find the potential diference between the exis of the conductor and ite sueface. The concentygtion of the fanduetion electrons in copper is equal to m= 0.0-40™ cm, ‘3.267. In Tall effect, measurements i a”sodlum conductor the strongth of a trazevers fel was found to be equal to B= 5.0 Vem ‘witha cnerent density J = 200 Alem! and maguetie induction B= “"h00°T. Find the conesotration of the cond\etion electrons and ite ratio to the total number of atome in the given eonditor ‘3268. Find the mobility uf the conduction electrons In 8 copper conductor if im Hall effect measurements performed In the magnetic field of induction 3 = 100 mT the transverse electre feld strength ff the given conductor turned out to bem) = S-lof0" times loss than {hat of the Tongitudinel electric Bela 3.269, A’ small currentearrying loop is located at a distance r from long straight conductor with current I. The magnetie moment 6 of the Loop is equal to Find the magnitude and direction ofthe foree vector applied (e {he loop ifthe vector Pe (a) i= parallel to the straight conductor, (6) is Srtented along the radius vector r {0} coincides tn direction with the magnetic feld produced by the cattent al the point where the loop is located S270. A small current-caryiog coil having ® magnetic moment jis cated at te ans of a sound Toop of radius Mwith current 7 (owing through it: Find the sagnitade of the vector force applied to thecal! its distance from the centre of the Toop is equal t= Sha the vector po coinees in divction withthe ani of the loop. "3.271. Find the interaction Torce of two coils with magnetic mo sents Pig 4.0 0A 08 ad Pam = 6.0 Anta and collinear axes if Uke seperation botween th colt qual to = 20cm which exceeds ty thelr linear dimensions z hae the shape of a sufficiently thin disc mopnetied slong ite asin The eedius of the dee is = 1.0. fBeniuoe the mayaitade oft molvcsler current Pfowing slong the Fim of the se ifthe matnetie induction ot the point on the exis of ie dice, Iying at a istance + = 40 em from Ite cone, is equal to ® 3.278. "The magnetic induction in vacuum at a plane surface of a uniform isotropie magnetic fr equal to B. the vector B forming an nea wih the riot of thescfae, The ormeability of the magnet fe lsequal to je Find the magnitude of the magnetic induetion Bin the magnetic in the vielnty of is surface ‘SA2TK The maguatie induction in vacuum at a plane surface of ‘a magnetic ts equal to B and the vector BTorms am angle O with the Pg nk, sais 2, whose eoutee lies of the surface of the magn {s) the Salalon of he vector Baron the sate pth F with side 1 located 8 shown In the figure ‘3.275, Adirect currant {lows in a Tong round uniform cylindrical wwiro mde of paramageatie with suseoptibility x. Find: (a) the surface molecular eursent Ty: () the volume molecular cusrent f; 2g age herent oa ech ter af ofan lnintety long mceght current-carrying solenoid ised with mognotie beance at thown in Pgs, Draw the approximate plots of magnetic indvetion J, strength H, and magne tet Jon the axis aa fanetions of = 3.277. An infinitely Tong wive with a current J fwing in it is located in the boundary plone between two non-conducting media with permeabilities wy and dy. Fund the modulus uf the mognetic riuction vector throughout the space as a unction of the distance fom the wire. {tshould be brae in mind thatthe ine of the vee. tor Bare eile whowe centres ie on the axis of thew ‘3.278 A round cartoncarrying loop Tes in the plane boundary between magnetic and vacuum. The permeability af the magnetic is equal tow: Find the magnetic induction B et an arbitrery potnt on the axis of the Toop it fn the absence of the magnetic the magnelie Induction at the same potat Becomes equal tp By. Genecalive the tained result to all points of tho felée 3.278. When a ball made of uniform magnetic is introduced into aifor magutic field with induetion By i gots uniform J mognetised. Find the magnetic indvetion B inside the bal with ‘Permeability recall thatthe magnet eld Snside a uniform mu etiod alls uniform and ite strength is equal to Hf = =I, Where J is the magnetization. 5.280, N= 300 turns of thin wis smanent magnet shaped as a cylinder whose length is equal to ‘Siem. When a current J-= 30 A woe passed through the witing ‘he eld outside the maznet disappeared” Find the coercive force Wy of the material from which the magnet was manufactured. B.281.'\"permanent mapuet is shaped as 9 ring with narrow between the'poles. The mean diemeter ofthe ring equaled = 20 em ‘Tuewiith ufthe gap is equal to = 2.0;mm and the magnetie induc sion in the gap iy equal to B.= 40 mT. Astuming thatthe scattering fof the magnetic fax et the gep edges is noeleible, nd the modulus ‘of the magnetic fld strength vector Snide th 3.282, ‘Anton core shaped an. ore supports winding with the (otal numl fare has a crosseut of width & = 1.00 mm. With 4 current 7 ‘0.85 A'fowing through the winding, the magnetic induction in gap sequal to B = 0115 T. Assuming thescattering ofthe magnet. tein at gap edges to be nein The peal ron under these conditions. fare uniformly wound on. per mssst us 3.288, Pig. 2.78 ilustrates a basic me commercial purity grade), Using ths plot, draw the permesbility 4, eg a aS nA Pig 8.8. equal tot 284, hthin iron ring with mean di 30 em supports winding consisting of 7 ~ 800 turns carrying curcent 7 = 3.0 A, Fone ing hase cross-cut of width b = 2.0 mam. Neglecting the scatter {ng af the magnetic fox et the gap edges, and using the plot shown intpig. 3.10, bnd the permeability of Iron under these conditions. ‘3.285. 'A iona thin cylindrical tof made of pazamagnetic with magnetic susceptibility yond having «cross-sectional area 5 is Ibeated slong the axle of a current-carrying col pe nt oe roca ath alegre wher the magnetic induction is equal to. whereas the thet end is located in the fegion where the map netic fields practically absent. What isthe force thatthe coll exerts on the. rod ‘2.286, In the arrangement shown in Fig. 3.77 it {s possible to measure (by means of a balance) the force with which paramagnetic ball of volume Vid} mm? is attrabted to 4 pole ofthe clocremag- Fig. 77 het MT. The magnetic induction ot the axis of the Doleshoe, depends om the eight z as B= By exp (—az4), where Be 1.30 fy a = 100 ms. Pind: a) ai what height zy the bal We fexperiences the maximum attrac: i the maxim- um attraction foree equals Fay ~ 100° UN. ‘3.287. A small boll of volusie V- made of paremegnetic with sus- coptibility x was slowly elsplaced along the suis ula current-carrying ol from the point where the maguetic inductien equels out to the region where the magnet fed ie practically absent. What amcunt tf work was performed during this process? 3.6, ELECTROMAGNETIC INDUCTION. MAXWELU'S EQUATIONS 4 Parday’s lw of elsctromugntis induction 4-2 een Tne eee shad nd dog ona. aay ‘whe 1 he our of tra, ith megnitle Gos trough ea am, "Sinden ef ice aes E> punt G4) + Tasosic ergy of current Tnteactin eomgy of 90 caret: Wate, yet 50 Volume deity of mae Sl ene: care eee + ipiecement core deny sa sn Mazo’ oqutins i Soret fom vine —2, vneo, ate whe © Xa ot (the rote) and Wm di the direc) wa geben sm fone me em enone ee Bt a Baieye a Ey Eaton a Bicivelet ad Vimar VER mcs the eymbol a 1 denote the Saldcomponets, epuctively pala nd perpendiclar to wo “a 8.288. A wire bont as a parabola y = az is located in » uniform imaguetic held of induction By the vector B being perpendicular to Therploue 2, y- At the moment t= Oa connector starts sliding trans Tetiguwrise team the parabola apex with coastant acceleration © (ig, 378), Pind thee of electromagnetic induction in the loop fue formed as & fanetion of y. 16 of >| 4 = ’ 7 Fir ams ig. 379. 4.289, A rectangular Toop with 2 sliding connector of length 1 1s losated ins uniform magnetie ld perpendicular tothe loop plane agnetle faguetion is equal to B. The connector has in Gjectee cesstence Pe the sides AB and CD have resistances Jt thd fe respectively. Neglecting the selF-inductance of the loop, Find the current flowing in the connector during its motion with & fonstant velocity v. "3.200. Aime dise of radius a 25 em rotetes with a constant angular velocity wo 130 rad/s about its ais. Find the potential Alference between the contre and the rim of the dise if (gh extra mapa el sas om? (3 the external uniform magnetic Held of induction fs dinected perpendicular to the dive S.201."A thin wire AC shaped as « somf-citee of diometer d ='20 cin rotates With « constant angular velocity @ ~ 100 rad/s im a unilorm magnetic. field of induction = 9.0m, with @ ft The rotation axis pastes through the end A of the wire and ‘2 Parpondleutar to the diameter AC. Find the value of line Itegral ar lng th itm pls 10 pnt. Geri the ob thined rest, ‘3202"°R wire loop enclosing « semicircle of radius os located on the boundary of a uniform magnetic field of induction (ig 380). At the moment {= the loop set into rotation with (egastant angular sceseration Babeut an axis O eninciding with 2 fine of vector B on the boundary. ind the anf induced tn the Loop ba fonction a ine Bra to aprorneta pet of ti anton ‘The arrow inthe Agure shows the em direction taken to be positive 3.200. log stalght wire carting « current F and « Teabaped candvetor wiih aiding connector are Toeeted in the same plane #8 18 ‘20h A square fae with side @ and along sraight wie carrying aptis sce he e plneashown o Pg he vr tana oth rt with consent weg. Find the induced in the frame as'e fonction of distance =. ‘iisent oko re 182 reat 4.285. A metal rod of mae m can rotate about horizontal sxe Oo. dig slong ceciotsondustor of tadlan o Pig 263). The Strangumeat isto in a wmitor magnetic held of indveton {rete perpendicular tothe ring planet exe and the tng ate Somected tan emf soute to fom a enralt of tesstanee Regie Sng the ftom eet indotene, nd ing ressanc, fa the oe ‘oring to'nbich the sate of onust vary to make the od aa Sith conmont angular velocity “oleh capper connector of ttt m slides down Seo smoot co er ars sent snangleto thy horizontal, Suet revity (is, rceied through revstance Te ‘Suel fo The evem tr lead in sm meguetc fin indeton 8, perpendicular tthe plane dos To rneancen the bare, ; ft and the shiding contact essed to Beil. Find the steady-state velocity of the stom dilfers from the one examined in the foregoing problem (Fig. 3.84) by a capacitor of capacitance C replacing the fesistance A: Find the eecdleration of the connector 6 A\« Co We. 8384 Mig 38 8.298, A wire shaped as 9 somi-citele of radius 4 rotates shout axis OO" with an angular velocity o in s uniform magustie field of Induction B (Fig. 8). The rotation axis ie perpendicular to the ‘old divoetion. The total rsistance of the eireut is equal to. Re Nep Ieeting the magnetic field of the induced current, find the meen amount of thermal power being generated in the loop during # tation “period: ‘8.29, K'smail coll is introduced between the poles ofan electro ‘magnet 59 that is axis coincides with the magnetic field Gteetion, ‘The cromesectional area of the coll is equal to S = 30 mm, the ‘aumbor of turns Is "= 60, When tho coil tars through {80° about iametor, a ballistic galvanometer connected to the coil indicates 4 change q ~ 4.5 uC fowing through It. Find tho magnetic indvetion ‘magaitude between the poles provided. the total resistance of the blecini ciecuit equels Zt’ = 40" 0. 3.300,°A square wiro frame with side @ and str conductor carrying e constant current / sre loeated in the same plane (Pig 3-88), 4 af EB 4 . a a Me. 358, Pi, 387 ‘The inductance and che resistance of the frame are equal to Land respectively, ‘The frame was turned through 180" about the axis OO" eerie ome ‘carenccaesig.condcta bya ditance ind the electric charge having fown through the frame. ‘301, A long x n end 9 from which a 10 ight wiro carries current Ty. At distances @ el 10, the former ene, ineeted by & feslstance (Fig. 881). A connector ides without friction along the wires with a constant velocity.» Assuming the resistances of the wires, the connector, the sliding feomtacts, andthe salf-Inductance of the freme to be negligible, nds ) the ‘magnitude and the diction of the current tnducod in ‘the. conneeto () the Torce required to maintain the connector's velocity com 3.302. A condveting rod (AB of mass m slides without tition ovat two long conducting rails separated by a distance 1 [Fig 3-88). [Av'the left nd the rails are interconnected by w resistance R. The ‘ystems ja located in a uniform magmatic eld: perpendicaler to the plane of the loop. At the moment t= 0 the rod 42 starts moving to {heright with am initil veloety vy. Neglecting the resistances of the alle and the rod AB, st well asthe self-ndustancs, Gnd {@) the distance covered by the rod until it comes to standstill, (0) the amount of heat genorated in the resistance R during this proces, 4 @8 Ler Fie 3.85, Pig 389. 3.308. A connector AB con slide without friction along a M shaped conductor locate fo horiontal plane (Fig 3.88). "Te eam beet Sasa longth I, mast my and renstaten Pt The whol ly. Oe constant horizontal force F starts acting on ‘he connector sifting ft translasionwice to the right, Find how the velocity of the eonnectar varies with tme f. Tho injuetance of the Toop an'the resistance of the T-shaped conductor are aasnmed be negligible 3.904, Pig. 8.90 ostrates plane Bgures made of thin conductors whieh are loceted in a anifor magnetic Reld dicectol away from SOO © @ reader beyond the plane of the drasting. The maguatic induction Starts diminishing. Find how the currents induced in these Toops a 8.805, A plane loop shown in Fig. 3.01 is shaped a two squares ule oo cond 10 en nd Ind nt 6 o- Form magnetic field at right angles to the Loop's plane. ‘The magnetic {duction veries with time as — By sin ot, where By = 10 mT and 400 5°". Find the amplitude of the current indueed inthe Toop if its resis- tance per unit 50 maim. The i be_noglected ibe A plane spiral with « bor of tants wound tetly to ope another iSjowated n'a suite magnetic eld pr pendicular fo the spiro plane. The outide Fetiue of the spirals turo# is equal 10 & ie. 804 eB = By sin of, where fo of emf ingwced In the spiral, ‘B87. A U-shaped conductor is located in a uniform magnet field perpendicular to the plane of the conductor and varying wi time at the rete B = 0.10 Tis, A conducting connector starts moy- ing with an acceleretion w = £0 cmis" along the parallel bers of the Conductor, The length of the connector Is equal to f= 20 em. Find the emf induced in the loop #— 2.08 after the boginning ofthe fnotiom, sf'at the moment f= Othe loop area and the magnet Induetion are equal to zero, The inductance of the Toop is 10 be oglecte Ft. in a long straight eolenold with crose-sectional radius @ ud number of turns pervunit length ma current veries with w con stant velocity 7 A/s, Find the magnitude of the eddy current ld rent es « function of the distance r ftom the solenoid axis, Draw ihe appre lta ds fein ‘Z800.°A long straight solenoid of cross-sectional diameter d = Be an ith nn 20 fare pe nen of eth has oud turn of eopper wire of erosvacctional area S — 1.0 mm? tightly put fon Its winding, Find the curgent Mowing ia the turn ifthe current in the solenoid winding is ineeased with e constant velocity 7 = {C0 Als "The inductance of the turn isto be neglected ‘S10. & long solenokd uf erosesetionol ras a has @ thin insu ted wire ring tightly put on its winding: one half of the ring has the revitanee times that of the other half The magnetic indvction reduced by the solenoid varies with me as = bt, where 6 is TTeonstant, Find the magoitade of the eletee eld strength in the M311. A thin non-conducting rg of mass m carrying «charge @ can frovly rotate about ils axis. At the initial moment the ting wet Strost end uo mognetic Held wes present. Then a practically uniform ‘apace eld was switched on, which was perpend cular tothe plane ‘of the ring and increased with time according toa certain law B (!). Find the angular veloeity @ of the rig a= a function of the induction ©, S212, A thin wite rng of radius a and resistance r is located in side-« long solenoid so that their axes coincide. ‘The length of the Solenoid fe equal to bite eromeseetional radius, to'b. Ata certain moment the solenoid wae connected to a source ofa constant voltage Fe Tho total resistance of the circuit loquel to A Assuming the inductance of the ting t be negligible, find the maximum valve of the radial force acting per unit length of the ting. ‘pal8. A moguctic fs thr stationary Toep with a resistance AR varies during the time ierval tas @ ~ at (e— 1}, Find the “mount of heat generated inthe loop during thet time. The inductance the loop i to be neplected. ‘3.314. In the riddle of & Tong solenoid there is » coaxial ring of square cros-sction, made of comdting raterial with resistivity Prithe thickaese of the ring i equal to hy its Inside and outside redit She equal toa end respectively. Find’ the eurrent induced in the fing tthe suagnetic induction produced by the solenotd varies with {inne os 8 — pty where B isn constant. ‘The inductance Uf the ring {sto Be nopocted. M5. How many metres of thin wie are required to ms ture a tolonoid of length Ty = 100 em end Inductance Z.— {tthe soleauid's cross sectional diameter is considerably les than Its length? Bst6, Find the inductence of » solenoid of length 1 whose wining is tad of copper wie of maze m. The winding resistance [Scan fo"R The cnltoid diameters coasierbly le then He lent siz. A coll of inductance = 900 mH and recstance R= {io ma is connected to a constant voltage source. How soon il the coll eurrontreech 1=20% of the steady-state value? "3.8. Caleulate the time constant of straight solenoi of length 1 210m having a single-layer winding uf copper wire whose total ings eal tom = 10 kg. The cromstetional diameter of the Shlenoid i assumed to be considerably Tese then its length, ‘Nater The time constant vis the ratio Zi, where L ts inductance and. ie gettve resistance. B19, Find the inductance of & unit length of eable consisting of two thin-welledcongiel metalic eylinders i the redius ofthe out Side cylinder is y= 2.6 times, that of the inside one. The perme- ibility of a meditm between the cylinders Is assumed to he equal to unity "3.20, Calculate the inductance of e doughnut solenctd whose nal redive i oqual to 0 anv crom-section has the form of @ square ‘with side a, The solenoid winding consists of N tums ‘The space ia Iie the solenoid filed up with uniform paremagnetic having per mesbility i 3.321. Calculate the inductance of Tine (F nit length of a doable tape 3.92) if the tapes are eoparated by'a distance h which is bly les than theirwidth 6, nome, B/h = 30. ‘5.322. Find the inductance of unt length of @ double Tine i the fading of each wire isn times lest ‘than the distence between the axee tfithe wires. The Sela. inside the Wires ig to be nepleeted, the pore TL; ‘meablity is assumed to be equal to unity throughout, and 97> 1 8.825. A" superconducting round ring of radius a and. inductance Z ‘was locatad in a taiform magnetic eld of indoction 2. The ring plane ‘was parallel to the vector By and the carceot in the ring was equal to oro. Thea the ring was turned theough i" s0 that Its plane Became perpendicul Fie. 392 (b) the work porformed during the tum. 2. A current fy = 1.9 A flows Ia. @ long closed solenoid ‘The wite it is wound’ of is in.» sipercomucting sto. Find th turrent Dowing Va the solenold when the length of the solenoid 1S ‘50 mm made of thin wire of radios ‘magnetic fold with induction B~O.50 m0 20th perpendicular to tho vector B. ‘Then the ting was cooled dowa to a superconducting state, and th smaignetic fed wos switched off Find the ring eurrent after that. Note ‘that the inductance of a thin ring along which the surface cursent flows is oqal t= jt (Io ‘3.326. A closed circuit consists of » source of constant emt & an ‘choke col of inductance Z connected In series, The active resistance Sf tho whole cireut ts equal to fl. AU the moment £—0 the choke oll inductance was decreased abruptly 9 times. Find the eurrent in the cireuit ae 8 fametion of time # Tastruction. Durlag stepwise change of Inductance the total magnetic fox (Sux linkage) remains constant, ‘S827. Find the time dependence of the current Qowing through the inductance Z of the elrouit shown in Fig. 3.08 aftr the switeh ‘So isshorted at the moment ¢ — 0. 3.528. Tn tho eireait shown in Fig: 3.94 an emf 8, a rosistanco 2, tnd coil inductances Ly and Ly are Known, The fntarnal resistance of the source and the coll resistances are negligible. Find the steady- Stave currents in the coils after the swith Ste wes shorted "3.820. Caleulate the miutual inductance ofa long straight wice and 4 tectangular frame with sides @ and 6, Tho frame and tho wire lie ‘amo plano, with the sido b being closest tothe wire, separated Dy a distance Irom it snd oriented parallel to it a fy Fig, 80 Fig, 396, 4.90, Determine the mutual inductance of « doughout innit straight wire gassing aon fla eroseectiom te onde aan one, {0 8. The iogth of th doushnut's cross tctional site porliel tothe wi equal toh Teal ha tre The system feces Uniform maguetie with permeability pe S51, hi sontntrc wire shaped ap cls with edit @ and le in the sume plane. Allowing for'a <0, finds ie hl maa iauaney the magmete du through tho surface enelomed by the outside ‘ite hen the fside wie cartes cutent feos ‘330R.A small eslinrical magnet Mf (Wg 3.95) ia placed in the cone ofa thin cll of radias a costing of turns. Th calli one ‘aca to's ballistic galvanometar. Th active renetanee ofthe whole Citelt is equal to. A~ Find the maguetie moment of the magast St Se femoral rom thou rns is shane owing through ht salvanometer, 3.383. Find the approximate formula expressing the mutual Ia- stueince of tvo thin goatal loops ofthe some Tedius f thei oo fren aveseparted by w distance with 13-06 7 te 3.884. There are two stationary loops with mutval inductance 1p, Th current in oe of the loop stats to Be varie a8 Fy a ‘where a is's constant, £ 1s Lime. Find the time dependence fy (hal ‘he current inthe othe loop whose inductance Is Le tnd resistance it. ‘8.885. A col of Inductance L == 2.0 pH and telstance R= 1.0°9 ls connected to a source of constant emf = 3.0'V (Fig. 3.96). A 18 resistance Rg = 2.0 2 is connected in parallel withthe coi. Find th bout of heat generated in the oll after the switch Swis Giconnect= a.'The internal resistance of the sources negligible 3.836. An ion tore supports N= 00 (ums. find the mognetie field energy if a curent 7 = 2.0 produces @ magnetic Hux actos the tore's crossection equal to @ = 1.0 mWb, 507. An ion core shaped as_a doughnut with round crocs-sec tion of radnis a Sem earries a winding af N= 1000 turns through hich a current 7= 1.0°A flows. The mean radius of the dovghiut Whe ho cm, Using the plot fn Fig. 8.76, ind the mognetie enerry red up tn'the curt. A eld strongth iis supposed ta be the some {Throughout the eross-soction and equal toils agutude in the cene ite of the cross-section ‘3508. A thin ring made of a magnetic has mean, diemeter a= 00 em and supports a winding of N= 860 turns, Th fectional aren of the ing is equal to. = 5,0 cm*. The Sfoseeu of width P= 2.0 mm. When the winding cartes « uvrent the permeability ofthe magnetic equals = 1400. Neplct- fe the dissipation cf magnetic Sux at the eep eget, find fa) tho ratio of magnetic energies in the gep and in the magnetic; (3) the inductance of the system; do iin (wo ways: using the Mut ‘and ‘tusing the energy of the feld "3.00.4 Tong liter of cava carrying a uniform surface charge rolates about its axis with sn angular velocity o. Find the mog- fotte eld energy per unit length ofthe eylinder ite Tinea charge ensty equals hand. = 1 “S310. At what magnitude of the electric eld strength in vacuum the volte energy density of thie fed i the same es that of the meg- De Fad ndtion BLOT Gay eco ‘S341 A thin uniformly charged ring of radius « = 10cm rotates spout fis oxis with an angulerwelolty @ = 100 rede. od the n Goof yolume energy densities of magnetic and clevtle fields on the {his of the ring at's point removed from its centre by s distance re ‘3.342, Using the oxpresion for volume density of magnetic ener- gy, demonstrate that the amount of work contributed to mognett Sion o's unit volume of pare: or diomegneti, is equal tod = in sr intereonnocted (a) in sovies Gb) In parallel’ Assuming the rafual tnduetenee of tho colle to be negligible, find the inductance ofthe system in both case. enoide of equal length and almost equal ross. fully inserted into one another, Find their mutual Induetance Hf their inductances are equal t0 Ly and Ly “3845. Demonstrate that the magnetic energy of Tateraction of ‘wo eurrent-esrrying los located in vacuum ean be represented Whe tl) § 4B e¥ her Ba Bre the magnet incon within » volume element gV, produced individually by the currents Sf the fest and the second loop respectively 8.346, Find the interaction enoegy of two loops carrying currents Ay and I, doth loops are shaped as citsles of radio and by with a'at. ‘The loops! centres are located at the same point and { Planes form an angie 0 between. them, ‘3.547. The space betwoen twa coteentric metallic spheres is filed up with a uniform poorly conducting. medium of resistivity p and permittivity At the moment #0 the inside sphere obteine 2 forts. charge: Find : the ieplacmant eurat den sity andconduction current density at an aritrery potat of the me ‘ium at the same moment of times ie {vthe giver moment of time the charge of that sphere Is opal to 3.MS."Aparallel-pate capacitor i formed by two discs with 9 wif poctly conducting medi between them. The eapesitor Yas initially charged and then disconnected trom » voltage soute Nepcting the wage oles, show that theses nn Tmagestic bald Intwoen capacitor platen. 049, ATparlleepate lr condenter whose each plate bas area 5 = 10h emis connestod in seis (oan ae eicats Bod the tise Geld strength amplitude i the capacitor ifthe nua rent amplitude ia lead wits is equal to Tq 1.0 mA oo the arr frequency equals © = 1.6-10"¢~% 51350. The space between the electrodes of » parall-plate capa- cltorie fled with a soiform poorly condveting medivm o conde Sty and pormitivity «The ceptor plates shaped as eound ses tro separeed by 1 distance d. Neglecting the edge elects, End the Id steongth betwoon the plates at's distance r from tele kien ae voltage V A ong : turns per unit length, Ax atternatingcareent rough e ind the dspisce ment current density a 0 stone rfum the slenotd {tin Tho crosesectfoal radius of the solenoid quale 3.852. A" point. charae q moves with a non-relativistic velocity vy = const, ind the displacement current densiy Jy at poi heat: a's distance FTrom the charge on » staight Ite (a) colueiding withthe charge path: (@) perpendicular to the path and Passing through the chase. ‘S.3sh.'R thin wite ring of radive carrying « charge 7 approaches a satin poin tht cet moves realy ich ‘constant volecty 2. ‘The plane ofthe ring Tmaine perpendcul tothe motion dinetion. At what distance 2m from the plat P will tho ring be loested st th moment when the caplacoment canes Alensty at the pont P bocomee maxims? What it the magnitode of this moximam density? 3.854. A point charge @ moves with » non vy = const. Applying the theorem for the eirel round the dotted circle shown in Fig. 8.7, nd I at the point 464 function of a radius vector r and velocity v of the charge, 3.959. Using Maxwell's equations, show that {aa imodependent magnetic held cannot exit without en elec wie eld 2 unit dependent {) inside-an empty eavity a uniform electric (or magnetic) Geld cam be timodopendent 850, Demonstrate thet the law of electric charge conservation, Le,"W.j = aya, fellows from Maxwell's equations ‘3.857, Demonstrate thet Maxwell's equations © XE — — Blot and ¥-B = 0 are compatible, Le. the frst one does not contradiet the second one, ‘3.458, Ins certain region of the inertial reference frame there is magnetic feld with induction 2 rotating with enguler velocity «1 YE in this region as 2 function of vectors @ and B. 5.359. In the inertial eforonce frame K ther ss niform magoetic field with induction B, Find the clecteie fed strength in the frame A’ which moves relative lo the frame K with a non-relativistic ve locity v, with ¥-1B. To solve this problem, consider the foros acting ‘on an imaginary charge in both reference framos at the moment when the volocity of the eharge sn the frame A” is equal to 280 " Fie 808, plate of non-ferromagnetie material moves with a constant velocity i 90 cms n'a uniform magnetic Held with io vetion Bo 80 mT as shown jn Fig. 3.08. Find the sutface density of eleetre charges appearing on the plate a8 a reewlt of ite motion 3.61. Along solte aluminum efllnder of radius a= 5.0 em rotates about its aris tn a uniform magnetic fed with induction B= 10 mi. The angular velocity of rotation equals o ~ 45 redi, with @ #1. Neglecting the magnetic feld of appearing charges, nd helt space nd surface densities .962.'A’ non-relativistic point cherge g moves with s constant velocity v. Using the feld transformation forziules, find the magnet- c Induction B produced by thie charge at the point whose position ralative to the charge is determined by the ranius vector 8.388, Using Eqs. (3.68), demonstrate that if inthe inertial ro erence frame X there is only electric or only magnetic feld, is any ‘ther inertia frame K botheleciic and maguetieBslds will coexist Simuléaneously, with BL ‘2.864. In an Inertial Teloronce frome, KC there ie only mognetic field with induction B= (yl ai) Gt-+ gh, where B is 3 cone stant and jae the unit vectors of the + andy axes. Find the lee: infe eld steongih E Yn the frame A’ moving felative toe frame Ke vith ac ok: Eis the wait vector of the taxis, The taxis is amwmed to coincide with the £ Sis, What is the shape of th Beld E? ‘3.365. In an Hnertialrlerence frame A there is only eletre etd of strength B= (at yt 8), where a isa constont, Land {are the unit vectors ofthe z and y axes. Find the magnetic induction B'in the frame A” moving relative tothe frame K with # constant non-tlotivistic velocity v= tk; kis the unit vector of the ax ‘The "axis is sumed to coincide withthe taxis. What isthe shape of tie magnette Induction B. . 3.306, Demonstrate. thatthe transformation formulas (36h) follow from the formulas (3.69) at eye ‘3.367, in on inertial reference fame. K there {3 only a uniform electri Bold E = BuVim in strength. id the modulvt and direc: tion “@) of the vector B (b) of tho vector Bin the inertia reference frame A” moving with 9 constant velocity ¥ relative to the frame ‘at an angle = 45" to the vector B. ‘The veloelty of the frame K" {sequal tot f ~ 0.60 fraction of the velocity of light 35568. Solve a problem difering rom the foregoing ene by # mag- netie eld with induction J 0.8 T replacing the electric eld ‘3.068. Electromagnetic field hs two invariant quantities. Using the transformation formulas (9.6). domonsteate that these quaatitie (o) Ew; (b) 2 — o0 0. ae nal tren ra hr ae to oor ily perpendiculer felds: on sleceicBeld of surengih B= 40 KV in tnd a magnetic Geld induction 0.20 mT. Find the electric Strength # (or the magnetic induction B") fn the roferenos faa "shore only one Belly electie or maghetc, is observed ‘Instruction, Afake tse of the feld invariant ete in the foregoing problem S371. A point charge ¢ moves uniformly and rectilioesrly with iativintie velocity equal to 2 B fraction of the velocity of light (G'<'vie. Find the electric field strength B prodeed by the charge the point whose radius veetor relative (othe charg ts equal £0 F'and forms an angle 9 with is velocity vector 19 17. MOTION OF CHARGED PARTICLES IN ELECTHIC AND MAGNETIC FIELDS Lanes fre + gv am Motion equtin of «mle partic om axe) ry tharos mas of the nV ‘e"ieutrn cnn, that is the comition for electon to move a a cnhie oii in natin = nada an thre Bethe magic ton tam obi’ pont, (By Isto monn value UPN Isactin Sede th ot 3.972, AL the moment ¢ ~0 an electron leaves one plate of a par alierplate. capoeitor with & negligible velocity. An accelerating voltage, varying a= Vat, where e = 100 Vie is applied between the plaies, The separation hutwoon the plates i f-—"6.0 cm. What {s the velocity of the eleetton et the moment it roaches the opposite plate? ‘3.573, A proton accelerated by a potential difference ¥ gets into ‘the tnlonm slectri¢ eld of parellel-plate capacitor whose plates extend over a length fin the motion divetion. The eld strength varies with time 98 constant. Assuming the pr ton to be none Angle bewooa the motion ree tions of the proton before ite Bight through the capacitor; ti pion gen ihe eat he moment #0. The ede eet are tobe neglected. ‘3.374. A particle with specie charge gin moves rectilinearly due to-an electric field E = Fy— az, where a Is'a positive constant, © Fs the distance from the point where the particle was intially at rests Find “@) the distence covered by the particle tll the moment it eame tovn standstill (by the seosleration ofthe particle at that moment, B75. An electron starts moving ia «uniform electric eld of strength B= 10 LVlems How soon after the start will the kinetic fnorgy of the electron become equal to Its rest enersy? ‘2376. Determine the acceleration of relativistic electron moving slong a uniform elects Held of strength & at the moment when its kinetic energy becomes equal to 7: 3.377. At the moment f— O'4relativistic proton ies with a ve- locity v ato the region whore there iss uniform transverse electric fold of strength wlth vy 1” E. Find the time dependence of (4) the angle 8 between tho proton's velocity vector and the inl= tial direction ofits mation (@) dhe projection vy of the vector v on the initial direction of motion ‘3.378. A proton accolorated by 2 potential diflerence = 500 XV ‘ioe throeh a uniform transverse magnetic eld with induction B= 0.51, The feld eceaping a region of space d = 10cm in thickues (Fig. 3.99). Find the angle a through which the pro- {on evan rom the nia eto of et motion. iat r= 100mm int {psec tng hell with indaton 70.0 mE Find ie velocity sd pos ‘Hod of reveluton it that porte Pig. 50, S.380."R relativistic particle with charge g and rest mess my cle of radive 7 ia'e uniform maguotic Sold of ton {@) the modulus of the particle's momentum voet (H) Ghe kinetic energy of tho particle; (6) the gece the particle V3. Up to what values of Kinetic energy dose the period of revolusion of an elecizon and a proton in a uniform magnet Bld fxoned that at non-rlativietic veloc Tout Ileal trajectory of the electron, ‘3.385, A slightly divergent beam of non-telaivistiecherged per Ailes acnlnstee by 8 potential deren propagates rom» poi A slong the axis of = sireght solenoid. The beat fe Brought fate focus at a distance {from the point A at two successive values of sto induction By and By Find the epee charge gin of the articles, ‘3.384. A non-relativistic electron originates at a point A lying on the axis of @ lraightsclonold and moves with velolty 0 a¢ an Sgle 2 to the axis. The magnetic indvction of the eli is equal to ‘BeFind th distance r trom the axis to the point onthe srern into ‘which the electon srikos. The sored is oriented at right angles to The axle ond is lente at distance {fom the point ‘3.385. From the surface of a round wito of radius @ carrying a direct current fan electron escapes with a velocity ty perpendicular tot sure, Find what wil be the maim dina fe ele toon foom the axe 8.806. A non-rlaiviste charged particle es through tho ele- trie eld of aeylindical eapscitr aod gete nto 9 aform transverse tagnetic feld with induction 2 (Fig. 3400). In tho capacitor the Particle moves along the are of «ciel, in the magaetie held, along BMemicircl of rads. Tho potential difrenee applied to the cape Sitor i equal to Vs the radi of the cletrodea are equal to @ apd by SG g''b. Pind tho weloctty ofthe particle sed spect charge a yy ea aL rg 2400, hig 101. 3.387. Uniform electric and magnetic fields with strength 2 and induction B respectively are directed along the y axis (Fig. 340%) ‘Aparticlo with peel charg gin loaves the origin O in thodiection Ot the ¢ axis with an intial nom-elativistic volocty vy. Pind: () the coordinate yy of the particle when it erases the y axis for the nth time {@) the angle a Botweon the particle's velocity vector and the y exis) at that moment, "RSME, 'A'narrow bea of identical ions with specific charge gim, possessing different velocities, enters the rion of space, where there fre uniform parallel eloetre and magnetic fields with strength Sind laduetion Bat the point 0 (soe Fig. 3.101)."Tho beam direction oineides with the x-axie atthe point O- A plas cereen orlanted st ight angles (o the x ati is Toeated at distance {from the point 0. Find the equation of the trace thet the iors leave on the screen Demonstrate that at -<°1 i isthe equation of 2 parabola. "200.,A non rclativstic protan beam pase without deviation ‘through the region of space where there are uniform trensverce tually perpenicularelectnie and magnetic elds with B= 120 V7 ‘and = S0'mT- Thon the Deam suikes a grounded target. Find the force with which the beam sels oa the target if tho boem current equal to T= 0.80 mA. 3.300, Non-relativietie protons move rectilinearly in the region of apace where there are uniform mutaslly perpendicular electric and magnetic feldswith £40 kV/m and 30 mT. The trajectory of ‘he protons lie in the plane z= (Vig. 8.102) "and forms an angle {g =\30" withthe 2 axis. Find the pitch of the helical trajectory along ‘which the protons will move after the electric fold is switehed of swe M.A beam of non-relativistic charged particles moves without deviation through the rion of space A (Fig. 2.103) where there are ‘ansverse mutually perpendicular eleetre tnd magnetic elds with 7 s LE, 7 i ig. 3802 Fig 2408, strength B and induction B. When the magaetic fd ie switched of, the trace of the beam on the screen 8 shifts by Az. Knowing the Alstancee « and b, ind the specific charge im of the particle, ‘3.302. A partic with specie charge gim moves in the rogion of space where there are uniform mutuelly perpendicular electsie and raguetic elds with strength E.and induc. ion B (Pig. 3-108). AU the moment t= 0 he particle wat located atthe point O and han "zero velocity. For the now-elativiste se, find @ Jew of motion x (2) and y (2) of the pertiles the shape af the trajectory, (phe length the seamen of te trict ry Dotween toe Yeloety of the par (@) tho mean "value of the particle's veloc. ity "vector projection on thes axis (Ube drift velocity). "4.308. systom constts of « long eslindrieal anode of radius @ and a coaxial eylindrical eithode of radius 6 6 um position ite velocity equals ry and b, respectively “is. A point pesforms harmon deellations slang © cr with a period 7 idan amplitude « — 10.0 cm. mean velocity of the point avoraged ovar the timo interval during ‘hich Te travels a distance 0/2, stating trom He) the extreme position; (h) the equilib position 4G. At the moment #— 0's point starts oselleting along the 2 axis According to the law # = sin of Find (a) the menn valve of its velocity vector projection (o,; (0) the modulus of the mean velocity vector |v {e) the mean value of the velocity modulus (o) averaged over 318 of the poriod aftr tho star, 47. Apaeticle mover slong the z axis according to the law x ~ = cor wh Find the distance that the particle covers during the time interval from f'~ 0 to & “48. At the moment £0 particle starts moving along the = axis so that Ks velocity projection varios as vy ~ 39 eos at cis, ‘where tis expres in seconds, Find the distance that this partite ‘overs during = 2.60 afr the start 48, A prticle performs harmonic dscilations along the = axis according to the law 2 = @ eas of Assuming the probability P of the paruicle to fall within an interval from =a to ato b Unity, Bnd how the probability density Pld depends on 2. Here UP denotes the probability ofthe particle falling within an interval from z to x 4 dz. Plot dPlds as-a function of = “410. Using graphical mesns, find an amplitude o of oscillations resulting from the superpocttion of the following eeeilstions of the fame. direction (a) 2 ~ 5.0 c0s (at + 2/8), 29 = 8.0in (wt + () 3 = 80 oe wt, 25 = 5.0 oa (ath), 2 4.110°A point participates simultancously in two harionie osil- lations af the stme direction: 2 ~ a cos af and 25 =a cos at, Find the maximum velocity of the point. “12, The superposition of two harmonic osellations of the same ‘ieeetion results inthe oseillation of a point according tothe Taw 4 s-g cos Date cos 30.0, wboro t is oxpressod {a seconde. Find the fngular froquencice of the constituent oeellations and the’ period ‘with which they heat 4.18, A point A oscillates according to a certain harmonic law in the reference frame A" which {nits turn performs harmoule scl tone ralative tothe referee frame K. Both osellaions oeeur along the seme direction. When the A” frame oscillates at the frequency DO or 24 He, the beat frequency of the polat A in the K frame torn out to bo equal to v. At what frequency of acellation of the frame ‘K" will the beat frequeuey of the point A Decome equal to 2v? 4014, A point moves in the plane zy secording to the law 2 = asin at y = bos of, wre 2, , and w are positive constants, Find: (@) the trajectory equation y (2) of the point and the direction of tts motion along this trajectory: (0) the ecesloation w ofthe poin relative to the origin of coordinates. “dS. Find the trajectory equation (2) of « point sf it moves ac- cording to the following laws! ain 20h, ot Dat. function ofits radius vector ‘4.6. A particle of mass mie located ina unidimensionsl potential fl whore the poteatil energy of the particle depends on the coor Ainato 2 e¢ U@) = Uy eos aa) Uy nde atm anatante Mind {the period of small oscillations that the’ particle performs sbout the eauilibrlum position “17, Salve the foregoing problem if the potential energy has the form @) = els* = bis, where a and b are positive constants. "248. Find he period of small oselations ta vertical plane per- formed by a ball Simase mem 40'p xed atthe middle of & haeion- tally stretchod string f= 1.0 m in lengths The (ene {Stazumed to be coma Determine the per pendulum, that ina ball suspended by «th Wivis located in «liquid hose density in = ot the ball. Th tsistance of the gui isto be neglcted. ‘20 em in Tenet, Mig Ae ig 42. the thon withthe Dall was deviated tough amall angle (> @ tnd‘ ls. Auuming the ealnon of the bal agit the ral o> Epnelly sooer Se tho ouciation period of fee endalus 1608 4.21. A pondulum clock is mounted in an elevator ear whlch starts going up witha constant acceleration ts with w= Ata height {horacelernton ofthe eat reverses, te magnitoderemeiolng constant {ow non after the start ofthe motion wil the lock show the right ime sania? “Coa Calculate the period of small oscillations of « hydrometer (hig. 12) which was slightly pushed down in the vertial direction ‘The mase of tho hydrometer fom = 90g, th radia of ie abe le 17°" 33mm, the density of the liguié lap —1.00 glea?. The ree: {ance of the lig amd toe negligible. ‘E33. Asondoformed spring whos els are xed has sstfiness x 2465 Nem A smal body st tase = 25 i alvached atthe fomoved from one of the ends by 9 — 4/9 ofthe springs Yngth, Neg. Teotng tho mace of the spring, Aad the period of smell longitsdina ‘seilatons af the body. Pe fee of grovity = assed tobe aunt. 4.25, Determine the period of small longitudinal oseilations of «body with mase m in'the system shown tn Fig. 4.3, The stiffness Walues of the springs ate x aid. Tho Tretion and the masses of the springs are negligible “4.25. Find the period of small vertical osellations of a body with sass m in the system ilustested in Fig. 44. The stfizese values of the aprings ae yaad, hel uses to neglge “Aremall body of mote m ts xed to tho middie ofa stretched string of length 20 In the equilibrium positon the string tension it foqual to Ty. Find the angular frequency of small ogeilations of the Indy inthe transverse dieetion. The mast ofthe string is negligil the gravitational Bold Js absent. l o i. Pies 44. Hie. 46. 42, Detormine the period of oscillations of mercury of mass m= 500 p poured into a bent tube Fig. 4.) whose right art forms fo angle 8 "MG withthe real, Th coaawtionl are of the tube is S'— 0.50 em The viscosity af mercury Is to be neglected. 60 4.28. A uniform rod is placod on two spinning wheels Fig. 4, The azes ofthe wheels are separated by a distance the cootclent of trctian between the od snd tho wheels isk = 0:18. Demonstrate that in this ease the rod performe harmonic oscil tons. Find the period of these oscillation. Qo 4 shoft going all the way through the Earth from pole to pola long i rotation axis Assuming the Earth to be ho. ‘geneous ball and neplecting the a drag nd: 12) the equation of motion of a body falling down i {0} how Tong does it take the body to reach the othe the shatt; end of the shat {pte velo of he body af the Barts cnt 4130, Find the period of small oscillations of a mathematical pen- dulum’of length if Ste point of suspension O moves rulative to the Barth's surface in en arbitrary direction with a constant aezoleration w (Pig. 47), Calenlate that period if = 21 cm, wm 12, and the le between the vectors w and g equals f= 120" 434, In the arrangement shown in Pig. 48 the sleeve MF of sm 0120 ig is xed botyeon two, identical springs whose combined ‘Sifipes is equal to x ~ 30 N/m. Tho losve can side without frle- tion over horizontal bar AJB. The exrangoment rotates with a cor ant angular velocity = 4.4 radia about a vertical axis passing throu the middle of the ba. Find the period af small osilstions fof tho sleeve. At what valuge of w will there bo no esellations of the Seve? “{92._A plank with « bar placed on it performs horizontsl harmonic cccillaiions with amplitude ¢ = 10 en. Find the coffielent of fre tow Between the barand the plank if the former starts sliding along 10 the plank whom th ampite of oxi — | —hErti“<—~Sw™t~stst~*~tCS—W inion pnt SP tae na me fants {devine thogh he gle 3.0 ad he tf without o a ~—E—rti of the Poynting vector outside the’ beam at « distance r 7208. A current Bowing i the winding of « long straight solenoid ‘is jnereusod at a suffctently slow rate. Demonsteate that the rate at wiieh the onergy of the magnotie field in the solenold increases 4 equal the Bas ofthe Poyting vector arr he ftra surface Adin tig £39 ilsirates « sogment of « double line cating diet cara wo detonate th ows. THRE nto account that the potential q, >, and making use of the Poynting vector, establish om which side lft or right) the source of the eacrent is Ioeated L —L~— +, 4.208, Tho energy i transferred from a source of constant voltage ¥ to a consumer ‘by means of a long straight coaxial cable with hogligible active resistance. The consumed current. J. Pind’ the nergy flux across tho erot-eection of the eablo. The conductive Sheath is supposed to be thin. “4.208. A source of ac voltage Vi = Vy cos of delivers energy to ‘consumer by means of «long steaight epaxial cable with wogligible fctive resistanen. Tho current in the circuit. varias 7 = wyatt ~ 9). Find the time-averaged enorgy Mux through the rostsection of the cable. The shesth it thin “£210. Domonsteato thai at the Doubdary between two media the normal components of the Poynting vector aro continuous, 1.0. Sie a Sm “L211. "Demonstrate that a closod system of chargod non-tulati- vise particles with Identical specie charges emite no dipole ra- iation ‘4212, Pind the mean radiation power of an eloctron performing srmonie oscillations with amplitude a — O-10.nm end frequen ey ‘a0 at 4.249, Find the radiation power developed by 2 non-relativistic, ace wih ergy « and ts m, moving slong erular orbit St radius R ia the fold of 2 slatlonary point charge g ‘4214 A particle with charge ¢ and mass m Glee with non-rolati= visti velocity» ata distance past stationary particle with charge g, Neglecting the beading of the trajectory of the moving icles nd the enorg Tost by this particle due to radiation during fhe total Might time, “4.215. A nonteltivistc proton enters a halfspace along the normal to the transverse ‘niform magnetic feld whow induction equals = £.0°T. Find the ratio of the energy lost by the proto {ue to radiation during its motion in the feld to its initial Kintie “CBI6, A non-olativste charged pasticle moves ins tranavere wpiform magnet Hold with induction Find ho time dependence of tho particle's kinetic energy diminishing due to raliation, How soon will its kinetic energy. decrease e-fold? Calculate this time intoreal Tor the cae (a) of an slotron, (8) of «proton ‘C17, A charged purticlo moves slong the v axis scording to the lawy — con of, an th point of obnervtion P is located tho = ‘hist a distaneo? fom the particle ("> a) Pind the ratio of electro. ‘aagooticradietion fow densities Sy, atthe poiat Pat the momenta Sin the egordinate of tho particle jj ~ Uland y, = a Caeolate at oni S310 an 1 218. A charged particle moves uniformly with velocity v along a cite adi ih to pla xy Pi. ll. An observer toca Nyy wa fn the 2 axis at a point P which 1s removed from the eontr of the circle by a distance much exeeeding A. Find (2) the relationship between the observed values of the y projec: ‘tion ofthe particle's seeeleration and the y coordinate ofthe particles () the Fatio of electromagnetic. radiation Now densities Sy'Sy ate polatP at the moments of time when the particle moves, ont the standpoint of the observer P, towand him aud away from hit, fs shown’ in the Szure. C219. An eloeromapnetic wave emitted hy an elementary dipole propagates in vacuum so thet in the fer held zove the mean value Of the energy flow density Is equal toy at the point removed from the dipote by x distanen r slong the perpendicular drawn 0 the Aigole's axis. Find the moan radiation power of the dipole. ‘200. The mean pawor radiated by an elementary dipole fs equal to By, Find the mean spnce density of energy of the electromagnetic Beld‘in vacuum in the fer eld zane at the polnt emoved fom t dipole by # distance r along the perpendicular drawa to the dipoles a, 4.224. An electric dipole whose modulus is constant and whose ‘moment is equal to p rotates with constant angular velocity © ut the axis drawn t right angles to the axis of the dipole and Passing through its midpoint. Find the power radiated by such 2ipote on 223. A plane electromagnetic wave with frequency « falls upon fn elastically bonded electron whose natural frequency emus on Neglecting the damping of oseillations, find. the ratio of te meeh ‘nergy dissipated by the electron per unit time to the mean value of the eneriy ow density of the inedent ‘wave, 4.224, Assuming a partici to hae the fort of a ball and to abe sorb all incident light, fad he redius of a particle for which ita ‘ravilational attraction t0 the Sun is counterbalanced by the force hat Tight exerts on it. The power of light radiated hy the Sun equals P= AMO" Wy and the dencity of the particle is'p = 1.0 shea PART FIVE oprics St, PHOTOMETRY AND GEOMETRICAL. OPTICS Spc respon fan ye Fin abowe i Pigs 5A. ‘+ Lominovs intensity / and omiaane ‘+ Mominance produced by « point Keotopie sure: tema on ents, 1% cris he ange toe the rsa to he ace and he distin to the = Lumiosity M ed minanee ¢: — ete Fors Lambert sores ~ const and luminosity Maat aa) 10 a ta at 1 enn ha det oan sh aratuy die of agate op ta 2 oto st etn oun are th ae ot the opt power of thik Tas hry dite thicken of the lan, This equation isan wali for «apt of ee" os ‘y's diem wilh mete olet n Fig 5. 4 Lagrange Helmbolts Sevan au = cant (tm 4 Mogitring powse of an opal dvi eam) where aed are the anes ubtnded at tb oe by ao age fod by the ‘pat aven Sth the orayonng sett dias fa fonant eke Bey the eos roe oe mining ws ht ian go I, Making uso of the specteal responte curve for an eye (se Fig. 34). Bn " " (a) the energy fx coresponding to the luminous Mux of 1.0 Im a the "wavolongthe 0.51 and 0.64 ym (@)the luminous fix coresponding to the wavelength interval from 0.58 (0 0.69 um ifthe respective energy Mx eq to y= “45 mW, is uniformly distributed over al wavelengths of the inurl Th function VQ) is aud toby Hear i tho ven Spectr interval 5.2. A_ point otropie source emits « luminous Mux @® = 10 im with wavelength =" 030 ym. Find the pen steength values of cleetrie and magnetic folds in the lominous fax at aistance F = iO m trom the souree. Male aoe of the curve illustrated in Fig. 5. 5.3. Find the mean iluminance of th iradiated part of n opaque sphere roesiving , ms (@) parallel luminous fux resulting in illuminance By at the point ‘of normal incidence '@) light from a'point fatropiesouree located at a distance ! = 400/m rm theente af he ape; theca ofthe sphere = 6)"em and the Tuminous intensity is f= 36 ed 5.4. Determine the luminosity of a susface whose Iuminance dopands on direction as L'= Ly cas , where 0 is the angle between the radiation direction and the'normal to the surface. 5.5. A certain luminous surface obeys Lamberts law. Tts lumi nance ig equal to. E. Find: {a) the Iuminons fax emitted by an element AS of this surface into’ cone whose axis is normal to the given element and whose spertize angle fs equal to 8: (0) the luminosity of sue a source 8. An illursinant shaped as a plane horizontal dise § ~ 100 e in area is wisponded over the contre of @ round table of radius f= = 1'0'm. Ts lumipanee does not depend on direction and is equal to E1610" eit AU what height over the table should the iluminent be suspended to provide maximum luminance at the fleeuumference of the table? How great will that luminance be? ‘The illaminant is assimed to. be a point sore. 7. A polat soureo Ls suspended at a holght = 1.0 m over the cenieeof around table of radius A= 1.0m. The luminous totensity 1 ff the souree depends om direction so vt iminanee at all poiats Df the Lae fs the same. Find the function J (0), whoee 0 ts the angle btovoen th radiation diseetion and the vericaly as well asthe Iu ‘minous Muy reaching the table if 1 (0) = T,-~ 00 ed, ‘BA vertical shaft of light from s projector forms a light spot 57100 em? in aren on the ceiling of a round room of radius = = 2.0 m. The illuminance of the spot Is equal to BE — 1000 Ix ‘The reflection coefficient of the ceiling ie equal to p — 0.80. Find ‘the maximum iluminance of the wall produced by the light reflected rom the csling. "The reflection” is" aseurned to obey Lambert's Tw 5.9. A luminous dome shaped as a hemisphere rests on a horlton- tal plane. Tis luminosty ip ualform. Determine the illuminance at the font ofthat pane is laminanes equate Zane i independent 10. A Lambert souree has the tom of ap infinite plane, Its luminance ie equal to L- Find tho illuminance of an area element ‘otiented. parallel to the given source. uff, An illominant shapes! as plane horizontal disc of radius Ao Bs cm is suspended over stable at s height h = 75 cm. The ‘luminance of the table below the contr of the illuminant te equal to y= 70 ix. Avsuming the spate to olay Camber law, Bad ite liminosty 5.12. A small lamp having the form ofa wniformty luminous sphere of radius R= 6.0 cm i suspended at a height h ~ 3.0 m abowe the Moor. The luminance of the lamp is equal wo L = 2.0-10" odin? and is independent of direction” Pind he ilominance of the Door ‘ireety below the lamp, ‘5.18. Write the law of eelection of light beam from «mirror in vector form. using the directing unit veclors © and eof eee tn thine The angie af ican 0 6 FLA the value of foams to come from am optically ative refractive index of these media pricm with refracting angle 6" provides the ‘lia i, Bink th ont deecton ange 3Y composed of two monochromatic components passes ‘through trihedral_prism ‘with refracting angle 0 — BO" Find the angle av between the enmponentsa the ray after its pas age hroah ‘he pr i their rnpctve indict of tact ae ‘qual to f-5t9 and 1.520. The prism is oriented to provide the least ‘lection angle ‘5.25, Using Fermat's principle derive the laws of deflection and refraction of ight on the. plane interfece between two media. 5.28. By means of plotting find: (G) the path of a light re) after roflection from a concave and convex spherical mirrors (soe Fig. 9h, where F is the focal point, 00" the optieal axis); Pig 54 () the positions of the mirror and its focal point in the exsns AMlostrated’ in Fig. 5,5, where P and P” are the conjugate points Ba ” a be vo. @ o Fi. 55. $ 0 5.27. Determine the focal length of a concave miror i (a) with the distance between an object and its image being equal to 1-15 cm, the trapsvorse magnitestion B= —20: (b) ina cartain position of the abject the transverse magnifistion fs B, = ~0.50 and in another position displaced with respect to the former by a distance f= 5.0 em the transverse magaifstion py — oo 28. A point source with luminous intensity fy — 100 of ie positioned 4 distance 20.0 em from the ereaf of a concave {ror with focal length = 25.0 em. Find the luminous intensity of the reflected ray ‘the tllotion ‘coefficient of the miror is = 0.80, 45.29. Proceeding from Fermat's principle rive the refraction formula for paraxial rays on a spherical boundary surface of = Aus oteeen media with refractive tn ‘oes m and ‘5.80. A parailel boar of light falls fom vecuum ona surface enclosing.» mediam ‘with tufracive Index n (Fig. 9-8). Find. the shape of that surface, (7), Ifthe beam is brought into focus at the point F ata distance / from’ the crest O. What isthe maximum radine of a beam that aa SUIT Be focused? 206 15.31. A point souree ig located at a distance of 20 em from the {rout surface ofa symmetrical glass biconvex lens. ‘The lens is 5.0 cm thick ‘and the curvature radius of Its surfaces is 5.0 em. How far bbeyand the rear surace of this les is the Image of the soutes formed? ‘3.82. An object is placed in front of conver surface of a glass plano-conves lens of thickness d= 0.0 em. Tho image ofthat object EMformed on the plane surface of the lens serving as-a sreen- Find (a) the teansverse magnibeation If the curvature ralias of tho lone's convex surface is Mt = 2.5. cm: () the Image illuminance if the Tominanes of the object i Z "700 elim and the entrance aperture diometer of the leas is D = 5.0mm; losses of light are negligible. 5.38. Find the optical power and the focal longths (a) of a thin glass Tons'in liquid with refractive index my — 1.7 it optical power in sir ¢ Oy ~ —5.0 D> (G) of « thin symmotrial biconver glass lens, with air on one side snd water on the other se, if the optical power of that Tena io ale is @y = 10 D. Sf, By means of plotting find (5) te pao ay ht beyond thin covering od dieing lenses (Pig. 5.7, wheee 00" isthe optical axis, Pand Fare the front and rear focal points): fe fe Pig 8. (0) the position of thin lens and its focal points Hf the position of the oplieal_suie 00" and the positions af the cojugate points , P* (we Fig. 5.8) are krown; the media on both sides ofthe Tenses sare identical: 6) the path of ray 2 beyond the converging and diverging lenses we SEA pat Pe Bean oie ta ie Pig. 88, ro all known; tho media on both sides of the lenses tel exis 00" ate identical, ‘5.35. A thin converging los with focal length f = 25 em projects image of an abject on a sewn removed from the lens By" dl 20s 0 (3. Then thescreen wasdruwn close to the lens by a dis 48 em, By what distance should tho objet be shifted {or its image to become sharp again? 5.38, Aree of ght i cited at ditane 1 = 60 em fom Seven. A’thin converging lens provides the sharp image of the Soares when placed bnt wen the sores of light and the sereen atta Position: Dtcming the fora length of the les i a) the distance betwen the two position af the lens is Al — ~'3) eam: (6) the transverse dimensions ofthe image atone positon of the logs aren = 40 greater than those at the other poston 5.37. A thin converging lens is placed. betwen’ tn object, and 4 Screen whose poaitions ae fixed. There ge too, positions af the {ons at which the sharp image af the object is formed onthe screen Fi the transverse dimonsion of the object i tone position of the lens the image dimension equals #2 mm nat the ther, Pe mam 3.88, A"thin convorging lens with aperture ratio D sf = 4 3.8 (is te lens diameter, [i ls focal lengli provides the image al & sulictntly’ distant cbject ‘on a photograph plate. The object Tuminance is = 260 ed/m®, The losses of light In the Tens amount tova'= O40, Find the illuminance of the tage 3239, How does tho luminance of teal image depend on diac eter D of thin converging lens f that image is obeerved (a) diceciy, (0) on white screen backscattering according to Lambert's law? 5.40, There ate two thin symmetrial lenstes one is converging. with refractive index m= 470, and the other 's divenglng with Feiriclivo indox mg 1 Both lens have the sane eurvatute Fadiue of thele surfaces equal to R= 10-can"‘The lonees wows pot lose together and submerged into water. What is the Toca lengths oF this stem in water At. Deterinine the focal length of « concave sphorical mirror which is manufactured In the form of & thin symmetsc biconver bliss ons one of whose turlaos fe slvered, Th curvature tadloe Stethe lens surace fe R= 40 cn 5.42, Figure 39 ilastrates an allgned system eons tin Fests The system is Doeated aes Deter eg of tne 5000 rhe s00m stip ton s00 Pip 5 (2) the postion of the point of convergence of a parallel ry incoming from the loftafer passing throesh the system {@) the distance betwoen the frst ens anda point Iying-on the axis to the lel of the syatem, nt which that point and {cba ao Inested ‘symmetrically with tespct to the lens system. 5.3. A Galilean telescope of 10fold magaiseation has the Tenth of iS em hen aajusied fo ndnity. Detenmine (a) the focal tengthe of the telescope's ohyoctive and ocular (5) by what distance tho ocr shouTa be displaced to seus the telteage, tothe stance of 50m ‘Sd. Find the magoifcation of » Keplerian telescope adjusted to Infinity Ifthe mounting of the objective hes a dione D and the image nf that outing formed by the teleceape’s cular bas 8 di meter i. On passing through a telescope » Bux of light ineweoses ie intensity 2.010 mes, Pind the sogular dimeton of tant abject its image formed by that telescope has an angalar dimen sion 9" 20" Su; Reniran thane wit mauiain T= 5 wo smonged into water which led up the inside ofthe telescope To ake the sssiem work as tlesnpe aztin within the former dieters tho objective woe replaced Wt basthemageitesson ofthe teeccop Become’ equal to? The refsstve inden of the glee of which the ala is" made Is equal to n= 1 sar. At what mapniention T-of telescope with x diameter of tho objective D8. em fe the luminance of the image of an thet mth retina aot ese than witout the telsenpe The pol ameter is asumed tobe equal tog 0 se, The lowe of Tht ine cape” are nee a 5:8, The optesl powers the obtetive ant acuae of iw re egual to 160 and 2 D eeprtivey The mieoscope ea nisl to 80. What wil ie magaifintion ofthe aerose Ihehen the distance bntween te cicetiee andthe ocular Snes bs 20cm 5.9, A microscope has » numerical pectic ino 0,12, whee iste aperture angle subtended by the entrance pupil of in mire scope, anuming the diameter af eye's pupil to be equal 0 do °F nm, detrnine the microscope magriicalion st which (3) the diamotor of tho beam of iuht coming from the micrssupe fsleqeal to the diameter af the eves pup (3 the hhiminanee af the tioge on the tine is independent of magnification (consider the case wien the beam of light poring tough the system “microscopeeye” i hounded by the mounting ofthe objet) 3.50. Find the positions of the principal planes, the focal and ol points of «thin icone symnetse lass fens with corsatne Tadias"ol Me rorfaces equal to t= 1.0 me Ther itr On one sl ot the lens ao water on the other 5.51, By means of plotting find the positions of focal points and principal planes of aligned optical sysioms illustrated in' ig. 5:10: (a) telephoto lens, thet is combination of 2 converging and 4 diverging thin lenses y= 1.5 a, f, = —1.9 a); ance ia 4 @ Fig 50, otal nett Vm, OOD, By meens of plowing fad.” ee {3 the Postion of the paint & ‘onjuzato w the poi 5 a As ee ta fa) ' (6) eS (©) the positions F, F", and AU (Pig. 54, where the path of the ry of light Is shown before and aler passing through the system). 5.58, Cappo F and Fare the front and ost focal points of a0 optical systam, and Hf and If are its frost and rear prinefpal points ‘By means of plotting nd the position ofthe image Sof the point § for the following relative positions of the points ‘Sy Fy 2 nd (a) PSITH'R": (by HSF'FH (0) HSE’ PH; (Q) PH SHE, Subd. A telephoto lens consists of two thin lenses, the front conver ing Inns andthe rear diverging lous with optical powers 40D and ®, ~—10 D. Find (@) the focal Tedyth and the: positions of principal axes of that system if the lenses ate teparsted by a distance d= 4.0 em; {d) the distance d between the lens at which the ratio of 8 foc length fof the system toa distance [between the converging lens and thereat pineal focal pont isthe highest, What i thi ratio equal 45.55. Calealate the positions of the principal planes and foes! solnts of thick conver-concaveglas lens ithe curvature rai Bi th convex sarfece i equal to y= 10.0 em sn the eancave Sulacé to Ry = 80 cin and the lcs thickness Ie d= 2.0 cm 38, An aligns optical system consists of two thin Temes with toca lngtha fan te dtance between the lenses being equal {0°a. The given system has to be replaced by one thin lens which, A aay position of an objet, would provide th erme rn tagnicsion atthe system. What must th focal length of tha ENGaT avai at poston mas be acl th nexpe {othe tora aytomt? ‘S89.\A sytem consists of a thin symmetrical converging glass Jenn wi ue earvatre audi f Hs rutacn A= 38cm and & plane Tniroc stented at righ angles to the optieal axis af the Tene’ Ths {isones between the ent and the miter is t= 2 cm, What is {he optical power af this astem when the space Between the lens the itor i Blled up with water? 8 At what theknes! will 8 thick convex-coneave gla lene mal (2) serve a a eescope provided the curvature radi of ts convex sulface i aft" te) can pester than that of ls comeave surface “t) have the opitea power equal to 0 D if Uo curvature ral of ts convex and eameave sacs are equal to 1.0 and. em reopetivelyt 350. Find the positions of the principal plans, the focal length the en of pia pool of «ti mvs conse es {s)whove thickness equal tod and curvature radi ofthe surfaces ant tie sme and equal ta. Ts) howe refacivesuraces are concentric and ture eal Randy la Rp) S.A tshcoe hte cots of wo lay ball with a a= 30cm 1 cas What ae the distance botween the ive oft alls dad thy nngniiction of he syste the ager aH teves an a0 sbjectye? ‘6, Two identical thick symmetries! Bionvex leases are pat clas togetter The thicknes ofeach lens equals the. curvtary fadine of he surate, te d'= Ht -c30 cm Find the optical power this ayo tn ai. Si6B Xray of Hight propagating Sn an Gotropic mediom with refractive tndex'n varying fradeall (rm point to point has aear- ‘ature radius p deermiaod bythe formula $= grien. where the derivative is taken with repct to the principal normal othe uy Dero this fermtas assure that in such 8d they retain usin 0 Sone Wali: Hare @ the angie be {een the rayand'th tection ofthe vcior Wa ats giver Polat oy 5.63. Find the curvature radius of a ray of light propageting an’ horizontal divection lose to the Eah’s surface where the {Tallent of the refractive index in ait equal to. approximately SM10-" mt. At what value of that gradient Would the tay of light Propagate all the way round the Path? 5, ‘TERFERENCE OF LIGHT Width of tinge 2 syhery. tie the distance tom the sources to the ee, ie he distance betwuen our 1 Tempo an sata cores, Cabaret length and cohen rae: ee oy ea an, 4, 2) whee ith angsar dinwasion of the sore. ‘Condition for icerecace asin in the case of ight rfid from thin flats of cen RV AHH, = oe EI, 20) eg ie ocd on rein of Uf th wan of 2 sie ieraer lore hl i ad la late with wich {he eonvexmurceofth lave iin conc Tha mug te ig VIR, 620) it the gg Hing bright Ce = 4 9, 5. and dank fo 2 6,6 ‘The sahus #2 eiaepoues to thle’ ot soe dark 5.04, Demonsteate that whea two harmonic oillations are added, the time-averaged energy of the recultant ostllation Is equal to the sum of the enezpies of the eonststzent rsellaions, if oth of them (@} have the same dizeetion and ate incoherent, and all the valucs of tho phase dillecence between tho oscillations are equally probable: (0) fre. mutually perpendicular, have the sum Treguency and an arbitrary phase dillrence ‘By means of plotting find the amplitude of the oscillation resulting from the addition of the following three osillations of the fame diction, Hrs aeoswt, Ey = 2esin ot, Fy = 1.5a.cos (ot + 21). 5.68. A curtain oscillation rnalts from the addition af eokerent agen of he sme dietion Ga weft >t) ‘rhere fis the number of the aseillation (k= the phace ditfrence hetween the Ath and. (k— Find’ the amplitude of the resultant ofcillation, 110 5.67. A system lustrated in Fig. 5.42 consists of to eoherent point sourees and 2 located in @eortaia plane so that their dipole moments are oriented at right angles to that plane. The spurees nee Separated by a distance dy the radiation wavelength [Siequal to 2. Teking toto account that the gece ions of source 2 lag in phase behind the oseillations of souree 1 by 9p (7 <3), find: (@) the angloe Oat which the radiation tatensity (i) the conditions under which the radiation inten sity in tho ditection @ = 1s maximim and in the ‘opposite diretion, malnimam. ‘3.63. A stationary radiating system consists of @ Linear chain of parallel ooellators separated by 9 is. tune dy With the oscillation. phase varying Vneariy, along the hain. Find the time dependence of the phase difference Ap between the neighbouring oscillators at which the principal radiation sax mum of the system will be “scanning” the satroundings with the Constant ‘angulat velocity” w. 5.69, In Lloyd's miror exporiment (Pig. 5.13) light wave emitted directly by the source S (narrow sit) interfres withthe wave reflect fot from a mirror Mf Asa result, aa Interference fringe patterns Be Pg. 82 ss % ig. 3.18 formed on the sereen Se. The source and the mirror are separated by distance 1 = 100 em. At a certain position of the soutee the Iringe ‘width on thesereen wasequal to Az— 0.23 mm, and after the souree was moved away trom the mispor plone by Sk ~ 0.60 mniy the fringe width decreased = 1.5 tImes: Find the wavelength of light ‘5.10. Two coherent plane light waves propagating with # diver- agence angle <1 fall lost normally on a Seren. The amplitodes Sf the waven are equal. Demonstrate that the distance between the heighbouting maxima on tho sereon is equal to Az = iy. where fe the wavelength, 5.71. Figure 514 ilostrates the interference experiment, with Fresnel mirrors. ‘Tho angle between the mirrors is 2 = 12", the Aistaneas from the miler” interection line to the narcow sit S| tnd the sere Se are equal tor = 10-0 em and b= 130 cm respec: tively. The wavelength of light is k 0.55 ym. Pind: ‘width of fringe on the sereen and the nomber of possible Shift of the interference pattern on the soreen shen the ced by 61 = 1.0 mm along the are of radins 7 with pola 0; (c) at what maximum width by of te slit the interference fringes on the senven" ave still observed sufficiently sharp. Fig 56 5:72, A plans light wave falls on Fresnel miteors with am angle 20" between them. Determine the wavelength of light ifthe ‘width of the fringe. the sereen x = 0.59 sam ‘37% A lens of diameter 5.0 em and foeal length f = 25.0 em sas enk along the dismeter ito two identies! halves, In the process, the layer of the Te ‘00 mm in thickoes: was Tost. "hem the hhaives were put together to form a composite ens. In this focal plane a narrow slit whs placed, emitting monochtomatic Light with ‘wavelength h 0.00 mt. Behind the Iwas a screen was Toeated at 2 iistance b= Sem from tt. Pind: (a) the width of «fringe on the seteen and the number of possible maxima: ih) the maximam width of the sit Syay at which the fringes on the soreen will be still observed rafctey sharp. ‘7a, The distances from 9 Fresnel biprism to. narrow slit and 14 screen are equal to a — 25 em and b= 100 cm respectively ‘The refracting angle ofthe glass iptiom {sequal to 620" Find the wavelength ff light if the width of the fringe on the sereen is Az = 0.55 mm 5.75, A. plane Light wave with wa- vylengih. x= 0.70 jum “falls normally bn the hase of bipriom made of glass (= 1.500) ‘with efracting angle 8 & Hadid “50%. Behind the biprism (Fig. 5.15) ie. 9.6 there is a plave-paralll plate, with the pace between them lied up with benzene (n' = 1.500). Pind the ‘width of a Tringe on the setven Se placed. behind this system. ‘576. A" plane monochromatic Lightwave falls normally on A fringe pattern is formed on a soreen placed at a distance 1 — = {00cm behind the diaphragm. By what distance and in which dicection will these fringes be displaced when one of the slits 1 Chverod by a glass plate of thickness b= 10 yin? ‘S77, Figure S40) illustrates on inteferometer sed fy measure- rmonts of faltactive Indices of Uansparent substances. Here SIs “ y® == |— Fig, 548, ‘narrow slit uminated by monochromatic light with wavelength A2'S80' am, 2 and 2 ato entleal tubes with air of length t= 40.0 can each, D is'« diaphragm with two silts, After the air jn tabe 7 was replaced with ammonia gas, tho interference pattern fn the secon Seas dixplaced upward by N = 17 fringes. The re- faclive index of air ls equal to n= LOQI27T, Determine the retrace ve index of ammonia gas ‘378. An electromagnetic wave falls normally on the boundary ‘bro isotropic dielecteies with refractive indless ny and my ng use of the continuity condition Tor the tangential Som HE and H across the boundary, demonstrate that atthe interface fc fed vetor E (a) of the transmitted wave experiences So phase jomp: (of the reflected wave ts subjected to fe jamp equal to Uf 10s reflected from 8 mediom of higher optical density 5.79. parallel beat of white Light falls on a thin him whose refractive index i= ‘angle of andiens Is By = SO, What must the Alm thickoess be equal to for the cellected Tight. to. be colosted yellow (k= 0.00. yun)” mast intensively? 5.80. Find the ssinimum thickness of fm with refractive index 4.33 at which light with Wavelength 0.64 wm exporionees maximum rellection while light with wavelength 0.40. pam is not relectol at fall, The ineldence angle of light 1s equal to 30% ‘381. To. decrease light Insses due to reflection from the glass surface the latter Is coated with thin layer of substance whose refractive index n’-— yy where n is tho refractive index of the maplitades of electromagnetic oslllations ih coated surfaces are equal. At what thickness of the glass tfletivity tp tha direction of the normal fetal to toro for light with wavolangth 4? ‘582. Ditused monochromatic ight with wavelength A = 0.60 pm falls on a thin fl with tefractive index m= 1-5. Dotermine the ae ‘im thickness if the angular separation of neighbonring. maxima observed in reflected Tight at the angles low to. 8 = 45""to. the normal is" equal. to. b= 3.0" 5.83. Monochromatic light passes through an orifice in a screen Se ig, 5.17) and being reflected from a thin’ teansparent plate. P Produces fringes of equal inelination Sn the sereen” The ‘thickness of the Plate is ual tod. the distance, bee {ween tho. plate andthe sereen if, thercadi of the th and fh dark rings fre /yand re. Find the wavelength. af Tight ‘daking into seconnt that ry 5.84. A plane monoeheomatie light wave ‘with wavelength Tallon the susiace of a glass wedge whose tacos form an angle <1. The plane of incidence is perpendicu the edge, the angle of inctleace is 0, Find. the distance between the ‘neighbouring. fringe ‘maxima’on' the ereen placed. at Tit angles fo reflected light 5.85, Light with wavelength i = 0.55 yim from @ distant. point soare falls normally on the surfaceof glass wedge. Ai ‘whose neighbouring maxims on the eu ta'by saistonce Ae (a) the angle betweon the wedge faces {) the dogeeo of Tight manoehromatism (AW/A) if, the fringes Alzappea fla istanae 1215 cm from the wedze's wie 3:86. The convex surface of « plano-convex glass lens comes into contact with gl “The edrvatare radius of the les's convex surface fhe wavelength of light Is equal to Find the width Sr of Nowion ring ass finetion ofits tus rin the region where aren, ‘S87. The convex surface of « plano-convex glass Ions with curva- ture radius ft’ 40" em comes into contact with 1 glass plate A certain ring observed in rected light has 8 Tadlos 7-= 2.9 mm. Watching the given rng, the lens was gradeally removed from the plate by'a distance ah = 5.0 ym. What has the radios of that ring become equal to? ‘5.88. At the crest of a spherical surfaen of « plano-convex Jens there is a ground-ot plave spot of vais rg —'3.0; mm throogh ‘hich the Teos comes into contact with'a last plate. ‘The carvature Fadius of the leas's couves surfaco ts equal tot = 150 em. Find the radius of che sixth bright ring when observed in reflected light with wavelength 635-1 ‘589: A planv-coovex elas lons with cureature radius of spherical suriace R12 5:0m in prened agaiast 9 glase plate The diameters ‘tthe tenth and fifteenth dark Newton's rings tn reectod light ace qual tod, = 1.00 mm and d, = 150 mm. Find the wavelength of tiene 6 5.90. Two plano-convox thin glass lenses are brought into con jr spherical surfaces. Find the optical power of such 9 system {hin eflected Light with wavelength i 0.60 ym the diameter of the fifth bright ring is = 1.00 mm in symmetric giass Tenses, one biconvex and the other biconcave, are brought {nto contact to make a system with fpllcal power @ = 0.50 D. Newton's tings are abserved in refected Tight with wavelengti % — 0.61 um. Determine: (a) the radius of the tenth date ring (0) how the radius of that ring will change when the space between ‘tho lenses is hilledup with wate. "5.02. The spherical’ surface of a. plano-convex lens comes into ‘comtact with a glass plate, The space between te lens and the plate {s'Blled up with carbon dioxide. The refractive indices of the fe facbon dioxide, and the plate are equal to m = 1.30, my — 1.63, 4nd my = 1.70 respectively. The curvature eudis of thesphetieal Surface of the lens is equal to Zt = 100 em. Detormine the radius fof the fifth dark Newton's ring in reflected light with wavelength £20.50" jm. 45.M8., In a. two-beam interferometer the orange mereary tine composed of two wavelengths %y — 576.97 nim and Ly = 579.03 am ie employed. What is the least order of interference at which the 2h Sa a, 8 erhalten teeth pee ipa ei enice ute oscay Se selena an i A for cul dunt y mec ~~ Mig St focal plane of a lens (Pig. 5.18). The centie rings St th Gtion ts equal 40 3. Determine how thickness ofthe (a) the position of ing: (0) the gular width of frnges Aopends’on the order of interference. 3.6. Por the lon of thickness d= 2.5 em fn the spectesl interval of wave: ths, within which there i ail no overlap with other onder of Inteterence if the observation ie tarred. "Oot approximately” at ‘wavelength A= 0.50 um 4.8, DIFFRACTION OF LIGHT ‘© Radlus of the priory of the Ath Fresnel zoe 1 Carns spi Pig. 5:49). The sumber long that opel corcepoad to the suscr af parumstr Lathe ease of plane wevo eV THR whee + 1.2, ean dius | dbiiety sd ce the daunce defining the potion ofthe lament 4 of waver Finke observation otnt Pas shown in the upper Ie coroet af the manele maton pode blight fling nrmally fom «shit Fainom Hh, R= 128-2 65) whore & tbe width of the ait, Ore deacon sug "sDitmetion.etigs ith igh ailing normally. The main Fravbotr maxima appear vole the condition atin@m Ah km 1 2 oy 39 the condition of sol mann ain ox ihre FH = 1,2, «except or 0,24, "e snglardsptsion of 6leeton gating "Tes om feslving power of diration gating Baan, wat ae 0 here 2 i the sumr of lines of the eating ' hetlvog pouer of an abjtive 6.30) here bp ifthe los saglar separation sestved by the objective, isthe Tisha? oR Sete “ > rng's equation. ‘he condition of Sractoneaxina snhece dia he Interplanar distance, 2 the glancing ange, A= 4, 23, 5.97. A plane light wave falls normally om a diaphragm with round aperture opening the frst XV Fresnel aones for point P ob 1'eeocr Toegtod ot a stance ® from the dlapheagm. ‘The stave feapth of light sequal to A Find the intasity of ght Ty infront tf the diaphragm ishe distribution af Inteanity of light 1 te on the ‘Sroon is known. Here Fi tho distance from the potnt P- B88. 4 point sonree of light with wavelength &— 0.50 um is located t's distance a= 100 em in front of. diaphogm with found apertre of rudlus r ~ 1.0 mi, Find the distance b between the tiaphragm andthe ofservaton point for which the number of Fresnel zones in the aperture equals f= 3. 538, A"diaphragin wth round. apertare, whose radius r ean be ‘varied during the experiment, i placed between a point source af Tigi and stveen. The distanets frm the dlapheegm to the source tnd the seen are egal to 2 = 400 cm and b= 125 em, Determine ihe wavelengti of light tf the tntensity maxiirum at the centre of the Givaction pattere of the sereen i observed at ry 1.00 tm nd the next mia at ry = 129 mom ‘monochromatic light wave with intensity Jy falls que sreea with a fount apervane, What i the ten. sity of light 1 behind the screen at the point for which the apertare (a) is equal to the first Fresnel zone; to the internal half of the itso () was made equal to the fest Fresnel zone and then half of it was closed (along the sdiameter?? 3.401. A, plane monochromatic Tight wave with inteusity I, falls normally on at opaque ise closing the fit Fost zon the observation point P. What did the intensity of light fat the point P became equal to alter (a) half of the dise (along the diargeter) was removeds (b) half of te external half of the first Fresnel zone was removed (oon the aameter ‘3.102. "A\ plane monochromatic light wave with intensity Ty falls normally an the surlaces of the opaque tereens shows i 5.20. Find ‘the intensity of light Pat's point > See 2, a) & ( (0 octet, ehigd the ener points of sees 79 and behind vot ad al ae (oo hich the rod edgy of serene 58 coicies with ope of the Et Bo ore retin oneal fonts tin roles obtelnd for seeene ashe LE VAL Same, for sereens 5-8. = a 5.108,"A plane light wave with wave- + 2 length = 0.60" ys falls normally on 8 | suftcieotly large glae plate having @ round recess on the opposite side (Fig. 5.24). For i the tevation poll P that en cores ponds tothe fist one and a half Fresnel foro Zones, Pind the depth kof the recess at a ‘whieh the tntensity of light atthe point P's Pig 52. (a) maxima () minimum (©) equal to the intensity of sncient Light. 5.104. A plane Light wave with wavelength and Intensity J, falls normally on a large glas plate whose opposite side serves 8¢ Sh opeque seteen with « Zound apertuce equal to the fret Fresnel zone fot the observation point P. In the middle of the apeetare {here {8 round rece equal to half the Preanel zone. What snust the depth af that rveess be for the intensity of light at the point P'to bo the highest? What is this intensity equal to? 5.105. A" plane Light wave with wavelength k — 0.57 qm falls formally ona surfaco of a glass (n'™ 1.60) dike whlch shats one fd a half Freene) tones forthe observation pint P. What must ‘the minimom thickness of that dise le for the intensity of Hight at te point P to be the highest? Take tnto account the intetference Gof Light op its passing through the die 5.106."A ‘plane light wave. with wavolength = 0.54 yan goes ‘hevugh shin converging Tene with focal length J —70"cm ana stop fixed. Immediately after the lens, and reaches "screen placed ata distancs.b Tem fom the aperture stop ‘At that aperture radit has the eentee of the diraction. pattem fon the sereen the maximivm iluminance? '5107."A" plane monochromatic light wave falls gormally on ‘round aperture. Ata distance } —~ 9.0'm from it there fe 4 screen Showing certain difraction pattern, The aperture Acereased 1) = 3.0 times. Pind the new ast Serean should be pasttloned to obtain 1 to the previous one bat diminished ‘5.t08"'An opaque ball of diameter D = 40 mm is placed between 4 soures of light with wavelength ~ 0159 yan and a photographic plate. The distanee between the sousee an) the ball is equal to 2 i2 m and that Between the bell and the photographie plate fs equal to b= 18 m. Fin (a) the image dimension on the pl of the source is y= 6.0. mm; (b} the minimum height of iregularities, covering the surface of the ball at random, at which the ball obsiracts Hight. Note. As calculations and experience show, that happens when the height of ietogolasties ie somparsble with the width of the Pressel sove sour f fad ‘Which the edge of an opaque soreen passes, 45.108. A” point source of moncehromatic Light is positioned fa Tront of = zone plate at a distance a= 1.3/m from ite The image fof the source. is formed at a. distance B= £0 m trom the plate. Find the focal the transverse dimension wave with wave: Fig 8.22, {50m and Intensity Zp falls normally ona large glass plate whose side view is shown in Fig. 5.22. At what hoight 1 of the Ledge will the fatensty of light suiplns dosti uty below be (2) minimum: (0) twice as Tow as Fy (the Loses due to reflection ate to be neglect= ) 3.11. A plane monochromatic Light wave falls normally. on an. ‘opaque half plane. A sereen is located at a distance b= 100 emt behind the half-plane. Making use af the Cornu splral Pig 8.19), nd {a) the ratto of intensities the Urstmaximum and the neighbour ing. minima; {hy the wavelength of light if the fest two maxima are separated boy. t distance y= (163° ma. ‘5.142. A plane light wave with wavelength 0.60 ya falls normally 4 strip 0.70 mm wide. Behind its screen ta placed 100 em. Using Fig.'5.19, find the ratio of intensities mm pattern and at the edge of the Heometrical ‘5118. A plane monoeicomatic Light wave falls normally on 2 long rectangular sit Behind which a sereen ts positioned. at a distance “00 em. Fist the width of the slit was adjusted so that in the le ofthe difraction pattern the lowest minimam was observed. ‘Afr widening the slit by Ah— 0.70 mm, the next aninimuan was ‘obtained in tho eentre of the patton, Find’ the wavelength of light S144. A’ plave. light wave with wavelength A O.5. pm. falls rally on age dam plate whe open side ha Ingrecta fpular recess 0.60 mm wide. Using Fig. 519, find the depth h af the recess at which the + + taht 4 tories Wn he iadmcs fo pet eed Sil¥5. A. plage: lightwave with wave =o loogth 4 = 6.05" yan falls normally ea a lane lass plate whose oppoctte side has at a distance b= 110 cm from the y intensity of ligt at point 2 of the Fig. 5.23 the highert_ possible” Makin 20 cut on one side (Fig. 5.24). The edge of the cul coincides with the Boundary line of the fist Fresnel one for the observation potat P. ‘The width of the sit ineasares 0,00 of the ead of the cut. Using Fig. 549, find the intonsty of light at the point P. ‘17. A plane monochromatic light wave Talls normally op an apa ran with Tong alt whowe shape te show io Pi. 9.2. Making ‘use of Fig. 5.40, find the ratlo of Intensities of light at pointe, 2 and 3 Teeated behind the screen at equal distances from Por point 3 the tounded-of! edge of the silt coincides with the Boundary line of the frst Fresnel zone ‘118. A plane ionochromatic light wave falls normally on an ‘opaque tereen shaped a8 Tong strip with a round hole ia the madd Por the observation point P the hole cceesponds to half the Pres one, with the bole diameter being oy ~ 107 times Tess than the Awidts of the strip. Using Fig. 9.19, bud the intensity of light atthe Point P provide tat fhe fteniy of the inert igh equal sifi9, Light with wavelength 2 falls norinally on along eeetengu- lap sie of width b. Fld the angular disuibution of the tntonsity of Light tn the ease of Frasnoferdifrstion, a9 well as the angular position of minim. 129. Making use of che result obtained in the foregoing problem find the conditions defining the engular position af maxima of the frat, the second, and the third ander. ‘Sl2K. Light with: wavelength 2 width 6 10 yim at an angle Oy — 30" tol ite normal. Find the Singular position of the Rest minima located on both sides of the Central Fraunhofer maxianom, Light wave with wavelength 2 ~ 0.60 yum falls normaly onthe face of glass wedge with refracting angle 6. 15° face of the wedge i opaque and has a silt of width to the edge. Find’ {@) the angle 40 between the direction Lo the Freunhofer maximum ‘of zesath omer and that of incident Tigh () the angular width of the Fraunhofer meximum of the zeroth ede, 5.128. A monocliomatic beam falls on g rellection grating with period d= 10mm at a glancing angle a ~ 1.0-, When tts dif Fncted at « glancing. angie. 3." a Feaunholer maximam of second onder gceurs, Find the wavelength of light, ‘2h. Draw the approximate difraction pattery originating in tue eae ofthe Frater deacon fom a grting anti tf thre identical slits if the Fatio of the gating’ period to the sit ‘width is equal to {a) two: 0) three 5.125, With light falling normally on a diffraction grating. the angle of difraction of second order Ls equal to 45° for wavelength 44 = 0.85 wn, Find the ange of difactioa of thin ondr fora wave Teagub y 0.50 um. ‘3126, Light with wavelength 595 nm falls normally on a dilfrac- tion grat. Find its poiod if the dieacion angle 2°coerespoude 10 of te Fraubofer maxi and the higbost oder of sperm is equal to ive Et Fin Whe mevlength of monochromatic ht fing ao amally ob difracton geaing with period d= 2.2 um if tho angle Between the direction to the Frausofer maxine Of the hrs and the secood order in aqual 089 = 49 ‘5.128. Light with wavelongth 530° om falls on a transpacent Aiftaction-grating with pestod 1.30 yan ind the angley relative {o the grating normal at whieh the PYaushofor maximus of highest fonder fe observed provided the light falls ou the grating (a) at right an {B) at the angle 60" to the normal 5.128, Light with wavelength k= 0.60 ym falls normally on a dttsctiom grating iaseibed on «plane nutface of a plano-convex {slindicl lass Inns with curvature radi fl = 50 ca The peviod of the gratiog is equal to d'~ 80 ym. Fiad the stance between {he principal Iocal plane of 5.190."A plane Light weave with wa soraally 0 th face of 4 = 0.50 pm falls 6 Sala the 080 lines are parallel to the gee edge. Fiad the anglee that the direction of incident light forms with the dlreetions to the principal Fraunhofer maxima of tho zero and th a, What Is thw highest order of the spect ‘rum AU what angle to the droction of incident light ie it observed? SASL. A plane light wave with wavelength i falls normally om 1 puso’ direction grating whose side view is howe in Fig. 2.26, ‘Tho grating Is cul oom glass plate with refractive index m_ Find the depth fof the Lines at which th Intensity of the central Praua- hofer maxisvum is equal to aero. What is inthis ease the dillection angle ‘corresponding to the brst- maximo? fet tap dd dead Pig 5.28. vg. 5.27 5.132, Figure 5.27 illustrates an arrangement employed in obsor- vations of diffraction of light by ultrasound. A plane light wave with wavelength — 0.5 jm pases through the water bla teak ma ultrasonle wave is sustained ats frequeney Vio 47 MHz. AS a result of diffraction of light by the optically Inhomogeneous pertodie structure ditaction spectrum canbe bserved in the focal plane of the objective O with focal length ‘35 em. The separation between neighbouring maxiai is 2 — (0.60 mms. Find the propagation velocity of ultrsonie oscillations water 51185. To measure tho angular distance ¥ between the components of a double star by Mickelon's method. in front of telescope's Ios a diaphengm was placed, which adi two narrow parallel sits separated by an adjustable distance a While diminishing d. the frat smearing of the patteea was observed ia. the foeal plane of the objective at d= 85 cm. Find y, assuring the wavelength of light 1 be equal to A = 085 ym. 5.18%. A transparent difleaction grating asa potid & — 1.50 ym. Find the angular dispersion D (n-angular minutos per nanometre) corresponding ts the maximum of highest onter fora spectral Line Gf wavelength % — 590 num of light falling on the grating (a) at right anges; (at the angle 8, = 45° to the normal. 135. Light with wavelength i falls on-a difaction grating at right angles Pind the angular disper of ifegetion angle 8. ‘of the grating as function ngth A — 589.0 nm falls normally. on k period = 25 ym, comprising N= 50 000 linet: Find the angular width of the didraction maximum of second order, 5.197. Demonstrate that when light falls on a diffraction grating at right angles, the maximum resolving power of the grating cannot txcoed the valve Thy where T is the width of the grating and bs the ‘wavelength of Light 5.198. Using a difraction grating as an example, demonstrate thai the frequeney diference of two maxima resolved according to Rayleign’s criterion te equal to the reciprocal of the dilference of prpagation times of the extreme interfering oscillations, fe. 6v = ree 5.139. Light composed of two specteal lines with wavelengths 00,000 end 600.050 nm falls normally. on’ diffesction grating 40.0 mm wide, Ata certain diffraction angie O these lines ave close to’ being resolved {according to Ravleigh's exiterioa) Find 3.140, Tight falls normally on a transparent dilfaciion grating of width 16.8 ca with 200 Tines per millimetre. The spectrum under investigation. Snclides”a spectral Tine with A= 64S. nm Consisting af two components difering By 8A — 0.015 am Find: (a) In what order of the spectrum these component wll be resol oat (by the least ditference of wavelengths that cam he resolved by this grating in a wavelength fegion b = 670 om. the IL order of the spectrum, Evaluate: a) the period of this grating: (G) shad mast be the width of the grating with the same period for'n doublet i = 460-0 nm whoso components differ by 0.13 nm tobe resolved in th thitd order of the spectrum. sls A teansparet dieaction seating of x quastz spectrograph 4s 33 mm wide aa has 200 ilaes per millimetre. The foal length Of an objective in whowe focal plane a photographie plete tr Located fs equal to 8) can Light falls'on the grating at right angles, ‘The Spectrum ‘under investigation includes 4 doublet with components cP ravelegthe 004 and 31.184 0m, Determine: (@) the distancos oa the photographic plate between the components of thie doublet in the spectra of the frst and the second order (@) whether those components wil be resolved. In these orders ot he sper. Sut The altimate rosolving power 218A of the spectrosaph's trihedrat prism ts determined By diffraction of light at th tages (as inthe case of 8 feast deviation. angle in’ accordanco with WOR = 0 | dnd, ‘where dis the width of the prsm's bo Aispersion of is material. Derive tl 5.144, A spectrograph’s toibedral prism is manufactured from lace whose refractive dex varies with wavelength asn—=4 3/38, Shore" and: Bare constants, with belog equal to 0.010 wm Making vse of the formula from the foregoing problest, fad {c) how the resolving power of the prism depends on 4; calculate the value of 0k in the vicinity of by = 4@hum and hy — 696 om ithe width of the prisms bate Is’ = 3.0 em: (by the width of te prism’s base capable of resolving the yellow doublet of sodium (580.0 and 389.8 nm), 5.18. ow nie te se of 9 ida par whic a the same resolving power 3s diffraction grating with 10 000 lines Inthe socond’ order of tho spocteum i] d/dal= O10 ym="? a 146. Thoro is a telescope whose objective has @ diameter D = '3.0.em. Find the resolving power of the objective atd the minke ‘mum separation between two points at dis 0 kan Irom the telescope, whieh it can resolve (assume. ~ 0.39 yan) 5.167. Caléulate. the minimum separation between. wo points ‘on the Moon which can be resolved by relleting telescope with imicror diameter 5 m. The wavelength of light ls asuimed to be equal toh = 0.39 4m. 148. Tetormine tho minimum multiplication of « telescope with diamator of objective D = 5.0 em with which the resolving power of the objective is totally employed if the diameter of the Byers pupll is dy = 4.0 mm, 448. There is @ microscope whose objective’s aumerieal aperture is sin = O24, whore a It the hall-angle subtended by. the objec tie’ tim, Find tho minimum separation resolved by thie microscope ‘when an object is illuminated by light with wavelength b — 0.55 ua 7150. Find the minimum magaiention of mieroso Ing power of the objective is totally employed if the diometer of the yes pupil ie dy — 40 mam, 5.15. A, beam’ of Xaye. with length 2 falls at a glancing ie f.0° on linear chain of saeingcentee With period ‘ind the angles of incidence corres- * peso fonding taal difracion marina M4 fee : S458" beam of anys with dit wavelength kay fale mor gL mally on plane rectangular arrey ttt of seatering canes anf procs Sete O Sutem of difietin oasinn 02s trig 5 on a plano ston To =| tnoved from the array hy a distance TO’ em. Find the array pesiods a tnd B along the 2 aud y axes if th ig. 5.29, Aistancas boteen syamerically located: maxima of second ordor azo equal to Az — 60 mm (slong the 2 axis) and Ayo i) mm (along the axis) 499. A beam of X-rays Impingee on s three dimensional rectan- gular array whose periods are a, 0, and 2. The dlsgetion of te iat ‘lent beam coincides with the ditvetion along which the arzay period {sequal to a. Find the divctions to the difraction maxima aad the wavelengths at which these maxima ‘will be observed. vin Anarvow beam of X-rays impinges on tie natural facet of @ NaCl single crystal, whose density Is p = 210 grea! ats glanc- lng angle = 00.0. The mirtorrelletion from this facet produces maximum of second order. Find the wavelength of radiation, 5.185. A beam of X-rays with wavelength i= 174 pm falls om the surface of a single ervstal rotating about its axis whch is paral: ost a Jot to its surface and perpendicular to the direction of the inid Iam, this ase the difetions to the maxi of second and thi ‘Seder from the system of planes pavael to the surface of tho siagle fiptl oa ttle =" fete they, Fad tho eoenponding inlerplanar distance 'S1b6. On transmitting « boar of X-rays with wavelength 2. — 78 pn through a polyergsalline spectmen spstem of fac tion rings ts produced on a sereen located ata distance 1 15.cin from ‘the specimen. Determine the radive of the bright ing comes: poudiug to sceond order of rection fom the stam of planes with Eiceplanar sintance = 185. pan Sc, POLARIZATION OP LiGHT f+ Degre of polarization of Hah a fs Moa Bayes’ law 4) 4. Fremel egatins for stents of Haht reflected atthe boundary. bee tweed ono aia tanto, That Sareea)» AT aera whee 7 any te the intensities of inldot gt whos lcs ecto eTe intone de rseively pependicalar and. prallel tothe plane of incidence ‘tipnlin plate bate to flares P ad Phe angle ote eerie ie opted axis 60" of the plate to eae) to 4 Tg sel pate cag te plait fon on to Be lotag edition: say the aie Mele’ or tlm tice the fe pin [ saw [ssattnn pana | een | je feet | rome | flue Hie 8.5 284 plhi the_ pe esoace Btwn the ein and n> Natural and taagnetie rootion of the plane of polarization: peor Gaga VU, ea whee the rotation constant, ¥ i Wede's constant 5.157, A plane monochromatic wave of natural light with inten- sity Zy falls noraally on a soreon composed of two touching Polarolé hlt-planes. ‘The principal direction of one Polaroid i parallel, and of the other porpendcalar, to the boundary between them, ‘Wat kindof ditraction pastor ie formed behind te sero? What Ist fntonityof light behind Ths screen atthe potato the plane ferpendicular io the seven and! passiog. trough the Doaadary Bstwoen the. Polarost ‘Su, 4 plane monochromatic wave of antral light with inten: sity pals ormally-oa on opague sree with round ble comme. Fouditg to tho hist Freel tone Yor the obssvation point P. Find The intensity of light the pat after the bolo wa covered with two Meutic Polaroids whose pracial directions are mutually Fespenicilar andthe boundary Between them pase {Q) along the diameter of the hole 1G) Sone the cicumfernse of the circle Kimiting the at halt of the Freel zone 51450 A oat of plaorpolarized light falls on a polariaoe which rotates about the au of tho tay with angular veloc w= 21 Pada, Find the aongy of light passing through the poariage por one roo Huta if the Bur ofanergy ofthe tneieat fay ie equal to Oy = 20 mW SHON beam of natu igh fas on a sytem of Nw BNicad prime tose toumsion planes ave trad each through a angle {Fe'3o" with respect 0 that of the foregoing prism. What fraction luminous doa pases through this system? .16l, Netaral Ligh falls on system of thro identical in Polatoléy the pyintipal dietion Of the male Polaroid ura fn angle y ~ bt with thee of two other Polaroids. The maximum ‘Tunumsaon coicient uf exch Polaroid is equal ta + = O81 when eepolaaad ight falls on thea, How many. ties wil" Icy "athe Might craw site pasog Cog he sya SSR, The degree of polarization of partially polarized right» p35, Find the catio Of inonntes of th polarized component ‘1 ie light"ande aataral.componeat 5.83, A Nica primis placed in the way of patally polarized been light: When the prist Is turned frm the position maxi Sun sransimason trough an sngle q = 60 the ineaaty of trans titted light decreas by a Tacoe Oty = 40 Find the detest Polatatin of incldent ght i164 Two identieal imperfect polarizers ace placed la the way oft tual beat of igh. When the plarae plane rw parallel, the tem transmits y= 40.0 titer more Tight theo’ in the {ti “Pc pans. Fad the ee of polation of ight produced (a) by each poasiner saparatly: 1S Theale Ss the ar of ee polar re paral "SUG, Two parallel plane-polried beams of Hight of equal inten sity whos oseiation planes Ny 200 fora sam angle etween ‘them (Fig. 5.30) fall on a Nicol pris, To equalize the intensities of the beains emerging behind the prism, ite principal direction ‘aust e aligaed along the bisecting Tine “Aor. Fad the value of the angle g at which the rotation ofthe Nicol prism through «stall angle Sg &@ from the position results in the fractional change Of intensities of tho Beams A//1 by the value = 100 times exceeding that resulting due to rotation through the same angle from the position B. 5-106. Resorting to the Fresnel equations, demonstrate that Light reacted trom the surface of dielectric will be totally polarized {tho angle of incidence 8, satisiee the eondition tax 0, — ne where n is the refractive index of the dielectric. What is in (his case the Angle Dotwoon the rallected and 3 irate. a { | Mig, 520. Pig 8a 5.467. Natural light falls at the Brewster angle on the surface of glass. Using the Fresnel equations, fd (2) the ‘lection a}, 10 dating a) light with Iefthand circular’ polarization trom: that with right-hand polasizatic “h) aatural light om Light with circular polarization and from ‘the composition of aatoral light and that with eireular polarization, ‘5.188. Light with wavelength } falls on a system of Grosead pola rier P and analyzer A. between which Babine! compensator © {inserted (Fig. '5-88). The compensa for consists of two quarts wedges with 0 | of the compensator. The refracting engle ‘of the wedges fs equal to 8 (Oa) and the ‘dilference of Tefractive ‘ndieee of 7 ca (@) th ‘magnitude andthe sign of the optical peth diference of ordinary” and extraordinary ays, which appears” due to. the Sample 5: plat the optical axis of one of them being parallel. to the edge, and of the other, Perpendicular to ty The prlueipal diese: quartz Is. ng — ng. The insertion of Pig 533. {vestigated birfriagont sample S, with the optical axis oriented as shown in the figure, results in dis- fons of the polarizer end the analyser form an angle of 45" with the optical axes placement of the siages ‘upward. by 62 mm. Pind (a) the width of the fringe Bz: 5.189. Using tho tables of the Appendix, ealeula of relractive indies of quarts for light of waveleng ‘ith Tiehthand and lelthaad ireular polesieations. 100. Flane-polaized light of wavdength 0.98" yum fells on Gia gua pi, FP 9 3 wih rating angi "G0" Inside the prisa light: propagates along. the opts! xi ‘howe diction is shown by hatching” Behind {he Polaroid. Pol an intelarence. pattern ot nd dark tringer of width ae = is oheorvod. Find the specie rota tion constant of quarts and the distribution tf iovensity of light bebind the Polaroid 5.101. Natural tonocbromatie ight foils on system of two cromsed. Nicol prisms Between which a quartz plate cat af right Sngles to" its optical axis ie verted. Hd {ho mista Cire of the plete at which {hs apsom wil trapamity = 00 of lagu Ts Th the” specie rolation comet of (quarts ie equal "to ee ang. dog 12. Light pases through system af two exosod Nicol prisms botifoon which 8 quarts plate cut at rght angles ta its optical axis in'placed. Deteraine the misimum thickness of the: pate whieh Slows ight of wavelength 438 nom to be completely eat of by the system and tranamits hall the Light of wavelength 407-am. The spee fife Totstion constant of quart? for these wavelengths is equal to 4.3 and. St-t angular grea per mm respectively ‘5.183. Plano polarized light of wavelength S80. nm propagates slog the aso rind eae Wess lind with eighty end the diferonee ‘= 580.5 nm Fie 544 Aetermine the specie rotation constant uf the solution ‘3.194, A Kerr eal is positioned between thu crosed Nieol prisms so hat ‘the direction of electric Hold E in the capacitor forms an gle of 45° withthe principal directions of the prisms, The capacitor bas the length =H a nnd ld path sobesene, Light of wavelength — 0.50. an passes throvgh.thesystem. Taking {nto account that jn this Cae the Kerr constant is equal 10.8) 2°f6e emiV, Kind (@) the minimum strength of electric field £ in the capseitor at which the fntenalty of light that passes through this system ie ade Pendeot of rotation of the rear prism: () how many times per second Tight will be interrupted whan a sinusoidal voltag of frquency ¥ = 10 Miz and strength ampli- tude qm 30 kV/em is applied Yo. the capactioe, Note."The Rert constant fe the cotfeient Bin the equation My — ne HAE 5.195. Monochromatic plane-polarized light with angular frequen- 22,2 pong {ico eltain eubanc along a volorm magnetic fold 2/2 Find the diffrence of refractive indices for right-hand and leftchand components of light heam ‘with circular polarization if the Verdet constant is equal to ‘3196. A cartain substance is placed ia a longitudinal magnotic field of a solenotd located hotweon two Polaroids. ‘The length of the tube with substance is equal to T= 80 em, Find the Verdot constant if at a field strength 17 = 56.5 A/mm tho angle of rotation of polarization plane ts equal to 9) = -Fs°Il’ for one direction of the field and’ to 7p = 320". for the opposite direction, 5197. A narrow ‘eam of plane-polaried light passes. through dextrorotatory positive compound placed lato « lougl tudinal magi Ue field as shown in Fig. 3.5. Find the angle vbrough which th a 7 ig. 5.5. polarization plane of the transmitted beam will tura if the length Of the tube with tho compound i equal to f the specife rotation Constant of the compound’ is equal to a, the Verdot constant is V, jth = 20 ca led with bons in't longitudinal agaetic fof «solenoid po Folarids. The ngie betwee the grata Gestion af the Pol. roids is equal to 43". Find the misianum strength of the magnetic fold at which light of the wavelength 380 nan propagats through that system only in one direction (ptical valve). Wet happens tho diction of the given magnetic Held is changed to the oppoite 5.199, Bspevienco shows that a body irradiated with light with ila polation aegie fray Th hapary bce soh {light posessos an angular momenta whose Bow density ia va sums is'equal to Mf =l/a, whore I is the tatenity of igh we the anguar” oscillation Hrequescy.. Suppose light’ with cirelar larization and wavelength b= 070 um falls normally os an form black dsc of tass'm ~ 10 mg whieh ean freely rotate about AWaxis: How soon, wil augular volacity become eqaal "9 sy 0 radie provided 1 = M0 Wien? 55, DISPERSION AND ABSORPTION OF LIGHT ‘+ Permitivityofsobtace aecoring to elementary theory of dsperin: ‘ty - | os he ete a I 7 vm wk, wm dulth, (G.5c) a. (85a) ‘+ Atumnaation of natow beam of eletonmgneic ration Tastee ™ os) ter n= fH ath one of ne anon, seep Bie 5.200. A free elciron is located in the field of » monochromatic ight wave. ‘The latensity of light {s 7 = 180 Wit ite Trequency fey 3.4108 anh Find (a) the clectron's Gecillation amplitude and its velocity ampli suet () the ratio Fy/fy, where Fy and fg ate the emplitudescof forces with’ whieh the mlagsotie snd electric components of the Tight wave fel aot‘on the eletron demonstrate tht that ratio is gual to Lvl where v is the eloctron’s velocity amplitude and ¢ i the velocity of light Tnatruetin. The action of the magnetic Held component can be egavded in the equation of motion af the electron since the calcu. fans show it to be nogligble Tn ‘plasmaa is equal Ung. Neglecting the interaction of the wave and plasma fobs, nd (a) the frequency dependence of plasma permittivity, (0) how the phate velocity of the electtomagnetie wave depends ‘wavelength 2 ia plasins. 202" Find the fro sleetton couconteation in ionosphare if its reftactive index is equal ton ~ 0.90 for radiowaves of foquency v0" Nile 203. Assuming electrons of substanca to be {rae when tubjocted to hard Xcrays, determine by what magnitude the relractivo indox of graphite differs from uaity in the eae of Xray. whose. wavelength Sn vacuum is equal to} SD pan. Ea ‘5.204, An electron experiences « quasi-alastic force kz and a “Iie tioe force” in the eld of electromagnetic radiation. The F-com- ponent of the field varies as B— By cos we. Neglecting the action Of the magnetic component of the Beld, Sind (@) the motion equation of the electron; (@) the moan'power absorbed by the electron: the frequency at which that power is maximum and the expression for the max ‘Bum mean power ‘3.200. In Some cases permittivity of substance turns out to be a complex of lve index, respectively, ‘) quantity. Write the “oF an Imaginary (v fon of plane wave for both of thei he fd nut he pha p = Intenng of such rftctne indie ; Soe: 4 scundig of dilute plas 1 ky radiowayo of various fequencle: | evens tha nadowaver wih wave | a Toogths exceoding fy = 0035 m expe. Fence total interoal rlletion, Fad {ho tre’leeton “concentration in 2 that, plasms, 5.207. Using the definition of the Fe. 58 group, loety drive Taylih's formule (6.54). Demonstrate that im. the vsinty of k= 7 the velocity a is equal to the segment veut by the tangent of the org Fatt. oi ie 8 ‘5-08, Find tho vaatton betweea the group velocity w and phase velocity for the following dispersion laws G) iV ook @ vertier Here i, hy and w ure the wavelength, wave avmber, and angular S00 cum the relationship Dor ins certain medium the relationship betweoo the group and phase velocities of an clectromaguetic wave bas the form uo or whee isthe velocity of light vacutit Fi the dependence of permittivity of that metium on wave frequency, © (a). 2210, The relractive index of carbon dloside a ibe wavelengths (oh Si, aaa 0am eal to OG, iad 28 pct y-"Calelate the pate aad group elects of ight ta te icity oF 58h aga eS RN _ S211. A tra of plane light waves propagates in the medium wir the phase vlotty © Se linear fusctou of wavelength v= ate tome positive constants Demonstrate {hatin such medio the shape of am arbitrary teeta of ight wa ferval t= tb. of Hntenaty Zy falls oo syst of two cron Nicol prisms betwecn which a tubs filled with certain solution fs placed in » longitudinal magnoti Geld of strength H. ‘The length of the tube is f, the coefficient of linear absorption of ton Is, and the Verdet constant ts V. Find the intenlty af Tight transmitted Uhroogh that syste, "Ria Apne monotonic Tlght wave of intensity Fy fll normally ou" a plane-parallel plete both of whose surface have ‘reflection coeftelent o. Taking into account moltiple Tefections, find the intensity of the transmitted light I (a) the plate is. perfectly transparent, Se, the absorption is sent: 1) the coolictent of Linear absorption f equal to x, and the plate thickaes, ie 6 a 3.214, Two plates, one of thickness d, = 3.8 mm and the other of thickness dy = 80 mm, are manufactured from a certala sb Stance. ‘When ‘placed alterately in the way of monochromatic fight, the frat transmits vy ~ 084 fraction of iumioous flr and ‘the second, t, = 0.70. Find the coefficient of linear absorption of that substance, Light fells at ight anges to th plates. The tecond- sry reflections yet A eat snl He “3 identical plane-pa plates ‘0.30 ea The cbeficieat of refetion at 0, Thera the Inet of ig ta ile of plates to. tho intensity of incident light Ts fopleting the secondary reflections of Tight, Aad the coveent of the ziven glass. 5.216. A beam of monochromatic light falls normally on the ane-paralel plate of thickness "The abaorpicn coef lent of the substance the plate it made of varies linearly along the normal to its surface from my 10m. The ooefhcint of rellction ‘each surface of the plate is eal 13 p. Neglecting the scondary Fefletions, find the transmission coeficent of such « plate LBZ. A beam of light of intensity y Tals normally’ ona tans: parent plane-parallel plat of thicknet Tho beam contains all the ‘avelengths inthe interval fom by to A of equal spectral intensity. Find the intensity of the transmitted bean i in thin wavelength interval the absorption coeftent is a linear function of 2, mith txtreme values y and xy ‘The coffin of rellection at each surface 1S agual to\p. The stecadary rellections are to be neglected 18. A light Alter fa a plate of thickness d whose absorption cosclent-deponds on wavelength 2 as xQ)=a(L— Hem, where « and hy are constants. Find the passband AA of this light Alter, that ete bend at whose edges the attenuation of ight 7 Limes that at the wavolangth dy. The eoeffetent of reflection from the surfaces of the Tight Biter {s Sasumed 1o be the same at all wave: rengths, eh ranma rough 0.36 298 5.219. A point source of monochromatic lig {tox @ is positioned at the centre oft spheric ‘The Hnside radius of the ayer isa, the outside oue is &, The coeth- cient of linear absorption of the substance is equal to, the rection onfcieat of the surfaces is equal to p. Noglcting the secondary Folloctions, find the lnteasity of Tight that pasos through that layer. 20. How many timer will the ‘atemsity of a narrow Xeay’ ‘oan of wavelength 20 pm deerease after pasing through a feat piste of fhckaes d= 4 mm i the must absorption concent for the given radiation ‘wavelength is equal Io yup = 3.0 ema? S.221."A narmow bum of X-ray radiation of wavelength 62 pm peuetenies an alumioium screen 2.8 em thick. How thick must e"Teadsereen be to attenuate the beam just as much? The rmass bsorption efficient of aluminium and lead for this radiation ual to 3.48 and 72.0 cme Texpeetively: 15.222 Find the thickness of slomiaium layer which reduces by halt the intensity of « narrow mouoelromatie Xray beam if the errepoadiag mass absorption cefiient ie wip = 0.22 ems. S'2B3. How many Sl%bctbmoption layers ate thre in the plate reducing the intensity ofa nartow Xeray beam 1) ~ 50 times? iting luminous Tnyer of substance 8, OPTICS OF MOVING soURCES 1 Doppler ofet for << Las ta) inthe wlosiy of » sure, Os the ale Sotween the source's motion ‘nd obra ee ae ‘Doppler eect In the genta cam way 10 ny transverse. (0) Tiendanthgyag ges he titi propeniondireton edb a 4, Tn the Fizeau experiment on measurement of th tthe distance Batween tho goat wheel and the 0 kan, the number ofacth is 2 = 720.Twosuccessive dis ances of light aro observed at the following rotation velo the hea:'n, 989 pe and ny ~ 313 rps Find the wlocity of cht vloity. 5.2058 sus f ight nove wth vei lative to receiver. Demonstrate that for vee thy Ioctonal vasiation of frequency flight is delined by Ea. (6.63) “ 5.208, One af the spetial lines omitad by excited He* ions as a wavelength k= fi am. Find the Doppler shift 32 of thet line When obierved:at an angie 6 30° to the beam of moving tons Powessing Linetie euengy T= 10 MeV. ‘227. Won a spectral line of wavelength } = 0.59 pm is ol served in the directions to the opposite adges of the solar'dise along its equator, thre i diferencia ravetengths equal toh = 0 pu Find the peried of the Sun's revolution about its owa axis. ‘5228. The Doppler elec has made it posible to Uiscover the seubl tar whic ay oat in hi nln by naa of ‘telescope is Hmpostible. The spetral igs of tac stasperfdical Become doublets indicating thet the radiation does con from t¥0 stare revolving about their centr of mass. Assuming the tases abe tno ho be oa dt tate ten thm ead ‘thoir masses if the maxintun splitting ofthe spectra Tins i equal aR 20a ecars ery = Saye 5.220. plane electromagnetic wave of Foqvencyy falls normally nthe surlae of 2 mimir appronching with ttivbaie velocity Te Makiog we of the Hopplerformula, nd the frequency ofthe velet- ed wave. Simplify the obtained expresion Yor ho cave Vic '5:280. A radar opeates at wavelength 2 ~ 30.0 ea Pind the cleity of an approscbingaiteraft ifthe boat frequency between te transmitted signal aod the tigal selected font tho aircraft ie sual to dv = 1.00 Kile at the vadar fenton ‘281. Taking Into account thet the wave phase of — kx is an invariant, i. it retains ite value on aneiten ftom one aerial frame to’ another, determine How the frequency aod the wave timber k catering the expression forthe wave phase are transformed. Examine the wnidimensonsl cae SBE How fst doo #cartain nebulerocde if the hy rugen line 2 208k aa te spc Se plc by 150 nm ower Tomer ‘eavslengths '.283. How fast should car move forthe driver to perceive a red trae Tight @ = 0.70 rn) as a green one fs = 039 pn)? B.204. An observer moves with velocity 1 = slong a stright linen font of im a ooree of monochromatie ight moves with velocity vy = cin the same direction and along the same seught Un. The proper roquency of ight i equal to wy, Fin the frequency Fgh ita hy the shearer. SBS. One of the spectral lw of atomic hydrogen bas the wa length } 086.3 am, Pind the Doppler shift Av of that line breed aright anges tothe bea of hydrogen atoms with ki saorey 7'E0 MeV (he teansvere, Doppler efos)y 5.288. A source emitting electromagnetic signals with proper froguency wp ~3.0-10 st mover at a constant velocity Ol80 ealong a straight lie soparated from a stationary observer P bya distance (Pig. 5.57) Find the frequency ofthe signs perceved by tho observer at the momest when (2) the source is at tbe point O; (B) the Ghserver sees it'at the point 0, 5.257. A narsow boas of electrons passes immediately over the surface ofa mall mirror with a diftsctoa grating with peviod E'S um neil oh ety monet ead Gormparatle toc, at ight aagles to the liner of the grating. ‘The “ tron can‘bo sen tn the form ofa srp, whore 5.39). fatorpret a observa {this phenomenon. Fld the wavelength of the stan langle. 0 = 45° 5.238. A gas consists of atome of mass m being ‘eqguilibriom at temperature 7 Suppose uy (9 the natural frequency of light emitted By the atoms. (2) Demonstrate that the spectral distribution of the emitted light is defined by the formula thormedynamie I (dy is the spoottal intensity cormesponding to the frequency ap, 2 meDkT) "Find the width of tho Tye-stt-atoe, Wve width Sa/ayof a given spectral line, ie Tete the requecles at which" 7 Find the velocity of that wave In the frame iif the refractive index ff the tedjunt le equal to'm and the propagation dieetion of the swave coincides with that of the medium ‘3.240. Aberration of light is the apparent displacement of stars attributable to the eflect of the orbital motion of the Earth. ‘The ‘lneetion to a star in the eliptic plane varies periodically, and the Sar performs apparent oscillations ‘withia sm angle 80 — it" Find the orbital velocity of tho Earth 2 5.241. Demonstrate that the angle @ between the propagation dieetion of ligat and the axis transforms on traasition from the ference frame Kt0.K" according to the formula whore f= Vie and Vis the tthe frame K. The = and 2" axes of the reference frames coined 5.242. Find ihe apertura angle of e cone in which all the stars ocated in the som-sphore Tor an observer on the Earth willbe visible Hone moves relative to the Earth ‘with relativistic. velocity V fering by 1.0% feo the velocity of ight. Make Uso of the formula Of the foregoing problem 5.243, Pind the conditions under which « charged particle moving wuniformly through medivm with Telractive index m emits Tight ihe Vavilov-Chereakov effec). Fiad also the direction of that the particle at various momouts of time ‘Sif, Find the lowost values of the kinetic enrgy of an electron snd’ proton causing the emergence of Chovenkov's radiation Ia ‘vmedinm with refractive index m~ 1.60, For what particles is this miatmum value of kinetic energy equal to Tin = 29.6 MeV? ‘5.245. Flad the Kiaetic energy of electrons ealtting light. ia fs medium with refractive index m= 1.50 at an angle 0 — 00" to their propagation divection * QUANTUM NATURE OF Lic Mens eae) wre is the 1 radiston ones. ‘Wiew'e formula ani Wle's dplacemen ae te WF (aT, Tig = 8 om the wavsength coempnding othe maximum ofthe fonction. oor i319 fae ate Suton ete a 1 Bless eget ‘nae toma EE, om 1 Compton tee Sim tate 4 — cond}, om sere te Ane ie Comptin’s wavelength, 5.246, Using Wien's formula, demonstrate that most probable ralistion frequency. yr oo Ts mmaxinium spectral density of thermal Fadietion (4e)mgx 22 Scare The temperature of one of the two heated Back bodies i 1,~ Sok. Fat the temperntare of th stor body tthe waves length corresponding’ Re maaimum emissve capacity excels By Si 0.50 gm the wavelength corrspondiag to the mavicta ssee capecity ol tho hist Back Badly 12 te radon le lack by is My = 0 Wie. Find the’ wavelength coresponding to the masini emlssive capac of that body, pene 2210. "The spectral composition of sue radiation ix much the same a that aa black body whose maslmum emission corres ot {o'the wavelongth O48 pm Find the mae Tost hy the Sun every Second de to radiation” Bvalate the tne fatorval ducing Wile 1 per vot 0, Find the temperstare of lay foniaed hydrogen plasma of iensity p= 0'10 rem a which the thermal cadatien pressure cal tite gs kinetic pressure ofthe particles of Para. Take Jnto account tht the thermal radiation prore p =a where ie the pace density ol radiation energy. at igh compere ll Substance obey the equation of state of a ideal ps 251. A copper ball of diameter 212 cm as placed in an xacvated vesal hse wall ae kept atthe abolue fro tempor {ore The inital Yeoneratare ofthe all = 300 Re Assuag the surface ofthe ball to Me absolutely Mack, find how nm temperature decreases = 20" times SSE, ‘Thre a to tates Fi. 90) with nmal hole of equal fmelers d= 1.0 em and petctly reucting ower surece, The Fu. 5.39, between the holes is f= 10 cm. A constant temperature io Ke inated to cavity 1 Calla the segy-state twmperature inside cavity 2 Tatruction. Take into account that a black body radlation obeys the cosine emision Taw. ma radiation frequency; (b) radiation wavelength. ht ea Het em Tega dese Sea ar he nt 5 PRegae hse interel rm to. Hs hh twat Metalingus of spt elton dest Cts Natltgtekeh diver the oolage Tie che) an io tn tact the fol eaten wba aa Stu 007 ow dation spectrum into ‘at the temperature {e) how many tines the power radiated at waveeggs exceeding 0. ‘Bae ‘will increase if the temperature rises from 3000 to 5000 K. 5.259, Making use of Planck's formula, derive the expressions termining the numberof photons por Lem? oft eavity at 4 tempe: ature 7 ta the spectral intervals. (w, + da) and. Gy Kos chy ‘5.200, An isotropic point soures smite light” with wavelength ‘A= 580 nm. The radiation power of the source is P= 10 W. Find (@ the'mean density of the flow of photons ot a distance r= ‘20-1 from the source, (b) the distance Between the source and the point at which the ‘mean concentration of photons is equal to n= 400 cm: Bao the anda of the opera ther dinoncate that the momentum transferred by a Beau of parallel light rays per unit time does not depend on is spectal composition but de pends only'on the energy tux 202. “Alaser emite a Tight pulse of duration + = 0.43 ms and nergy B = 40 J. Find the mean pressure exerted by sich «light pulse wlten itis focussed Hato a spot of diameter d — 10 pm ‘oa 2 susface perpendicular to the beam and pissessing a relection Coaticient p= 0.50 5.263, A'short light pulse of energy B ~ 7.5 3 falls io the form ‘of @ narrow and elmost parallel beam on a mitzor plate whoa refoc: ‘ion cosfcient is p'~ 0.60. The angle of incidence i 30" Ia terms of the eorpastular thesey tod the momeatum rasfered to tae plat, ‘5.266, A plane light wave of intensity 7 = 0.20 Wicm? falls on 4 plane mirtor surface with reflection cosficient p = 0.8 The angie (of incidence is 45°, Ia terms of the carpuscular theory ind the mage tude of the normal prosure exerted by light on that sure ‘5.265. A plane Hight wavo of inteuelty 7 0.70 Wem llumic rates a sphere with idea! mirror surface. The radius of the sphere is H=5.0'em. From the standpoint of the corpuscular theory find ‘the fore thai light exert on the =p 5.206. Anisotropic point source of radiation power P is located on the axis of en Ideal mror plate, The distance between the souree fand the plate exceeds the raaius af the plate nfold. Ya terms of the ‘iepuseular theory tind the Torce that light exerts on the plate ‘9.267. In a reference frame Ka photon of frequency’ o falls norm- ally on a mirror approsching it with relativistic velocity. V. nd {he momentum npared to the isto dria th reo of photon {a) in the reference frame fixed to the mirror; (3) in the ram K. 5.268, A'suall ideal mieror of mass m = 40 mg is suspended by 4 Weightloss thread of Tength {10 cin. Find the angle through ‘which the thread will be deflected when’ » short laser pulse with nergy Bm 434 is shot in the horizontal dreetion at right angles to the mitror. Where does the witeor get ite kinetic enersy? 5.269. A photon of frequency ay is emitted from the surface of ‘star whee mass is Mand radive H. Find the pravitatoual shift, af frequency Sway ofthe photon ata very great distance from the S270, A voltage applied to an Xccwy tube being Increased 9) = TES times, the shortwave limit of an X-ray comnts sperm Sits by 22 "bi pm "Fd the ‘initial voltage applied to the tube ‘Ai74. A narrow X-ray beam fatls on a NaCl single erystal. The tetany at hich ir ree fn the ‘gstem of crystallographic planes Is still observed is equal to = SEE. interplanar distance fs d= 0.28 or. How high ls the ‘Fottage appli to the X-ray tube? ‘3972. Bnd the wavelngth of the short-wave limit of an X-ray continuous spectro I eectvas approach the antiathode of the {ube with velocity » — Ol here ey the velocity at light. afd. Find the phoualctie thesia for rine and the mxioum weleity of photoelectrons liberated fom Its surface by electromag etic radiation ‘with wavelength 250 nm, S.b7ts Illuminating the sutface of a eetain metal alternately ith light of wavelengthe dy = 0.5 pm and by = 0.54 my Twas ound that the coreponding maxinitm velcities of photoclettons iter by a factor 4 220. Find the work funetion ef thot meta, S75: Up to what maximum poeatil wil copper ball, remote from all otter bodies, be changed when iradited by electromagnetic radiation of wavelength 3 = 140 nm? ‘276. Find the matimum kine energy of photoelectrons Ubert- al irom the torace of Lithium by sletromgnete radiation whose STectre component varies with time as, =a {I “+ cus wf) cos «a Sere e na constant, a 60-10% 8-2 andy = S.0ei0% 5-7 ‘SVE. Blectromayuetic radiation of wavelength — 0.80 win fails on photocell operating in the saturetion mode. The correspond- ag epectal sensitivity ofthe photocell is J = 48 AW. Find the isd af photeloctroas Le. the number of photoaertnoas produced Bytench fneldeat pote S278, ‘Thor i's vacuum photocell whose one electrode is made cof ces anf’ the olher of copper. Find the maximum velocity of photoelectrons approaching the copper eletzede when the cesium Thetrodo ts subjetied to slectiomagoetic radiation of wavelength 5192 pm and the electrodes ave shorted outside the cll S279. N photoleciie current emerging inthe cirehit of a vie cuvin photo! hen ssi electrode subjected to electromagnetic dion of avelenth SES ne cance ia ete eclrt Ing voltage 1°V te applied. Find the magaitode and pol UE Sater Cniact poet dierent of the even pistol ‘3.280, Compose’ the expression for a qusatity whose dimension islengt, cing seloety of ight, mass oft particle m, and Planck's estat’ A What = tht quently? ‘S81; Using the conservation Taws, demozstrate thet a free electron cannot absorb « photon completely on 3.282, Explain the following featores of Compton seat Tight by matt Ya) the inerase in wavelength AA is independent ofthe nature of the scattering substance: ‘O) the intensity of the displaced component of scattered light grows withthe ineensing ante of seatterng and withthe diminish fg atomic womber uf the substance; fe} he proce of a nomasplored component in the seatered rodinton 5.258. narrow monochromatic X-ray Beam falls on a scattering substance. The wavelengths af radiation scattered at angles Oy = ODF fnd y= 120" dillr hy factor 20. Assuming the fre electeone {o be tesponsibe fr th scattering, find the incident radiation wave- Tength 3-284. 4 photon with energy fay = 1.00 MeV is scattered by a stationary {eee eect. Find the Kigetie enengy of a, Compton elect if tho photons wavelength changed by = 268 due to Scattering 5.265." photon of wavelength 4 = 6.0 pm ie sattered at right angies by a Stationary fre electron. Find (a) the frequency of {G) the kinetic energy ofthe S26" A "photon we angle = 480" by a stationary tee electron, Find the euergy of the Scattered photon 5.067. "photon with momentum p = 1.02 MeVic, where ¢ is tne’ velocity af hight is Senttored by a stationary free eleeton ing in the prices ite moment fo the value p” — 0.259 MeV, ‘At what angle fs the photon scattered? 5.288 3ptolon ss seattved at an angle 1 — 120° by stationary fe electron. Ae a renull, Ue electron acqutes a kinetic eneray T= sin MeV. Find the nergy tht the photon had prior £0 aca foci 589, Find the wavelength of X-ray taiation if the meximum Kinetic energy of Compton electrons 1s Tngq'= 0.19 Mev 5.290."A photon. with eneepy ‘hw = O'S MeV is seaitared by 4 statlonary fre clecton changing its wavelength by Ah = 3.0 pm Find the sngle at which the Compton lectron moves 5.201. A photon with enengy exceeding 20 tlines the rest nergy of an electron experienced a head-on collision with a ste. Alonazy free electron. Pind the curvature radios ofthe trajectory af the Compton electron ina magnetic held B = O42 - The Compton lect is assume to move at ght angles to the direction of the fe 3.202. Having collided with a relativistic electron, photon is Aefieted through an angle Bi" while the electron stops, Fad the Compton displacement of the wavelength of tho seateeed photon PART SIX, ATOMIC AND NUCLEAR PHYSICS * 64, SCATTERING OF panmieta. f.Azele © a which « chared ori le dfectes by tho Coulomb Sal ota fadadary toe necks Sebel y the fot: 7 &* mle 10) ny anda hg hes oft par andthe aul, i the ning sins tho Kiso esrgy oa te fe pole . f atherford formula The rlatve matte g bee Oi Ipttcles sated ate aleentary scluanfite ata anleD to thelr nal po aot direction > (4 a |r So (EY sae om | whee i th nner of cla the fil pe NV fat efit upace an 9 2 "Gesteliaed Baler fortla (PE 6.) a 4), n= 3%. 1a (op Pig. 6 BT the tain roan (ie 2“) Btwn energy leva with qua ‘eis my ad my Hs the Tydergcoarans Zi Ws weal nuubes of & ‘doen tis oa 64, Boyne Toss model cleat the rds af hyo sgn aiom and the waselength of emitted ight if Ue fotaation os Gf the atom is kaown tobe equal to. B= 15.6 eV vs 6.2. Au alpha pottle with Kintic energy 0.27 MeV ts deflected through an angle of 60" by-a golden fol Pind the eomerpondiog Wale of the aig poramete 8, To what mista distance will an alpha particle with incic enegy 700° MeV approach in the case fa head-on (@) 2 Hlatioary Pb mycloos: (0) Stathoary fees Li nucleus? 4.'A alpi particle with Kinetic euergy 7 — 0.50 MeV. is deleted through am angle"of 8 =" by the Coulomb held of 8 Sationery Ig cleus, Fads TT le or 59 tls Part ave poe In the Gaui estan of wits, (a) the least curvature radios of its trjetony {2 Senn aprach Astance Seton he pate ad the 5. A proton with kinetic energy 7 and siming parameter b was defecied by the Coulomb hold of Matioary Auroucleus Find the momentum impart to the given nucleus #8 result of scattering b.'A proton vith hintic enengy 7 Ale Bloe pat 8 se Lionary fe electron at distance 840 po Find the energy oquied By the electron, assuming the protons trajectory to be fectlinear andthe electron tobe practically motionless as the proton Tie 167."A particle with kinetic energy T's defectod by a spherial potential well of radius MT and depth Cy, ie. by the beld In which Eoo‘pteatil eaogy of he particle tals the fem on {2a ree -U, for FER, ‘where ris the distance from the centr of the well. Find the relation- ‘hip between the aiming parameter bof the particle end the aagle © theowgh whieh it deflects frou the initial motion ditection, GS. A'stationacy sll of radivs Mt is iadisted, bye. poralle] stream of parties whose radius is 7. Assuming the collision of ele andthe ball to be elastic. fn: (G) the deflection angle @ ofa paricle as a function ofits aiming parameter () the feaction of particles which after a collision with the bell aro scattered Into the angular interval between 9 and ©» dB; (©) the probability of a partiele to be defected, after a collision wih the bal into the fon hemisphere (0 <3). 6.9. A nerrow beam of alpha particles with kinetic energy 1.0 Me falls normally on a platinvo fo 1.0 yim thick. ‘The seattered par teles nce obeerved at i angle af (0° tn the incident beagy direction by means of « counter with a citealarlalet aren 1.0 eu" Toeated at te distance 10'em from the scattering section of the foil. What Traction of seattered alpha partiles Teaches the counter inlet? ‘6.10. A narrow beam of alpha particles with kinetic energy T= 0.30 MeV’ and dntensity’/ = 50-10% particles per second falls ‘ormally'on a golden foi. Find thethicknesé of the fol fat a distance Ponts eat froma" seaitering section of that Toil the Mux density of seaitered particles at the angie @ — G0" Lo the incident beam is ‘qual to J = 40 particles/ent=s). 11. narrow beam of alpha particles falls normally on a silver foil behind which « counter is et to register the. seattred partite. On subtitution of platinum foil of the same mass thickness for the silver fol, the sumaber of pha patticles registered per unit time increased n= 1.32 tines. Find the atomic number of platioum, ar tome number of silver and the atomic masses of bath Platguis aad’ slvr to be hagwn, 6.12. A narrow beam of alpha particles with kinetic energy T= = 6.56 MeV falls normally on a\golden fol whose mass thickne== B pd= 1.5 melem* The beam intensity Is 1, = 5.0-f06 particles per second. Find the number of alpha particles sented by” the foil {uring atime interval r'— 90 min indo the angular interval (a) SBM": (0) over By = 60" GUS. A narrow beam’ of protons with velocity.» — 0-10" m/s fails normally on a silver fol of thicknes= d”~ 0 pa, Find the Probability of the proton to be scattered into the tear hemiephere of alpha particles with kinetic enexgy = G00" Key" fale normally” on a" golden foil incorporating m 11-400 nuelesem® Find the fraction of alpha particles seattored ‘rough the sngles 0 <1 ~ 20" ‘6.15, A narrow beam of protone with kinetic energy T = 1.4. MeV falls normally on a bene oll whove mate thickness pl ~ 4.5. ‘mgiem ‘The welght rato af eopper and zine in the full 2 equal to 723 2e- spectively. Find the tetion of the protons scattered through Ure gles excooding By — 10 Find the ctv ers sexton of rant nels ur responding to the scattering of alpha particles with kinetic enon Tas MeV through the angles exceeding 9, z GA7."The effective cross section ol 4 kvld muclews correspondin to the sealtering of monoenergetic apa pucticles within the angular Interval fom 40 to IA Is eqval to A050 kb. Find (a) the euergy of alpha particles: (3) the difeential eros section of seatering daldO (kbist) com responding to the angle 00" ib. In accordance. with classical electzodynamics an electron roving with acenlerstion w loves is energy sve to ealinton eet ‘where ¢ is the electron charge, I the velocity of Hight. Estimate the time during: which the energy of an electron, pesforming. almost jarmonie oscillations with frequency «== 5-100 s~? will lecreace 0 times G19. Making use of the formula ofthe foregoing problem, estimate the a eieculae opbit of radius + fucleus. Pet the sake of simplicity assume the vector w tobe perma ently directed toward the contre of the etom, 6.20. Demonstrate that the fequency @ uf @ photon emerging when an electron jumps between neighbouring cievlat orbits of 2 hydrogentike fon satisfies the inequality tne) > ty oy. Where log and yy ate the frequencies of revolition af that elctton arovad 8 the nue lag be tarot Make ste hat a8» ve the frequency of the photon 1 ~ Ws 2ai0'S pasties of masa moves along a civsla obi i centro symmtriea poteolal fold U () Ar. Using the Bob qualia {Un condi Sad the permite bt adit and exes lees ot tat particle 6.38, Ciculete for a hydrogen atom and He fon (a) the dian ofthe st Hot exit end the velocity fon moving along it (Oy the kisetie energy and he binding energy of an eleetron in the ground sat (etn fotation potential the Gia exttion_poteatil and the wavelength of the resonance line (u" = 2 => “ABR Callie the solo. teqooney of electron occupying the send Bob abi a Hew ion For hydrogenlike systems ind the magnetic moment tn corapning to the motion of an electron along the m-th orbit tnd the ratio ‘of the magnetic and mechania! moments fi llevlote the magnetic moment of an elect occupying the Bek Dobe ost (5 Caicuate the magnetic field induetion at the centre of a frragen atom caused by an electron moving along the frst Bolt cai 6.26. Calculate and drow on the wavelength seale the spectral Infervals in watch the L3man, Balmer, and Paschen series for atomic Lydragen ate conned. Show the ‘visible portion of the spec: ‘G7. To what saies does the spectal lie of atomic hydrogen belong if ts wave Bomber In equal tothe difference belweea the wave uumbers of the following two lines of the Balmer series: 486.1 a {Hod nm? Wht ts the wavelengtr of chat Hine? 16.28. For the case of atomie hydrogen. Gnd {a) the wavelength of the fst thee Lines of the Balmer sviess {b) the minimum resolving power 18h of spectra fastrument capable of resolving the frst 30 lines ofthe Balter series EQ. Radiation of toute hydrogen als soem ons fraction eating of width —'6.6 mm. The 50th live of the Baliner series {the sberved spectrum i close to recoution ata dilaction angle 0 {in accordance with Revleigi's criterion). Find that angle. ‘630. What loment hos 4 lydrogunicke specteam whore ines hae wavelengths four Uaes aorter than these of storie hydrogen? ‘3. How many spectral liner ae emiltad by atomte hydrogen eseted to the mah enerey level? 6.3% What lines of stomie hydrogen ebsortion spectrum fal within the wavelength renge from 94 to 1900 nm? (G38. Find the quantum nuinber n corresponding to the excited scate of He* lon fon tranltion tothe ground sate that fo ets Wo" plitons in succession with wavelength 108.5 and. 0-4 nm. 6.34, Calculate the Ryaberg constant if He* ions afe known to liave the wavelength difference between the fist (of the Tongast wavelength) lines of the Balmer and Lyman teres equal to k= SiS om. 6.35. What bydrogen-ike ion has the wavelength diference be. ‘weenie ist ines uf the Balmerand Lyman series equ 0.50.3 nm? ‘6.36. Find the wavelength ofthe Nest Line of the He" lon specteat series “whoso interval between the extreme lines is a= Sa810 st 6.37. Find the binding enengy of sn electron in the ground state ‘of hydrogenlike ions in shot spectruuh the thind lin of the Baler Series 1s equal to 108.5 nm, ‘5S8. The binding energy ofan electro in the ground state of Ile stom is equal to By-= 240 eV. Find the energy required to remove Doth electrons fromthe atom. ‘6.30. Find the velocity of photoelectzons liberated by electsomag- attic radiation of wavelength A= 18:0-num foo stationaey He Jone ithe eroud eat. a ‘At what minimum kinetle energy must a hydrogen atom ove for is inelastic head-on collision with another, stationary, Hydrogen atom to wake one of them eapsble of emitting « photon? oth atoms are supposed to be inthe ground state pros tothe cll eit. A stationary hydrogen atom emite « photon corresponding the frst Tine of the Lyman series. Whot velocity does the som ltions ofthe fuegoing prublem find how mel (in por cent) the energy of the emitted photon difers rom the euengy of the correspouding tansition in a hydrogen. stow, 6.43." stationary He* fon emitted photon coreesponding to the first line of the Lynaa series. That photon liberated t potoelectton from s'stationary hytdrogen stom in the ground state. ind the velocity of the photoeletrun ‘6.44, Find the veloelty of the excite hydrogen atoms if the frst Tine of the Lyman serie is displaced by As —"0.20 tun when th radiation is observed at an angle.) ~ 45° to their motion direction. 1S. According to the Bobr Sommerfeld postilate the. periodic motion of particle in & potential el must sty the flowing ‘quantization vues § dem nto, where ¢ and pare generlized coordinate aod momentum ofthe article "nave intages. Making waa his rule, Gnd the pornitted ‘elues of eoengy for 8 porte of mete m atving in unidimensional rectangular potential well of width 1 veil Intel high walls 1) along’ sie of radius 15 0 (©) & vnldimenvione potato Sold 17 = a2, where a Is {ay along’ round orbit central Geld, where the poventst cowry of the particle equate U'= aie (2 a's poses ote St ZMK, Taking into szoun the tion ofthe nucleus of hydrogen ato had th eetsons ar the thetsons Bading energy ie oud tata anor the RyGbeny constant. How tah per cat) 0 the nding eoegy un the Hyubers constant obtained without taking into avant the motlon ofthe Solos afer athe te Sceurtecotapondig valuta of thee quetieet 6.47. For stor of fight and heavy hydrogen (HE and D) find the dufeeace {c)Yooen he binding eerie oftheir leszons inthe ground ae i} nine the wavelongte of Sat lines ofthe Lyman sere. tia Eason epteon etueen he patel of 5 ssh ta ihe grunt state, the orespnding Malog eneres. at the Satna Bet in of Ihe Lys p S uc gtem is ‘c) atiaatehyeogen tom whoo muse fa pon ait hom sn lcion swept by s meaon wlose charge i ie Sine snd ges 307 dat of on ec i) poston coming of a election and os arwind leit comon coir’ ames 62. WAVE PROPERTIES OF PARTICLES. SCIMODINGER EQUATION ‘+ The de Bropiewaveangth of «pate with momentum p oy + oceans pe aeare om Shiner Cnecependet and tine deen eunions aa At weer 29 wt GF eo y=0. are ig te as st, Gi the Laplace oor save function ator, nd ar he tts potenti emerges pile. tk Spee a et rane ws (805) are aye 29 + Cecttiot of uansporeny of» rotonta har Va pawol, | vRw=a «] oa har andy am tho coordinator of the points Betas which 7 > #: “al Caste the de Droge wayslengtb of an lection. ton, jum stom. all having the same kinotte energy 100 eV. hos What amour of snetzy should be added 10 gn electron to raduce its de Broglie a 51. A neutron with tionary deuteron (heavy hydrogen ucleus), Find the de Broglie ‘wavelengths of both partiles tn the frame af thelr contre of inertia, 8.52. Two. Kdentical non-relativistic particles move at. right angles to each other, posersing de Broglie wavelengths by and by Find the de. Broglie wavelength of each particle Im the Teame of ‘heir cantse of inertia. 6.58. Find the de Broglie wavelength of hydrogen molec which corresponds to their most probable velocity at room temp 8.54. Caletate the most probable de Broglie wavelenath of hydrogen molecules being in thermodysamie equilibrium at room tempersture ‘6.55. Derive the exprssion fora de Bross wavelength of «rela Livistte particle moving with Kinetic energy TAU what values of 7 does the error in determining 2 using the non-relativistic formula not exered 1% for an electron aod a proton? ‘8.36. AL what valve of kinetle oxerpy ls the de Brogtie wavelength of an electron equal to its Compton wavelength? 6537. Find the de Broglie wavelength of relativistic electrons reaching the antisthode of an. X-ray tube If the short wavelength Timitof the continuowe X-ray spectrum Is equal to hy = 10.0 pan? 6.58. A parallel stream of mooenergetic electrons falls normally ona diaphragm with navcow square slit of width = 10 qin Find the velocity of the electrons if the width ofthe cones) dif tion maximum formed on a sere located at a distance f from the ait is equal to Az — 0.35 mm. ‘G89. A parallel stream of electrons accelerated by a potential Aiferoace (25 V fale normally on a diaphragm with two narrow flit seperated by a distance d= SD ume Calealate the distance between neighbouring maxim of de dilfraction pattern on a screen Tocated ata distance 1 100 era from the site, 6.60. A harrow stream lol moncenergetic electrone falls st on angle of facenee 0 = 30" on the natural facet of an aluminium finle crystal. The distance between the neighbouring crystal planes Dacale to thot facet is equal tod = 0.20 um. The maximum mirror Feflection is observed at s certain accelerating voltage Vy. Find Vy oa {1 che next maximum mirzor reflection js known to be observed when the aceslerating voltage tu inereased 7) = 2.25 tines, G.61. A narrow beam of moncenergtie electrons falls normally fn the surface of « Nt single eeytal, The rellection ‘meximum of fourth order is observed in the divetion forming an angle 0 = 55° with tie aormal tothe surface st tho energy of the electrons egos! (oP = 480 eV. Caleulote the corresponding valve ofthe interplanar {stance 6.62. A narrow stream of electrons with kinetic energy T= =i keV passes throug « polycrystalline aluaiaium foil, forming ‘system of diffraction fringes on seen, Calelate the isterpia distance corresponding 1 the rellection of thind order from a certain System of erytal planes if tis responsible fora difracton riag of Ghameter D. 3.20 cm. The distance betwen the fil andthe sefeen 1000 om 6.63. A stream of electrons acelerated by a potential diference F fullnon the surface of etal whose Sonor potential fs Pp = 48° V Fine {a) the sefrecive index ofthe metal forthe electrons accelerate by 2 potential difference V == 150 V (tthe values of the ratio VV, at wich the refractive index difers from unity by not more then n= 10% 64. A particle of mats m Is Tocated ins unidimensional square potential wel with ifnitely high walls. The seth ofthe well Is Ejual to Find the permitted vslues of enrgy of the patil taking {nto aceount that coy. those states of the particle's motion re Zrallsd for which the whole number of de Broglie half waves are fitted within the given wel 8.65. Describe the Bohr quantum conditions iu terms of the wave theory: demonstrate that an elect in a hydrogen atom can move ‘nly along thoxe round orbits which accommodate a whole number Of de Broglie waves. Gib. Batinate the miainui errors in determining the velocity ofan eeeton, «protons ands ball of ws oft mg ithe coordinates ofthe particles and of the centre of the ball are known with uncer: tainly 1 6.87. Employing the vncertainty principle, evshusto the iadoter- aminancy of the velocity of an electron in a bpdrogen atom if the Shoo the atom caused tobe O40 a. Compare the obtained magnitude with the velocity of an cleetron in the Brst Boh orbit a the given stom. 6.68, Show that for the partile whose coordinate wncartainty whore. ts de Broglie wavelength the velocity uncertaia~ {5 of thesameordr of magaltude os the particle's voocity Hef, 6.69. A free electron was initially conined within « region with Aisear dimensions 1 = O40 nm. Using the uneertainty principle, ‘valuate the time over which the width of the coresponding of waves becomes 1 = 10 tues 35 large: 6.70, Employing the uncertainty principle, estimate the mini- He energy of an electron eanfined within a region whose 0.20 nt ‘An electron with kinetic energy T= 4 eV is confined within’ a region whose Tipear dimension is = 1 jum. Using tho Uncertainty principle, evaluate the volative uncertainty ofits velo- uty, 16.72. An electron is located ina unidimensional square potential ‘well with infinitely high walls. Tho width of the well is [From the uncertainty principle estimate the force with which the electron ossassing the mnininm permitted energy sets on the walls of the we “6.74, A particle of miss m moves in « unidimensional potential field 1 = Re#/2harmonie cvellator). Using the uncertainty pris- ciple, evaluate the minimum permitied energy of the Particle in that et 6.74, Making use of the uncertainty principle, evaluate the mi ‘mum permitted energy of an electron in» hydrogen stom and 1 corresponding appareat distance from the nucleus. 6.79. A parallel steam of hydrogen atoms with velocity » ~ G00" fs Tals normally on a aphasia with a narrow sit Behind which a screen is placed et a distence f —"1.0 m. Using the 0 equation for freely moving particle of met m. rik posi th got sa sed » unidimensional square pata ell f eg {wih sbsctely pert sale Gee Sip Bad the probity of tie atic soyog wa tiern $12 <4 6.78, A psile lost in unidiensionsl square potential sai ily igh wallet eis af he wel, Pd he Bormalled wavs fonefns of the stonay sass af ts pate TEL le atdpoin of te ell fr the urfe ef thes cosnate: 7H. Demons hat the wavefunctions ofthe saioery sates of pestle nhac ta sidionsnnal postal ntl ois tae Sirs hgh walle ste otogota, Le, ey ttn he coi J bear 0 ifn’ am. Here [is the width of the well, mare Integers. G.8D. An electron is Tocated in » unidimensional equare potential well with inGnitely high wall. ‘The width of the well equal to 1 is uch thatthe energy levels are very dense. Find the density of energy levels dN/aif, i.e. their number per unie energy interval, asa to of £. Calcuiste avid& for # = 1.0 eV if 1= 10 em: ‘GSI. A particle of mass m ie located in a two-dimensional equa potential well with absolutely impenetrable walls, Fin (a) the particle's permitted energy values ifthe sides of the well aro hand Ug (a the exergy values of the particle at the frst four levels if the swell has the cape of a square with side 1 1.82. A partite Is located In a twovdimensional square potential voli with tbsolutely impenetrable walls (. 1h if Uy sin bry hry VETDRPDay where km V DRE (4) Calculate the value of the quantity F207, at which the fest loval appears. 16.90. The wave function ofa particle of mass m in a unidimension- al potential eld U (2) = Ase!2 has in tho ground state the for (2) = deme, where A 9 normalization factor and is a positive Constant: Makidw use of the Seeodinger equation, tnd the constant 2 land the energy’ of the particle tn this sate. 6.91. Find the energy of an electron of « hydrogen stom in esta tionary stete for whieh the wave fuzetlon takes the fora '§ (0) ACI paren" where Ay ay and c are constants, 6.92, Tie wave function of an electron of hydrogen atom in the aground state takes the form 9p(°) = der", where is.a certain Sonstant, ry isthe fist Bole radius, Find: fa) the ost probable distance between the electron and the nucle 0) {c) the mean value of the potential energy of the electron in the field. of the nucleus, 6.93, Find the mean electrostatic potential produced by an electron in the contre of a hydrogen atom if the electon is in the fund slate for which the wave Tunction ia (r) = Ae-"™, where fsa certain constant, ry is the first Bole radius mean velue of modulus of the Coulomb for» acting on the ls of mace m and energy # move from the left to the je shown in Fig. 6-3. Pid: nt fof the barsioe for B > Uy (©) the etective penetration depth of the particles into the region £>0 for B-< Uy 10. the distance from the basrie boundary to ‘he point at which tho probability of fading « particle decreases ol | f y & T 7 6.95. Bmploying Bq, (6.20), ind the probability D of an election with caorgy”# tunnelling through s\potential Harrier of width thd height Uy provided the bareier is thaped a chown: (a) in Fig. bus (@) in Figs 6 lw I fg Lit = Pig 64 Pig 68 Fin 66, 6.96, Using Eq. (6.20), find the probability D of a particle of mass m and energy tunselling through the potential berier showin Fig. 6.0, where 0 (2) =" Uy (1 257, 1.3, PROPERTIES OF ATOMS, SPECTRA 4 Spel balling of erga: Miz, where x= 28-4 is tbe i cto RS te ean bers a eeaeees PHL EAH E (iS, P, DER, 6, Mb rast a Terme of skal meta stom 7 ean whet fe the Rydberg constant he yor correction, ha: @Miotitar Stiga cs ius how os. 0 "nner momenta of a5 atom My = AVEO, em) ith snide expressions for Mg aad My God rate Main electnle contention the tr of the Fi. 8. Fig. 88, eer ral a on hype! sea th stl eh ep fhe ope ema, eat robabiis of stomis ransitons ge pit tne butwee lve Zand high ieel'P rth cas of spontaneous riatan, neaeed venti, aad Bar " Pm bay, P= Baten P= Bh, ean ere Ain a Bayan instin cofcens a the apctal density of radi TOS"eiping to temeney af teoutoabtwees the gov Tove Relation batween lnvnn coutclente ps Ba ae Aa om coe Diagram showing formation of X-ray spcta (Fi. 6.8, 1 Montes nw fr Tne: ox, ER wan ‘+ bagrte erent of an atom and Lande ¢ te: VroFh mtg 2Gtoesisey—euen VITFB en et pt ea 4+ Zeeman aplittieg of mcr lng a «ak magnetic Sel 20m tse magd Hal. ean) 4+ .Mithraiationdieeted long the megeetic eld, the Zeeman compo cata bythe tain my = my Sr tat. = 6.97. The binding energy of valence electton ina Li atom in the sales 28 and 2P is equal to 9.30 and 3.94 eV respectively. Find the Rydborg coreetions for Sand 'P terms of the stom 1.88. Find the Rydberg corretion for the SP term af » Na atom whose first excitation potential is 2-10 V and whose valence electron Im the normal 38 state has the binding energy #14 eV 6.99. Pind the binding energy of a valence electton in the ground state of a Li stom if the wavelength of the frst line of the sharp feries is known to be equal to. 813 um and the short-wave cuLof! waveleagth of that series to 4, = 350 nm, 16.100. Determine the ‘wavelengths of spectral ines appearing toms from the state 38 down to the “The Rydberg corrections for the $ and P terms 0.0 401. Tho wavelengths of the yellow douse com, resonance Na Tine causod by the teansition SP—~ 589,00 and 580.58 nm. Find the spliting of the AP term in eV unite, 6.102. Tho fest line of the sharp series of storie esiva is a doube with wavelengtis 1958.8 ane 1408.5 nm. Flad tho frequency. {intervals {in rad/s units) between the components of the sequent Hines of that series. ‘6.103. Write the spectral designations ofthe terms of the hydrogen ‘atom, whose electron isin the state with principal quantuat bumber 6.104. How many and whien values of the quentum number J can an atom possess in the stato with quantwsn numbers Sand L equal respectively to (a) 2 aud 3; (b) 3 and 3; (@) 522 and 2 6.105. Find’ the possible valucs of total angular momenta of ‘tome in the states *P and *D. ‘6106. Find the greatect possible total angular momentum and the coresrgncig petra deigaton of the tem 1 (a) of a Na atom whgco velonce electron posses the prinetpal quantum ‘number n =f (@) of an stom with electronic configuration 12pi 6.107. Te is known that in F and D states the number of posible values of the quanti sumaber J isthe same end equal to five. Find the spin “angular momentus ia these states. tents of the ‘equal to 6.108. An atom isin the state whose multiplicity is three end the total angular momentum is RY/20. What can the corresponding (quantum number Z be equal to? ‘6.109. Find the” possile: multplici types {@) Ds: (0) "Pye (0) *Fe E410, cerain Son has thee elatrons (e, p, and d), ia adaition to filled shells, snd is ine state with tho greatest possible total mechabiesl moment for a given eondguratfon. In the corresponding Yector model of the stom find the angle between the spin momentum aot Singular momentum ofthe given atom. ‘GALL. An atom posessing the total angular momentum RY fs i the state with tpin quantum number S'= 1 In the corresponds ing vector model the angle hetween the spin manentum and the totel fagelar aenentan "0 735. Waite the seca syatl fr the term of that st ‘GAL, Write the spectral symbols for the terms of a two-electron cystam consisting of one p electron and one d electron. ‘113. A system comprises an atom In "Py state and ad electron. Find the possible special terms of that syciem. ‘GALA. Pind ont which of the following transitions arg forbidden by the election Tales! "Dys—> Py Py "Syn Pee Py Sys Duy G415, Determine the overall degeneracy of a SD state of a Li atom, What isthe phiysieal meaning of that value? ‘GAM6. Find the degeneracy of the states *P, D, and *P possessing ‘the greatest possible values ofthe (otal angular” momentum. Ib.A17. Write the spectral designation ofthe torm whose degeneracy i equal ty seven and the quantum numbers Land 8 are interrelated mbm 38. 6.118. What clement has the stom whose AL, and M shells and’ e-subshell are filed complately aad. 4p tubshell ts half-filled? ‘119. Using the Hund rates, fd the basi term ofthe atom whose pariially filled subsbell contains (2) three p electrons; (b) Tour p electrons 6it20. Using the Hund rule, find the total angular momentum of ihe atom fo the ground state whoce partially filed subshell, contains i) three d electron; (b) coven a electrons Eclat. Making use of the Hund rules, Gnd the number of electrons in the ouly pattislly filled subshel of the atom whose basic term is 28) Pu (0) Sse 6:122."Using the Hund ‘les, write the spectral symbol of the basic term of the atom whose only partially filled subshell, (a) Is filled by 113, and S = fy (G) ie flled by 7086, and § = 32 6.123, The only partially filled suibhell ofa certain atom contains three electrons, the basie term of tho atom having =. Using x of the terms of the as vi20," Atomic liv of eoncentration m — 36-10% om i at tne wavolength of te Kline in son (23) Te keown to e equal 0 spectral symbol of the ground state of (othe dierence in binding energies of K end L slectrons in 6.195. How many elements are there in_a row between those whose wavelengths of A lines are equal to.250 ond {79 pm? 6.196. Find the voltage applied to an. Xray tube with nickel Aanticathode if the wavelength dflerence between the Ay line and ihe shortowave cutofl of the continuoas Xray spectrum is equal to 84 pm. 6.1%. AL s certain voltage applied to an X-ray sabe with slum hiuin aaticathode the shortwave cut-off wavelength of the contin= os Kay seceum wat 0.50 na Will he, sto of he ‘Characteristic ‘spectrum whose excitation potential is equal to 41.56 KV" be also observed inthis ease? ‘6.138, When the voltoge applied to an X-ray tube increased from ¥, 540 kV" to V, = 20 WV, the. wavelength interval Between te Ka line and the shortwave cucof of the continuous” Xray spoctram increases by a factor n= 3.0. Pind the atomic number of the element of which the tube's anticathode ‘s made 6.138. bas aban spec the deren between the frequencies of ‘Koray A and Z-abuorption edges equal too 6850 nt F Laan 6.140, Caleulete the binding enerzy of a K electron in vanadium whose 2 absorption edge has th he 34 om tel Find the binding lesion im titanium if jort-wave cut-off Ay 28 pm. 6.142, Pind the Kinetic energy and the velocity of the photoelect- rons liberated by Ka radiation of sine from the K shel of tron whose ‘E' band abeorplion” edge wavelength te hg = 174 pon, 6.145, Gateulate the Lands g factor for stows {eh in S states: (b) in single states 8.144, Calculate the Landé factor for the following tera (9) SF ig 0) Dy (0) Pas @) "Pye 0) *Pp 6.145." Caleutete the magnetic moment of an stom (in Bohe magneton). (a) in F state; (0) in Dy y state {e) inthe'siate in which $ = 1, £ +2, and Landé factor ¢ = 413. 8.148. Determine ‘the spin angular momentam of an atom in the state Dy if the maximum value of the magnetic moment p ‘ection in that state is equal to four Bohr magnetons, 6.147. "An atom inthe state with quantum umbers L = 2, {is Jocated in a weak magnate Held: Find its magnetic moment the Teast possible ‘engle between the angular momentum aad the eld ditection is known o be equal to 3 6.148, ‘A'valonce electron in g sodium atom is in the state with principal qaantumn number» being the greatest possible What iste magnetic moment in that 449, An excited atom hat the electronic configuration 1!22p84 boing in the tate with the greats posible total angular momentum. Find the magnetic moment af the stom in that state, 6.150, Find the (otal angular momentum of an atom inthe state with S'= 312 and l= 21 its magnetic moment ls known (0 be ‘ual to aro. S-45ts Avcortain stom isin the state in which S— 2, the total angular somentum Af = 2A, and the moguetic moment is equal toPvzero, Write the spectral symbol ofthe coresponding ri52. An stom in the state *Pas located in the external magne Lice of indtaction B= 4-0 KG Tn tm of the vector model Rnd the angular precosion velocity of the total angular momentum of aston 8.158. An tom in the state *Pxis i located on the axis of 10, of radia r Seem earrying » cattent '=40 “A. The distance bee {ween the atom and the centre ofthe loop 1s equal to the redius of the intr. How great may be tho maxlaim force thatthe magnetic fei of Cat cartnt exes ou the ston? normal state i located at «distance jght ‘conductor carrying a current on the atom 6.55. dium atoms in the ground state 8 is pased through e transverse strongly inhomoqensous mao fe iba ‘of lengthy ~ 5.0 em as in the Stern-Cerlach experiment. ‘Te eum splitting is observed on a serwen Tocsted st distance 1, 13 em tom the magnet. The Kinetlc energy of the atoms is 7 — 22 MeV. At what value of the gradfont of the masnotie Feld Induction Bis the distance betwen the extreme components of the split beam on the sereen equal to 8 = 20 mm? 58. Into what. number of eblvels are tho following terme split ina weal magnetie ei (8) 3Pa (8) Pore (0) De? 8.157."An atom ‘is located 'in a magnetic eld of induction 2 "2550 KG Find the value of the total spitting of the following erm, expresed ino ani): (@) 1D. 1b) 8.158, What Kind of Zeeman effet, normal or anomalous, is observed in'a weak magnetic fd inthe case of spectral ines eased bythe following transitions Me) Poe tS; OY Diarra (0) Dye *Pas (0) Myr He 59. Determine the spectral Symbol of an atomic singly tormn ‘the total spitting f thet term in a weak magnetic Geld of inde: tion B= 3.0 kG amounts to AB 104 peV 16.160. 10 is known tate spectral Tine’? = stom Js caused by's transition between singlet terms. Calculate the inter- ‘Yal Ah batween the extrnme components of that line in the magnetic ‘0 ko, eld with induction 2 29 6.161. Find the minimum magniide ofthe magnetic Feld indus: tion Bat which a spectral Instrument with tsoving power 6k = 4.0-40""is capable of resolving the components of the special Tine = 536 nm caused ya transition between singlet tem. The ‘obeervation line is at right angles to the magnetic eld direction. ‘62. A spectral Tie caused by the transition sD, *Py expe riences the Zeeman splitting ina wesk magnetic fehd. When observed at rig angles to the magnetic fed diretion, the Interval Between the nelzhbouring components of the split line is ho = 1.82408 5 Find the magostie Bald induction 2 at the plat where the source fr 'Toeate. 85165. The wavalongths of the Na yllow doublet (P —-*S) are qual 16 589.59 and 389.00 nm. Find (sth ratio of the intervals betoronn neighbouring sublevee of the Zeeman splitting of the terms #Py and "Py ih a weak tagnelie i (b) the magnetic feld induction B at which the interval between neighbouring sublevela of the Zeeman splitting of the term “Pe, fq 5D times smaller than the nataral splitting of the term 12. 6.164. Deas a diagram of permitted transitions betwoon the terms *Py)sand 8, Ina'weak magnetic Held. Find the displace” ‘ments (in rad/sunits) of Zeeman components of that line in'a magnetic held B ~ 4.5 WG. direction 2. How many Zeeman components are observed in both Airetions it the spestal line fa aud by the transition () *Payg > BSuyh (b) *Pame 254? 6:t66,"Galeuate the total splitting Ao of the spectral Line 2D,» + 3Py in a weak magnetic held with induction B= 3.4 kG. + Rotation eoegy af # diatomic molecule Brush an here Js the slcle's moment of inti fo rational oergy of «doom molecule: 1m (e+) en ‘whore w i the otural frequency of exlations of the molecule, 4 Mean eowtgy of quantam harmonic ouilaar at a tempera 7: ont oe Debye female for waar viens engy of» et waar lees) | ae oa tere @ she Detseampeam, = tomas 9 Wola irons heat ego a a ep or #0 co aon (2) oat) 4 Disstbvten oft lsc mal th ely ofthe abate an Vat year, 4) rs 4p eth concntation of eatrans, whose anngy fle within the lata SATE, EEE hetncegy 2 i counted of the htt of he conduton bad 2 Porm nel at T= Os Era Bt one, (aby shore i the concentration of fee eatrone in mete 167. Determine the angular rotation velocity of an S molecsle pronoted to the st excited rotational level it te distance betwee Aa auch! i d= 189 pn {08. For an HCI molecule find the rotational quantum numbers ot two delghbouring levels whase energies der SY 1.98 pe Tho ‘clot of the molecule are soparatad hy the distance of (27.5 pm. loo. Find the angels newentum ofan oxygen molecule woes rotational eneagy is E = 246 moV and the distance between the tucked = Bt pm, 70. Show thatthe frequency Intervals between the neighbour {ng spectral lines fa true rotational spectrum of diatomic molocslo ft equal Find the moment of inert and the distance between the duce! of a CH molecule the fotervals between the neighbouring Tines ofthe tr'rtational spectrum of these molec are ul to Bo 5arJOo st Gi, For an HP moleele nd the wumber of rotatioel vets located between the aeoth and frat excited voratonal levels ssme ing rotational states to be indopendent of vibrational ones. Ths atutnl vibration egutney ofthis ‘moleeale i equal to Fiavstou ails, and the distace between the nuelel is 94.7 pm 6.172, Sealant how many ins there ois tre sotatonl spotram of CO molecales whose natural bration frequency eee 0) 408 Fond moment of inertia T1440" em 6.173. Find the number of rotational lvels per unit enegy inte. val, GNIdB, for a distomte molecule ass function of rotational nergy £Caleuate that magnitude for an iodine motectle in the stave with rotational quantum number J 10. The distance between the eles of that molecule f equal to 267 pr 6.174, Pind the Taio of enespies required to excite » diatomic rsicala to the frst wirational and tothe fst sotatonal le Gaicaate that ratio forthe following molecules Mowatt 61041 pm am 33 mW ou 63s se ol oo = Here w is the natural vibration frequency of males distance between nucle. 6.175, Tho natural vibration frequency of « hydrogen molecule Igeequal to 8.25-40%" 71 the distance between the nucel ls 74 pa. Find the ratio of the number of there moloeales at the Bret exestod Vibrational level (o'= 1} to the number of molecules st the hest excited rotational level (J — 4) at a temperature T= 875 K. Tt ‘Should be remembered that the degeneracy of rotational levels is qual to 37 ‘6.176, Derive Eq, (64o), making use of the Bolt tion. From Pq. (@-4e) oblain the expression for mol heat capacity Uy ey of dlatomle gas, Caleulate Cy yy {OF Cly gas at the temperate $00 K. The natural vibration frequency of these ‘molecules ts equal to 1.064" 10% a=, B.177. Tn the middie of the totation vibration band of emission spectrum of HCI molecule, where the "seroth” line is forbidden by the selection rules, the interval between neighbouring lines is Sw = 2070-10 go8.” Caledlate the distance between the noclei of an Het molecule 6.178. Calculate the wavelengths af the red and wiolet satolites, closest tothe fixed line, in the vibration spectrom of Raman seater Ing by Fy molecules ifthe incident Tipit wavelength i equal 10 hy 404 Tim and the natural vibration frequency of the molecule fs = 215-10 got 6.179. Find the natural vibration froqueney and the quasielstic force coefficient of an Sy molecule ifthe wavelengths of the red ond Viplet satellites, claret to the hxed line, in the vibration spectrum ‘of Raman scattering are equal to 348.0 tnd $90.0 nm 6.180. Pind the ratio of Intensities ofthe violet and red satellites, closest to the Bxed line, the vibration spectrum of Raman scatter. ing by Cl, molecules ata temperature 7 — 300 K if the natural ais the bration frequency of these molecules is w = 1.08-408 =! By what factor will ths ratio change ifthe temperatare is doubled? i8t Consider the pose vibration moder inte follwing (a) CO, (0—C—O}, (0) GH, (H—C~C~H, 6.182, ind the number of natiral traverse vibrations of a string of length inthe frequeuey interval fom to 0 ++ da ifthe propa: gation velocity of vibrations is equal to. All vibrations are supposed {o'ocear in one. plane, 16-183. There ist square membrane of area S. Find the mumber of natural vibrations perpendicular to it plane inthe frequency intereal From w to w + da if the propagation voloelty of vibrations is exal 6.184, Find the numberof natura tranaverse vibrations ofa right: angied parallelepiped of volume Vin the frequency interval fom {0 fo w a sf the propagation velocity of vibrations ts equal {0 6.185. Assuming’ the propagation ‘velocities of longitidinal and transverse vibrations to be the same and equsl t9 ©, hind the Debye temperature (@) for a unidimensional crystal, ic. a chain of identical atoms Incorporating’ my atoms per ualt length (0) Tor 8 Oeovdimensionsl crystal, 8. plane squate grid consis. ing af identical atoms, containing ny atom per ent are {o) for'a simple cuble lattice consisting cf Identical atoms, con- taining my atoms por unit volume 6.186. Calculate tho Debye temperature for iron in which the propagation velocities of longitudinal and \ransverse vibrations ate qual to 5.85 and 3.28 kms respoctivly ‘6.187. Evaluate the propagation velocity of acoustic vibrations in aluminium whose Debye temperature is 8 = 398 K. 6.188, Derive. the formula expressing molar heat capacity of 4 unidimensional crystal a chain of identical stoms, as a funetion at temperature Tif the Dobie temperature of the chats is equal to 8. Simplify the obtained expression for the ease > 8. 6.189. In'a chain of Ientical atoms the vibration frequeney depends on ave number fas «tinge sin Ga'2), Whee Oy, is the maximum vibration frequency, k="3a/h. is the wave numbe corresponding to fraqueney o, 0 is the distance botween neighbour Ing atoms, Making use of this dispersion relation, find the dependence of the number of longitudinal vibrations per unit Trequeney interval, on o, Le. dV ido, if the length of tho chain is 1. Having obtained Nida, find the total number ¥ of possible longitudinal vibrations of the chain. 10, Cacaate the arena gungy pet one gram of copper whose Debye temperature ls 8 = 330 K 6.191, Fig. 6:10 shows heat capacity of a crystal vs temperature in terms of the Debye theory. Here C, is claslcal heat capacity, Is the Debye temperature. Using this pot, find Pe (4) the Debye temperature for silver if at temperature 7 = 68K lus molar beat capacity 1s equal to 45 Jifmol- ke): (@).the molar best capacity of alumintam at T= 80K if at Pm 950 Kit is equal t0:22-4 J/(a0l-K} () the maximum vibration frequency for copper whose. heat capacity st 7-425 K difers from the classical "valde. by 25%: a on (aetna Oriana ae Oa a. 60. 6.192, Demonstrate that molar heat capacity of «crystal at a temperature T<:8, where 6 is the Debye temperature, Is dened by Ea. (0.40, 16.193, Can ono consider the tomperatures 20 and 30 K as low for 4 crystal whose heat capacitin at thear temperatures are equal {0° 01298 and 0.700 J/(ol-K)? 6.194, Calculate the ean zero-point energy per one oseillator of & crystal in terms of the Debye theory If the Debye temperature Gf the crystal Is equal to 6. 6.195. Draw the vibration energy of a crystal as x fonction of 2y (neglecting the zero-point vibeations}. Consldor two ease: ‘of ¢ phonon (acoustle quantum) in copper whose Debye tomperatu Is equal to 390 K (97. Employing Eg. (04g), find at T= 0. (@) the maximum kinetic energy of free electroms in a their concentration is equal to m: () the mean Kinetic nergy of freo electrons if thelr maximum netic energy Tones is known 16.198, Waat fraction (In per cont of free electrons in a metal at TO has a Kinetic eonrgy exeooding hall the maximum energy? 6.190. Find the number of free slectrone per one sodium atom at T= Oif the Fermi level is equal to Ey = 807 eV and the density Of sodium fs 0.97 glem® tal 200. Up to what temperature has on to heat clasicl alectronic ‘gat (o make the mean energy ofits electrons equal to that of free Electrons in copper at TG? Only one free electron is supposed 9 ‘oreepond to eazh eopper ato ‘BI, Calculate the interval (ia eV units) between neighbouring tovele of free slectrone in a matal at T= 0 peat the Ferm level if the concentration of fve electrons, n= 20-10" em and the lume of the metal ig V1. em ‘8.202, Making use of Bq. (64g), find at 7 — 0: (a) the welocty dation af ten elsteons {B) tho rato of the mean velocity of tee electrons to heir maxt- mim woos "B03. On te basis of Ea. (6d) find the number offre electrons ing molalat T = 0 as function of de Broglie waveleagts, 6.204 Caloulate the electroaie gas prestur [a totale sodium, nF. in mich te concatatign of ese im 2 Zs:40% em. Uso the equation for the pressure of ideal gs re of» eatde i slecronie (ibe by" AP = tb K from the valve 7 = 2000 K revals in the inerease of saturation current by = 1.456. Find the work function of Slectron for the material of the cathode, ‘6.200, Find the refractive index of metallic sodium for electrons swith Katie energy T= 195 eV. Only ove free electron ts assUmed Te correspond to: tach sodium atom. 6207, Pind the minimum energy of electron-hole pale formation in an Impurty-iee semiconductor whose. electric conductance creases n= 50 times when tho temperature ineeases from T= ‘300K to Ty = 400K. 6.208. At voty low temperatures the photoelectric threshold short wavelength in an impuritytree germanium fsequal toa = 1.7 ue Find the temperature coffeont of toatanca of this gormaniam ample at room temperate ‘boo. Fig bt strates logarithmde slots conductance as 1 Tanetion “of rweiprocal_tempernture (7 tu KK units) for some hs ’ ‘ n-type semiconductor. Using this plot, find the width of the forbid ‘len band of the semieondvctor and the activation energy of donor level ‘E210. The reslstivity of an impurty-tre semiconductor at room temperature is p = 80 Geen Tt becomes equal to py ~ 40 G-em ‘when the semiconductor is ilumingted with light, and ¢= 8 mo filer switching off the light source the resistivity becomes equal to 45 Q-em, Find the mean Iifeime of eoudution electrons and ry Mas O.211, In Hall effect measurements a plate of width k ~ 10 mm anit length {= 0) mm made of type omicandctar was placed {n'y magnetic Bold with tndvetiow B.C kG. A potential Wer tance P10 V was applied across the edges of the plate. Tn this ‘ase the Hall eld is Vy = 50 mV and resistivity p= 2.5 Q-em Find the eoncentration of holes and hole mobility. G22. In" Hall tect measurements ins magnetic field with induction B= 5:0 KO the tranavrse lace Bld stongth ian itspurtytne germanioas turned out to be = 40 tes ee than the longitedinl electro Gold strongth, Flad The dlference ia the Inopiiies of eoadction elestrooad holes in the given semicon theta. ‘5.213. The Hall efect turned out to be not observable in 9 sem contluctor whose conduction electron mobility was. = 2.0 times that af the hole mobility. Pind tho ratfo of hole and conduction electron concentrations in that semiconductor. + Fandaneotl law of radiousive deny xy cantant the aan tine 6 Agno btwn th 1 Specie activity isthe stvny of anit mas 6f a radioisotope 6.214, Knowing the decay constant } of @ nuclous, find (a) the probability of decay of the aueleus during the time from 0 (0) the mean lifetime + of the nucleus 6.215, What fraction of the radionctive cobalt nucle whose half- life's 71.3 days decays dung a mouth? 16.216. How many betacpartiles are emitted during ane hour by 41.0'ug of Net radionuellde whose hall-ife ls 15 hours? BIT. To invectigate the beta-decay of Mg radionuclide, a coun ter was activated st the moment #0, Tt togistened Ny beta: parti los by a moment {,~2.05, and by @ moment f, = 9f, the number of registered beta-particles was 2.66 times greater. Find the mean Titetime of the given nucle 6.218, Tho activity of a certain proparation decreases 2.5 times after 7. days. Pind its halt, ‘8.219. At the inital moment the activity of «cerain radionuclide totalled 650 particles per minute. What will be the activity of the preparation afer half ts halflife period? 20, find the decay constant and the mean lifetime of CoM sadionuclide if ite scivity is Known to deiterse 40% per hour, ‘The decay product is nonradtosctive. 201. A"US" preparation of mass 1.0 ¢ emits 1.2640 alpha partici or son. Pi he at of hs lie andthe ary BF te preparation 6.222. Dotermine the age of ancient wooden items if i is known thatthe specie activity of C™ nuclide in them amounts to 319 of Uhat in lately feed tree. The halle of" nucle is 3520 years. 1.233, Ia ranlum ore the ratio of US nucle ta PD™ sacle int) = 2.8: Evaluate the age of the ov, asvuming all th Tead P= to be a final decay product of the uranium series, The half-life of Ur adele is 45-108 years 6.224 Caleulate the spose ativitiog of Na* and U** nuclides wince halfies ane 15 hor and’ 71-40" year rcpetivay 16225. A small smount of solution contsining Nae radonuclide with activity a= 2040" dlinteprations per second as injected In the boodstram of man. The activity of em" of blood semple {hen ? —'5.0 hours leer taraed ost to be." ~ odisiatagrations por aninate per em The hal-tifeof the radionuclide is f= 45 hours, Find’ the volume of the man's bod 16.226. The secite tivity of preparation consisting of rado- active Go! and ‘ngnradioactive Ca¥ is equal to 22-10% disse, ‘The halflife of Coie 70.3 days. Find the ratio ofthe mass of rao ‘active cobalt in that proparaton to tho total mast of th preparation (Gn. per cent ‘6327.8 certain preparation includes two beta-active components with dierent half Bien The means2ementsromlited inthe following Aependence of the natural logarithm of preparation activity on time # exprewed. tn hours to tk 8 8 tw ina G40 300 310 200 2.08 4m 1190 1182 0. Find the half-ifes of both components and the ratio of radioactive ‘clef of these components at tie moment t= 0. 6.228. AD radionuclide with halflife 7 ~ 143 days is produced Jn 8 ron at constant tog = 27-40 nuclpa acon, How Son after the Beginning of prosuetion of that radionuclide wil its setcity be equal tora 0-10" dls? 6.22), A radionuclide A, with desay constant hy transforms into 1 Tadionvelide ay with decay constant hy Assuming that atthe initial moment the proparation contained only the radionuclide Ay, squation describing accumulation of the radionuclide 44 (the time interval after which the activity of radionuclide 4 roaches the maxtouum value 16.290, Solve the foregoing problem it iy = by = i 2B. A radionuclide , goos through the (aasformation chain Arr dards (stable) with respective decay constants %y and ‘Atsaming that st the initial moment the preparation contained ‘only the radionuclide Ay equal In quantlty t0 My nuelel, And the ‘squation describing accumulation of the stable isotope Ay. 1.282. A'BIM radionuclide decays via the chain itp Pom Sr PM (blo, where the decay sonstants are fy = 4.80-40-° sty hy = 5:80:40" s Caleulate-alpha- and beteacilvties of the Bi reparation of mass 1.00 mg ¢ month afler its manufacture ‘253. (a) What tsotope is produced from the alphe-radioactive ‘Rat asa rosult of five alpha-disintegations and four P-disintegra- () How many alphas and P--doeays dow U* experience before tuming finally into the stable Pb" isotope? ‘2M. A stationary PLO nucleus emits an alphe-particle with kinotic energy Tq — 5:77 MoV. Find the recoil volocit of» daught- ‘nucleus. Wit fraction ofthe total energy liberated in this dacay SU Mev and practically all daugiter nuclet are formed directly i the gronnd stat Tue alphe-decay of Po!™ nucle (in the ground stats) is accompanied by emission of two groups of aiphacpartcles with intie energies 5.30 and 4.50 NV. Following the emission of these particle the daughter suclei are fowad ia the ground and excited Fates, Find the energy of gamma-quanta emitted by the excited asl 257, The mean path longth of alphe-paticls in ale under standard conditions (dened by te formula = 098-40" oom, ‘hore ry (emis) i the initial velocity of an siphapastcle. Using {Rts foria, for an alpharpardele With nil Kets energy 10 Mev" {5 ioseorageeutber of lon pales formed by th r ‘ho average amber of fon pales formed by the given alp fe whola path A aswell as overt fs hall suming {ie ion pair lormation energy to be equal to 34 eV ed in Band -dseays and in he mae te parent atom My he dougie tom 299. Taking the valves of atomic masses from the tables, find ‘i mania hie egy etal eid by: Be hele andthe corresponding kinetic energy of reclling daughter ‘cle formed directly in the ground state. Sie 240, Exeluste the amount of hat produced during a dey by active Na* preparation of mass m "40 mg. The bela-patices fey men nua il amy aaah the ightst possible energy af the given decay. The bali of Ne* ip Pois hous, * 6.241, Taking tho values of etme masses fom the tables, calex late the kinotte energies of «positron anda neutrino emitted by CH cleat fr theca "when the" daughter mule” not ‘6.242. Find the kinetic energy of the recoil aueleus in the positron- fe decay ofa nusiu forth cso when the emrgy 9 oatons ots, om the tbls of atone masse determine the nucleus appearing a rent of K-suptare ina Be" atom provided the daughter milous tans out to bein the ground state, 6.24 Posing own the, nnd snared Ap" nude ‘mit either gamma quanta. with energy 87 keV or K conversion ng ergy Is 26 keV. Find the velocity ofthese .215.'A tee slationary Ie! nucleus with excitation eve 128 eas tothe ground sate eming 9 gona ae tum, Catelate the fractional change of gamma quanta enerey do to recoil of the nucleus ss ot ome o 6-246. What mast bo the eelative velocity of « source aud sn absorber consisting of fe Te nucle’ to observe the imax imum absorp tion of gemma quanta with euergy em 13) keV? 1.247.°A source of gamma qvants is placa at height h ~ 20 m above an absorber With whet veloety should the sours be daplaced ‘pero unabalance compatly the graviaton!yrtion af amma quanta energy dve to the Earths ravity at the pint Sehr the abuorber is otated> pease 6.268. What isthe minim boight to which gamme qoanta soute Contaning excite Za naclel has to be cased for the ravi {ational displacement ofthe Mausbaver ine te exceed the linn With {tel when repstred on the Earth's surace? The rristered pamina guania"are Known to hove an energy ¢~ 98 kev'and appear on {fansition of Zn nucet tothe gourd stat, andthe meat heting St the excited state is t= 14 ps locity 1.6. NUCLEAR REACTIONS. Binding energy of + nucle: Epa tng + 2) wo sehece 2 eth chang of the sucess swat fe, As the mast number mye Iter and MT ave the masts of hydrogoo atom, a neuton, and an oer cores Siding tp the given nucleon Prodaleoltlns the falowlog formula is more convenient to we: Bom Thy tA Bide omy here day Soe Aad a the maa aurpuse of a hydrogen atom, neutron, “os bw conmepning tothe given muss ‘P Bhony diagrams of a auclenr reastion mb Me Moen EO (ee) laitostratd In Pig. 612, ware mf M and 4M re the ume of rs asi ot artes Eno, and fterthu restion, F and Pare the total hietc ener Geb patie bore ma fer the te Fs et the conte of inert ie Molin, enc th ronal SnG'E a the lading ocr te pot TELE tnd nto miner, 1 Wash aio eonegy levels of thn taal i. a ‘Tirmahold (jaime) kinetic soony of aiming pst Mme a Sodoe- (de hcler eon tan 2G e188) Fig 612 — ob, Mal Taal dnty 28 wale Hd eae mts I ean of gee eee Pear Set atzly ith an nial ttoary Hee ean: a yi asek er a tt Coo ee eat NER palate angle thvugh which a denteron —LULUrUrL—— ray areas) ‘Assuming the radius of a nucleus to be equal to R (= 0.13 1/4 pm, where A is its mass number, evaluate the density 7 SSLY as ince aceon patent wolime of the mls Lr —™”_— Bre Tb, () O84, x {e) Na® (2) Ne (a) 2 (ee A 6.254, Demonstrate that the binding energy of nucleus with ‘mass number A and charge Z can be found from Eq. (6.60). 6.350. Find the binding enorey of a ucleus eansisting of eyual numbers of protons and neutrous and having the radius one and a ball times smaller than that of Al? nucleus, 6.256. Meking use of the tables of atomic mases, find (@) the mean binding nergy ‘per one aueleon in 0" nucleus; (@) the Binding energy of neutzon and aa elpha-particle in «Be Sucteu (6) tho enoggy required for sxparatien of an O! nucleus into four {dentical particles 6.257. Find the diference in binding energies of a neutron and 1 proton in 1 BE nucleus. Pxplasn why there fs the dilrence ‘8.258, Find the energy required or separation of a Ne nucleus {nto two alphe-particles aud a CH nucleus if iis kagwn that the Dinding energies per one nucleon in Net, Het, and C™ nuclei are equal to 808, 7.07, and 7.68 MeV reepectively. 258. Calculate th atomic mass vats the mats of 2 Ei atom who le as he Dindig energy 413. MeV: (@) 4. nucleus whose binding energy per mucleen ia equal to 6.04 Mev, ‘6.280. The nuclei involved in the nuclear reection Ay -f Ay > ry +. A, have the binding energies By, By, By and Ey: Pind tho nergy of this reaction G.Bb1. Assuring that the split cvergy of 200 Bev’ Sn {a) the energy liberated in the fission of one kilogram of US feotape, and the mass of coal with calorie valve of 30 kdig ®beh Is equtvalent to that for one ky of U= G) the mas of U"fotope split during the explosion ofthe atomle bomb ‘with 30 kt troty! equivalent ifthe ealoriBe vali of trot] iat Wig. 16.252. What amount of hest is liberated during the formation of ‘one gram of Het trom douteriv He? What mass of coal with calo- ile value of 30 Ki/g is thermally equivalent to the magnitude ‘obtained ‘6.283. Taking the values of atomic masses from the tables calen- Into the energy per ncleon which Ss liberated in the nclen reaction it "HY S07Hfee Compare the obtained magaltude with the energy ‘per nucleon Iiherated inthe Bssion of U2® nucleve ‘6.264, Pad the energy of the reaction LI’ +p —+2Het if the binding energies per aucloon in Li" and He wuclel are known to be ual fo'5.60 and 7.06 MeV. respectively. "6.265, Pind the energy of the rection N* (a, p) O°" i the kinetic energy of the incoming alpha-particle is Ty = 40 MeV ing of « U" nuclens Iborates the je Rs na eee ee By Bg "6.212. What kinetic energy must » proton possess to split a deuter con il! whom binding energy te Ey —2.2 MeV? "6.273. The irradiation of Hh{om and Derylium targots by @ ‘monoersic stream of protons reveals that the reaction Lp, mBe PPf6s MeV is initiated whereas the resetion Bet(p, m)BP "1.85 MoV oes not take place. Find the possible values of kinetic energy of the protons. ‘6.274. To activate the ration (n, a) with stati neurons must have the threshold kinetie energy To, Find the energy of this reaction, "8.275. Caloulate the threshold kinetic eaorges of protons required to detivate the reactions (p, ») and (p, 3) with Li? nucel 6.276. Using the tabular values of atomic masses, find the thre old’ kinetic energy of an alpha particle required’ to activate the ticlar revlon La (asm) 8%. What fe the weocty of the B ‘A ‘neutron, with kinetic energy T= 40 MeV activates faction CH (x, 2) Bs! whose threshold Is Ty, = 6-17 MeV Find the Kinetic energy of the alpha-partiles outgoing at right angles to the incoming neutrons’ direction m0 ry BM nucle, ‘4.0 MoV 6.278, How much, in percent, does the threshold enenry of gam smaquantuan exceed the binding energy of denteon (Ey = 2.2 MeV) inthe eetelion yt HE nore BP 27D. A proton with Kinetic energy T= 1.5 NeV ts captured bya deuteron Hf: Find the excitation energy of the formed nucle, ‘bg. The yield ofthe musa resction CAC, n) Xt fas mas ina ‘magnitudes athe following values of Kinetic energy 7, of bombard Ing deuterons: 0.60, 0.90 1.55, and 1.80 MeV. Moking use of the iatte‘f atomic maces, ad the comesponding energy loves of the {fansitional nucleus through, which this reaction proved "A Stow ea heal nian ergata 9 = “30 dimes after passiag thebugh & cadmium plate of thickness FLA. ai'mnmn. Determine the elective crass-section of interaction ‘those eutrone with cadmium nucle 6.282, Determine how many ties the intensity of» narrow beam of thermal neutrons ill decfese afer passing through tho beet ‘rater layer of thieknvos d'= 8.0 cm, The effective eros-ections of TMersctlon of deuterium and oxygen, auelet with thermal neutrons fe equal 9 0, = 70 band oy = 42 respectively ‘628 nariow beam of therinal neutrons passes Uhcough « plate of ion whose absorption aud scattering elective eros-sctions are gust to'g = 2.5 Band oy ~ ib reapoctively. Pd the fexction Piteuteon’ quitting the Seem due to tatering if the thickness of the plate is'd = 0.90 om. ‘B84 The. yield of « nuclear reaction produlng radionvclies say be deteribed in two waye either by the ratio W ofthe number Siuelear reactions to the aumber of bombarding particles, of by fhe quantity by the satio of the activity of the formed radionuclide forthe aumber of bombording pets. Fla (G) the halt of the formed radintlie, assuming w and tobe known (the yield w of the reaction L(y, »)Bet if after irradiation of a lithium target by a beam of protons foyer = 2.0 hours and with Team current = 10 4A) the Setivity of Be" became equal to A "35-40" diss -and its balllite to = 5 days. 6.285. Thormal neutrons fall normally on the surface of a thin gold fol consisting of stable AuM* nucle. The neutron Hux density By S40 partuterem. The mass ofthe foil im = 10 mg Tha noutton capture produces beteactive AU™ nuclei with halflife P='3. days, "Th ‘fective eapture erowsection is 0 88 b sr which the nomber of Au! nuclet (G) the irradiation time aft decroases by n= 1.0%: (0) the maximam number of AuS® muclol that can be formed dur- ng protracted Iradiation, BabA thin fol of eran stable iston ated by thermal ‘neutrons falling normally on is surface.” Dus to the capture of hutrons a radionuclide with decay constant A appears. Fine the law m Akeseribing accumulation of that radionuclide ¥ (?) pet unit ares of the foils surface. The noutron ux density Is J) the number of ‘nuclei per unit area of the fail's surface ism, and the effective cross fection of formation of activo nuclei is 5.287. Avgold foil of mass m = 0.20 g was irradiated during 4 = "60 hours by a thermal neutron Aux falling normally ou its surlace, Rellowing + = 12 hours aftr the completion af irsdiation the activity of the foil became equal to A= 19-10" isl. Find the neutron flax density if the ellctive cross wotion of formation of a radioactive nucleus iso" 96 , and the halite is equal fo aT days 16.288, How many neutrons ors thee in the hundredth generation A ths) proces atari hy 00 natn takes Place in a medium with multiplication constat k= 1.067 6.280. Find the number of neutrons generated per unit time a : apliting Is eee each spliting te anomed ty celeae an enol) 200 tteVe Pe : ae 00. ‘a thermal reactor the mean Mftime of one generation futrons is t= 010 2. Assuming the mulipliation Constant to be equal t= 1.010; bade 7 (a) how many tres the umber of houtons tn the reactor, and consequently ft power, wil incase over f= 4.0 mint (0) the period 'T of the reactor, i.e, the time period over which ita power Inreases eld 7, ELEMENTARY PARTICLES, very and momentum of eltvie particle Bo mgt +0, jem VERS Ia, on here 7 ie he kine ssn of the pes, When ecntting olsoas 1 piles i pays tase th fas be ftp a te van oe teat om Collin, ete et maar ihe foned partic Me S74 Fier iy silt nic gary perils seking a ttionery pociie td setiating the ender aston ey See ge 2 ah Goreme JIE ox ~~ ae Bibi Satie ‘Semenary patel sy te proietl Santee, ¥, Napercarge, YS ion batwoon quantath nub of storey interacting patil Yor, 2ts, en) ont + ntegetions of parties aby th ls of consreatign of th Q, and 2 chtogut ie dovag arco be inws of consarvetion of Stor Pe sad Racpetion Ty a alo vad 6.201, Caloulae the kinetic energies of protons whose momenta are 010,40, and 10 GeVle, where ¢ f the velocity of light 1.202; Find'the.meon path travelled by pions ‘whore Kinetic energy oxcods thelr rest enegy = 1.2 tines, The mean hietime Gtevery slow pions fa ty = 28.5 Bs 6.20. Negative pions with Kinetic eneray T= 100 MeV travel an average stance f= 41 a from thelr origin to decay. Find the oper Istime af these pas PEA Thre ts a asrtow beam of negetive pions with kinetic energy T equal to the rest eneray of these particles. Find the ratio aT titco tthe sections of the Beam separated by’ distance f= Di". ‘The proper mean lifetime f these pion is ty 25. ns, 6.295, A statlonary positive plon disintegrated into t muon and a neutrino, Pind th kimtie ndrgy ofthe muon and the nergy of the aestrine “5.06. Find the kinetic eoezgy of a noutzon emerging 2s result of the decay of stationary Be hyperon (2 =» n 2) GT. A sationary positive muon disintegrated into & positron and two neuttinos. Find the grtes posible kinetic energy a the positron. 208. A stationary neutral paticle disintegrated into 2 proton with kinetic energy Ta 53 MeV and. negative pion, Find the Tas of that partite. What is It name? £29, Arnative pion with Kinetic energy T= 50. MoV disiteg- rated dising is ight ito a muon and-4 nestsno. Find the energy ‘ST the neutrino outgoing at right angles to the ploa's motion deee- fon 7300, A Z* hyporon with kinetic energy Te. ~ 520 MeV disintea- rated: Gung it Bight into a neutral petcie anda postive pion outgoing ith inotie suorgy’ P= 42° MeV at right anges to the Sypeon’s mation diretion. Find the rst mast ofthe aeteal particle i'Mfev units BOL. A neutral pion disintegrated during its ight into two gamma quanta with equal enerpies ‘The angle. of divergence of fama quante &— 60 Fladthekinetie energy of the pion and of ‘Sich gamma quantum. G02 A relativistic particle with cest mass m collides with a stationary particle of mass MF an activates a reaction loading to formation af" particles: mf “omy ty i --, where the feat masses of newly formed particles are waitten om the righthand Sie."Maling use ofthe Invatisnce af the quantity #2 pee, dom- ‘oustrate that the threshold kinetic energy ofthe particle m requited for this reaction is defined by Eq. (6:16), 38. positron with kinetic energy J) — 750 kav strikes a sta onary fee electron. AS a result of annihilation, two gamma quanta Jith cova energies appear. Pind the angle of divergence between ‘6.304, Find the threshold energy of gemma quantum required to form (a) an electron-positron pair in the eld of a stationary electrons (b) a pair of plons af opposite signe in the field of a stationary proton, weray T strike a stationary hydrogen 6.35, Protons with kineti target. Find the threshold values of 1 for the following rese.ons yp tPamp + pt p+ oO) p+ pop + p+ a Gch A ‘hydrogen target is bombarded’ by pions, Caleiate the theshold valde of kiwte ners of thes pine malig posible the following reaction (a) an shp eK} E(w poe Kf AL hot, tt he weaazonos& an he byperchate Y of aetel stagtary yur how topic apn pation te Td fd baryon charge B= “+4. What particle i this? 6.88, Which of the following poteses ae forbidden by the law of conservation of lepton change Wnaptety — Wateenty Qate betes Guedes Ors te OR eee 6.909, Which of the following processes are forbidden by the law of conservation of strangenes Marte d tk: Wnt p aaa ey Qetpse tes Gave a Ret A Om Ep ER tm @) Rb pede KE Re 6.310. Indicate tho reasons why the following processes ate for- ident (0 2 atts n+ p> a Tp Les Oe be te OR eno EER, Ope tut ap ANSWERS AND SOLUTIONS: Af. v = U2 = 3.0 tem per hour 1.2. (oy = Dog topo BIB Oh + oh 13. AtacVT—Toiwe 15 = ATA (a) 10 emis; (0) 25 emis; (0) fy = 46 5. 15. (ayy mlm toll Ye — 16. v= VIET OTT agrees g = 40 km per hour, 4 = 19" ogee y HO km por hour ae : fe | L @ o polnts merge provided the following two conditions are met: fomneorayatnt, fvemaatus, hors sought ie 18a fro tate two ogutions tat. ea wlio a Athy — ay = Lm ae (+ a) = O26 kn, Toward the tain wie etsy P= Ao a Go) 0.7 950) 7 aad 1.8. raps. ot Wie EAS See Figs 1, ‘Rls = 50 erm/s;(b) (v)| = 20Ie = 92 emis, ¥ (gh aha ent © fig (a) all —2a1), w= —2n8 const; (b) At = Aa, 24 and oslo merece oe Cae vee gant 0) one Vin, y ~ 2h v= VFLTBE, we 2h; (@) tan a al0H i. Kol aT oR, 125. () y=z—ale: (b) v-aV FURR, w meonst: (6) 6 12 (a) Faw) 2 127. y—V tau, 1.28, (yea + ge (by Ov) gee Weve — 8 (elle gen Pilsner # “(3e) sintas 1 (tie) sin 20, (y= Ftane — (gidtersta) 2% gain Ry ieosta, 9 130. sie 32. O41 or OH min later, depending on the intial angle 13, a1 Be IB at 124. (9) 2oeDoy gH 0) wary, we —ayV TER VF argh 1.35. (0) y= 0/20) 2% 0b) R= othe = WV nyt ob 3 V Zeyher 140. (a) toy ata 2.8 mish 1, — ao? 3.2 mis (8) wy a0 VITA 25 mist, bye ea VE AGE 057 m2 AL, R=, waa VIF ORT $42. (@) w= Dat, R= she: (0) w 148, v= 2Ro = 0.40 els, to — Au? $44. w= ol Vir tar mist. 1545. Zane! =3.0-40" ral 146. (0) (o)— 20/34 radls, @)—=VIB=6 rails; 0b) =2VFab 12 nds, Lat. 21 Gayna=1 Eas Co (9) pat 6%) apie: () w = wet bet, 33 mis 130. a VIRAG, see Fin LL. (@) p=9Is (hyperbola); (b) y= V Tost (pseabole 138 et TE Nee ae Mal ee (once vetor wtb potmanently dimcted tothe “t * {entre atte wheel) * = 82 138. (0) v4 20t— 100. ani tn — = Vium Tt ree (5 WHORE=56 emis, wm me Conny 2.5 omist, wg 13 Rai By=2 VI 155. o@Volbotes radi, Poe, =12 malt 158. @) eaai/fF Gia) =8 milo, pa VT FOP ated’ tr 1.57. (a) w=v/R cosa =2.3 rad/s, 60"; (6) B= (v/R)ttana— 25 ral Ls anaVOERTAR=08 nds, pV T FOR ‘ile Om ~ Smug + 0) 190. we tae EP 4 Pin. 3. dom Fam eS 1.61. (a) 5 (B) tam en oe = tint — Aye or tan = 0.16. imino ales + Beast) my fy = bmg cts d's mame Soe ee Sa ins Lor, tan £68 @)» fmt am 1.70. <—V2TEe Te on mE 0) Fm 2 (ging, 472 "Bg anna in), 2 no RE Ag 1.7. Py aia a fam ee : % 7 wa = wi + neat), w w= eV 3/00 Mm qin = 8 — I F , pen = 81 + Heat api a — 8), oS psn a cos alent + my/my) = eltan a + not a, EB REBRES one: a) (F]—2V Pmeelak; (b) KPL—me tae ODES ©) KP= me 185. (wo VTE TED, P—Angcosts ) TamgV3, (6) cos 1/V3,, Om dur. 186. = 59 187, Omarccos (ig) 48", wm V TATE US8 e= helmut 1} is ‘indopesdent of tho rotation aie 1,89, r= RID, Uae Va Be 1.90. 5 = VaR V (hero —t 1 vce V eta £92. 7S (oot 8 wR) melon 4188. (a) Let us examine a small clement of the thread in contact with’ the pulley (Fig. 3). Since the element is weightless a7 — Mary, —"bdhy and diy — Pda. Hence, of? tea, Tategat 0m. {og this equation, we obtain = (ln nya; (B) to e(g— gn +m et. Bm iA) cot mons 1.95. F= “rate, where © i the radius vector of the particle relative to the origin of coordinates; Pans V Fay 196. (@) Ap = mages 8) | Ap (va tor, "ep O) ee aati, 1.98. ¢ = (ut — sin ot) Fema seo Fig 6. 1.98." 6m plo; # = 2Fyimo, maz = Feimo. ‘Lto0."3)» tee) 1.402, Instruction, To reduce the equation to the form which is convenient 1 intograte, the accelerstion must be reproseuted a8 doi and then change of v VE kori. Chim | sn (WH!2)[; (B) AS =BEimot, (0) = 1.405, 0 = v(t 608), Inateuetion. thertore ¢ = Wy cmt from at const = vg, Besides. vg =v eo 4607. = it — coe ti) Rell Here te, = ing, and initial condition ie follows 1408.) 0 VEIT, 0) 08,2 EEE, vars ye amg oye 1108. Form <1, including negstive values, $10, When g*f > 2. there are two stanly equilibrium pos tions: Oy = 0 and 0, ~aecece (@/a*A). When tft vas thee w= mimi my £5168. (6) n= dm yn > dmmalims i ma 9: ming a8 (i img Bk = 2 wig a = 1D tpaeo(: VERT) 1.0 he pe cad 1.172, Wii coatiaue moviag tn the came direction, althoogh this time with the velocity v= (1—VT— 2) For nt the velit ze cme (78. AT! wet 1th. APF = (0 smtan tat 6 + mi — 1» 00%. L176 @) penVET HR) T= nwibt ep. Hew pe mamaling TP sa Ore alms 1176. v=" Gt oi — Wi. Respectively at smller 9, equal, oF greater than V2. 1478. Suppose that at a cortain momont ¢ the rocket hes the mass m and the velocity v relative to the reference frame employed. Consider the inert reference forme moving with the same velocity js the rocket as at a given moment. Ta this reference frame the ‘omentum ineeoment that tho system *racketejectad portion of as” sequites during the time dt is equal to dp = m dv +p dtou = Feat Wat follows ts evident 7 15178, v= ull), 80, = mye St8t. = (ary) Ini). 1.182, v= En a 1183. y= Fei, Froth + pel 18h, & 1185. N= 2b Vai. BM = (mi!2g) sat a c08 = 1.487. (a) Relative to all pointe of the st ight angles to the wall through the point Or (| ANE 2 modes 3. Helative to the centre of the circle ight line drawn at jam] ney 1689. | Me £3190. = moet. 194. mo hr 1 V Daten, Hisa, ena, $4960 3, ngs £195. A1°= fmt sin 2. Will not change. $196. MM ipl. In the case when p= 0, ie, in the ‘ram of the conte of inertia, 1198. Hy Im | enas © miynit. Tho problem is easier to solve in the fame of the Contre of inaree, 1200. 7 Bey Ae? = 225, days A201: (5.2 hens: (6) 13 mlm 2.9-10-t mi 1.202, 7 = xV'ir+ FORA It is suficient to consider the ‘motion along the circle whnse radios ie qual to the major semi-axis of the given ellipse, (+H), since in accordance with Kepler's laws te period of revolution is the. came. 1.208, Failing of the Body’ on the Sum con be eonsered as the ‘motion along a very elongated (in the limit, degenerated) ellipse whose major somisaxts fe practically equal Us tha radioe It 9f 146 Earth's" orbit. ‘Then ftom” Kepler's Taw, (u/7)¥ = [(/9) RP, shore + isthe falling tie (the tie weeded fo complete ball evo Intion slong the elongated ellipse), T ts the period of th revolution around the Son. Hence, x= T/4V2 = 65 days 1.204. Will not change 1.208. 1 = 9 IAT Ta 1.208; (=) "0 Cymimgirs (8) U = year In (4 Paimitiata tt), S288 ° ‘i ‘ 1 ‘ 1 1207. Ma=mY RimerraNTPD. where my ie the mase of the Sun. AOS. B= 7+ U = —ymmgl2a, where my is the mass of 1.200, ram got VT=O=Waairal, where rare, ma beng the mass of th Sun, 1.210, Fogg Comal LV) FUR me — 1}, where ms is the ‘PAT AIL (a) Fst let us conader a thin apherical Iyer of radi 9 and nats 84. The enrey of intoracion of the partite witha le Imontary bolt 83 af tat ayer sequel to (i. 8) aU = —y(nBMI2) sia 8 8. ¢ According to tho cosine theorem in the telangle OAP I — 0? 4 aa "For cos 6. Having determined the aiirentil ofthis expre= on, we can rodueo Eg. (*) tothe form that is convenient for iteg- ration. After integrating over the whole layer we obtain 60. Setar Ae ally integrating oye al layer af he apes, ‘we obtain U—~ ymiiry () Be = —aU/ar = ym ot Q) 14" taer areah Therefore ‘these volumes will attract the particle A 1.217, (a) Let ws subdivide the spherical layer into small ele- iments: cach of mass Gm. In this case the enerzy of interaction of (heh elemeut with all others is 812 = —ym 8m/A. Summing overall Bip. 1. ‘loments and taking nto aceount that each patr of interacting ele Stents appears tice in the result, we obteln Um —om 2M (Ts Synth. nA _{ 45 days (00), 4.218, At 77 sararzB™ | 0.84 hour (6=2)- 4.219, wy: wh ey = H+ 0.0094 : 0.0008, 1220; 32 kant 2690 km, 122k Rien). 1322! k= i ighth®'— 1). 1.223, r= Yat (Pop = 42-108 kam, where M and T are the rast of the Earth and” i=. period of revolution about its ows sig respectively: 3.4 m/e, 0.92 mi 12a ar astrelyry (Tin) = 6.40 kg, where Tis the period of revolution of the Earih about its own axl, Haan, 2g BE 0 ls, WIM (14 BBE x Ve) ~ 49 mt Hore AF is the mass of the Kerth, 7 is ite period Bf zevolution shout its own axis Pa. 31 times EEE Th dosreace in the total ngrey ofthe sate over the tint iaceal ih ean to 08 Pak Repreeating B ano as {udatno distr + batten te satelite andthe ente of he Aitsnne ean vue te eqheton, tothe form convenient for ine atin, Finally, we eet n= (VA.— 8) la ee ost OS ana og 57 ka 1208, So VITA (V3) —0.70 kml, where MF and aro ihe ins andthe radi 9 the Moon 1.230, Ava V eR (V5—1)—3.27 km/s, where ¢ ts the stan~ dant Teecialh acelration, isthe sedis of the Ba La3t nit} 38-0 i. 2 1.282, A a ym (IR, Myla) {1.50108 A, where A an ‘the mass and the radius of the a teehee ome 3t. t= 20F,ime ~ 1.0 m (295. N= (eta, where k fs the unit vector of the # = jab —bA VV TTR. 298. t= Jad 38 VR 1.287, Fyy m= DF, This fren is pra to the @iagonal AC and fs applied at"the midpoint of the ade BC Ps. i i 3 : “home (6) Pm on 1-288. (a) 1 = 44 pbk oa 130, Pah i 15 ita) mee = 048 kee Ta (+ Mz 0) T= met 14, T= Same, Wy — emri/T. . iatatad 1.265. @= V6F sin gr. 7, _ mu in-bty sa trot 1.207, A= ra 120 ne SBR a 1.249. t = ‘iobt/kg. oe 13h io ah BL £2 Seti a om CO) ae 28 gmt th mi uP sinh Be eI = 510% reales By (g — wd), P= ym ig — sinai(t Fim) = 16 met ae = gl — Ss omer = gl — 38 1.257, (a) w= Loe, gy 4 Patten rine Tomar a 1258 T= ty mg oD UBS AS amy + om 7, tao, (o wn ftattan yy raat Laat m= aya vee LF ig) 8 = Nim 3. «Viste ee yn aa iar 367 Pg mt 4 8 0 ms. Soo eae N. Az = tly is 1272 v= oll t EIR. UHR Pail oR 1216 (v=, OP . re (Mim) VATRT sin(al2} ) Talla (a), (e) 2a, CBee eS SEPA van ATT. (0) 9= — yet 9's (b) Nem — 3 $278, @) «= pt 4 tam v0 Hh Do (a) Aue = Ht a -H foo Fema ot eae ERE + 2d. 6.0 mates Poms fino nn be Me He nae = mgliTy = 0.7 rads; "sind = 10m. See Pig. 11. = G+ 0) Hank of = mV EP Tue vector" forms the stetan (wig) with the var 286. P= 24, mea! !t = 0,30 KN, TERE Paes camry a8. WL Dante = AN, TBO Rug ='Bunportl "CRN. ‘The force exer on the ‘outside rail Terwoses by this vaiue while thet exert on the inside "0" atm, whore is th thermal expan "L291. (a) p= oq Ari = 20 atm; (b) p= 20q Arir = M0 atm. Hire is the glass strength. 1.292, n= V2iq]plnt=0.8-408 rps, where oq is the tensile strength, and p is'the density of copper. 1.298. n=ViqJp/2nR=23 eps, where om is the tensile strength, and p 18 the density of lead. 1.206, 225 Ly mgia PE = 2.5 om ies. = PES. m2 il an.) alm Hy pgDE; () AVIV = (L— 29) a, een and jes tio for copper. 7 steel 4301. (a) Here is independent of = and equal to tng ec the iia Egan sith aur to the bana tons dyids (0) = 0 and y (0) ~ 0, we obtain y = (N, fy the equation of «parabola. ‘The bending def SZ NehDEL, where I= GH2, (Wn hs ease 4 () =F (L— 2) and y = (F/2ED) (0 — 2) as 4 SPPVBEL, where J's of the same magnitode In () , ALS 4 potP/E, where p is 1 FUE = 1.6 mm, where y is Poisson's oa hii 1.903. (a) = */y pRit/ER* (b) h = "ly pgiVEh*. Here p is the deca it Signe, wre te denny of se ta) @ bar Schon (by Sata 1B 2 5 at a) it) Po Nm: 1B 8 = ies ra £308 yA Bf ie 13on, BS VEnCsp — OU, Deep isthe density of tee 1310. (2) 0 = HaaPtgiB; 6) 0 is (aU Hore Js the density of steel. Ce : ABIL a Me seRO°E It = 0.08 M anGgt = T'S a = Hs Gate ABE. = HEB (ophyt = 2.5 LE/m', where fin the compress bait 10315. ps > Pero < Pe Tho density of streamlines grows om ‘rasition Fo print to point 2 1.316. Q~S,S, V HGH S5. 1317. 0= sy Tethpte 1318. 0/2 (hich) —3 mls, where py a0d py are the Aeasities of wht snd hibieoe, Cee pera 30 em. 1300. hy hy = 3 a, 1321. p — pe + poh (h— Mi whore Ry . ‘se of the selaton betwoon the aceleration ion w hth reference trae fed to te Earth wen s—VT—P op, where B= Vie howl be shown fit tht IF AL = 41> 0, then ‘This formula is given the clution of the foregoing problem (item (a)) where itis necessary to assume V = 2. tntograting the fiven equation (Gr w= cont), we obtain 9 = wlV (OTE. ‘The sought distance is 1 — (Vip WHE! — 1) ec! = 0.91 lighte year, (e-— Oe = 42 (clara = 047%. 1.967. Taking into account that v=w'iVTF (WHR, we get sen | phage ae [V1 (FEY) =3.5 months 1.308, m/my = aes = 70, where B 4.900, oe) a+ mt + 1) ~0. of gh "The ge ton’ of ensty"as the indy to ts vome employed here Testo. te aye = nN tml" = O48. 1394. 0 tem VT = Se 1512 AO ne tatend 046 me 1.978. v = Ngc¥3 = 2.6-10° mis, 1374, For e £1 the ratio ib Timg! < My © = 0.008 1375, p=VPUPEDmeie= 1.09 GeVie, where ¢ is the velor city of Heke her i the velit in nf te st me of P= ee) VF TED, P= Pie ‘B77, p= Dumoti(d vee. A978. ym Peal mE PE, 1 V ORT TE mb ina. 1.380, a) In two cases: Fijv and Fv; () Fs mw VIF here Baie where Baie, Vote Ph aie tn a at sha pa 1.385. My= V Img PP ImCTe, Vey TAT Tig. 1386. 1Sar FS amysimgt 1 AO ABET. Ey meg = SEEM ma os, THe particle m, has the ignest ergy when the sntrry of the system of the remsining two particles my and my isthe lowest, te. when they move as s"Sagle whole. 388, 0 (wim (as in solving Problei 1.178) and the relativistic formula for Velocity transformation pF App { ear @ yy ~ 5 Bas. Ono aha) oy = Oy BM alee ga amin pila 287 9 = Rin y) ATI — 1) fy — 1) = OAL sO) Gre) ob we 8 <0 for tcnay 2 1/Ksmol, where Pig. 1a. 248, 0) AU = athe = ty — 495 0) A= HV OF — 0 Beta y+ ig fae. et Ne My, rvs-nm const) A= Hr, (tf @ 2501 lad BRAT: 0) = Ally — 01+ BO — a 6 <0 for a > ag — Bat. te) 4 Pi te @ fig UU + Nek HO a) Cy Ras 0) 6 = Cy + FA a 259.) Gm hed i) aeioare” a) “alm po tvy — et YA 1; A (y=) Fan (VV); @ V3 th as Were aia: Oe + Arvin iy Bees) Pet = cons: = cout. 8 251. Am AT In gt +0 (fe der Waals custants 58. (8) AU =a/¥,—a/¥,— 0.11 eu. 290, () TV — BMY — comnts (b) Cy Cy ma Sa 260, a= — penton 20K. 264. 0 = vats! Vilisyy = 0.0 262, n= pk? = 010 cmt, (y= 02 mm. DU a maTIMY = 1.9 atm, where Mf is the mass of an Ny ole. ol ar — pnt mn) 50 en nae oe al al tS 266. = Bioep — 1) = 5. i one AEEBT os 00s 09 098 AE Oy vag ad ile te ye he EE, — a) my =P Basal ah AY = aS at | Hema etn aa » ANO= 1 4y(3N —312) for lines Se Ie cei pmo BI) ToS titty ey a Ae = san gt haere 275. (0) vg=V SRTM = 0.47 keys, (e)=*/4kT = 6,0-10-% J; BY IETTap B= 05 mis me EB teint 5 molecules Ab. (0) BNI = Y/R) 5y~ 1.60% (9) Na BY Be 285, @) T= TyiF 2a () P= BAD 90 Kj oS scan BV min eteuts 2.8, = / TAT 2 .6¢ 2380, P= umn 2.0, dBA (Spe) Meme 200, dy ge 291. p)=0, (os) VERT. 232. SB okhim. 280, v= indo where. (o)= VERT 296, p= J Omoy-oydn(o,) = nk, where dn (og) = (maakt)! nce mBt doy 287, Te: ‘The principal pian the value of the integral is provided by the smallest values of ¢, namely e = ey. The slowly varying factor V Fan be taken from under the radical sign if ascribed the constant value Vey. Then ANI =2V ek ot, 2.99. (6) pr VORTIOG (b) typ = RP. 2400, eve F dn (este) cond =m (kT)! sin Deon. 10-8 x BAG. NTS Gntiad dog ie noe EL AO% mol 20k ns team 2106, warns change 2102. (0) — BT Bows not depend Oa ania 1 2.408, = tele Toa ‘SHR — 20 nae 2.110. w Baath. (a) a Tis dr (9) rye WET (0) ANEW = = (ofakT)emerinrénrs dr; (0) Will increase fo BAND. (a) AN = nna) e-UAF VD ad; (b) Upr= Mab. BEB oe [rae EOS Hie sc] 0 Rie sei ti. nbow oo ei 2.424, In both cases —4— Act aime gat act, 2428, (a) n= ty SEE (b) nt 22h. @) nat papas hy net satan . ones 2425. 1 Taye coe ea 2AM, net 2 2.198 The ineolty | S2— {Bio wenmes oven stronger when 7, is replaced by Tess and n by Prune Then Qy/Timox— = QTmin <0. Hence Si Si Fowe=Tale or n-ne 2.12 Accurdng tothe Carot theorem 4h = afi 7. Lat wx fd she sprsons for 6a'and 80, For an niniesial Corot goles (og. Psiclopram 7894 show i He) 44 = apea¥ = (iploryat-av, M0, 2a # 9a ~ (OUI + play. ett Teremias to substitu the to Ittrexprensions nt the former one. 2430, (@) 45 =£2—19 34K-mop; ) as— Tle =% 8-0), ZAdhe n= Oem Bh Alvan 230. 85 2AD4, AS = (yaa — Inf) wA(y— 1) = 18 VK, a = 20, an 1.0 4K. 2.137, a9 = 20497 nw =06 1K. 28. Yanna (in), (Ria) in (V9). Bln {hy — by, — 8), Bia a8 = Erle Tits) An, — Buh — Bh Bia s. 24 aS 0, ASK) Pent; 8) N= ISLEY we 0, where HO» the mean time which tes « ells atom to cover distances of vistel's dinensons BE Pan = Mp" m 24.0%. 1192, 89 1002, 40/9, 9/99, 1/92 respectively . BABI. Pym saggy PMN", where p=ViVe. 2.158. d= 9/ agri 0-4 yn, whore ny is Loschimidt's num bor, (n= tn 10-40 ‘2.159, Will increase QI, = (1 + A7/7,)!%A% = 1ohst-1e" snes, 2100. (2) a 261: b= dalpgd ple vo C= aint — = ald = 18 atm; (0) Ap = Sald = 1.24404 atm. 21 em, © oes. "Ep eld. am B06 Rf ab 2 a t= Dig = 5m. Ego. an "tacos 01 aNiddypa = 8 266. R= 2aipgh = 0.6 mm. Bor. 2 2 Hd pydléa) = 1-4 em. 21108. c= log poll — B)l ad cos 6. Ble 6 lieth EI A eS Sea? 2A. V4 = tad / BESTE 09 ent, RATE. Ry — ym gta = 050 om. RETR ot sale Get eh = 02 Eire BS Ema OG NT OT Ae Eins Flak = 08 WW, 2476. FH dottogae = 18 No Bin t= atgnan, Bite. = Saat 2.478, Mrs nad? — 3 eer eee 10 J. Bawo, AF — dead 9-10" 'sy 15 BaBt: A = Fpl in ip), whew P= Balla, p= pet + beh, V = Yai. 283, C— RIG 4 pala. 2AB4. (9) aS" Fee Bo!) av = 2 (x — T dea x x 2485. A = AmRTIM ~ 4.2 4 V — m¥)i(Vs ~ Vi) = 20g, Ve = 1.0 1. Horo 1; is the specie. volume of water. ABT. Im ~My (Vo — VJIRT — 2.0 g, where py is the stan dard atmospheric presie, em th AVN — Ah, = AUN a, the charges can fhe sur C= dae (8) Cran = Cy + Ca + Cet () Cet = 6. {a} Cm ey S/Sd: (0) C = 348124. Vier Wed CuCl = 9 KV UB) Ean) = 0. B.t16. Ce = C (V5 — 1) = O.02C. Since the chain is iafnite, ait‘ lnk beginning sith the second ean be eeplased by the ce Daclanae Cua 10 the rsh oe St 0 vac, = 5, = Gan as Heege, cp where 9 Siw = a Rt Clcg, vate, — aad + cy Bu. 7 Suet alee by 3120. ga ap— Fog teres. Inthe ese whom CC =0i MBL oper 0.08 me : Be a Ee csc +60 Se fancies hs Sau ENC, CCS 80 ml y= BE —C)~ +808 6 (Cs CRMC H +O ‘a icysco BG t HGH Hp Tees 3:27. () ow -VI—4 (6) W= —V Dythneya. 2128, Wee 22 BB = oA S550: W = gaa S181. ae TEMAVIE CAC, + C) = —0.08 mJ. Peeyoe +c Sis 8 2 ee independ of & BAS Wm Wy We + Wag a+ hs we (ih + dh BIS. (= Seat We = Bs 3 piBaege) (Ma ~ 110) “27 a, Sit 4 > (ores i AR. suas aes copes Ho Ole ie — a BAG. (a) A = g? (ty — 2)/2e 9S: ot Gyles Fl Shey AEE — or 15 wy, he EAMES Bae Rh wy “ B48. (aya = fay F nasil $m): 0) @ aa + nail +n B.A50. (a) lt, ) att (0) Hal BSL Ry RVS =A, 8.182. #'= (1 + VES GTETI) Hyl2—=6 9. Instruction. Since the chain is infinite, all the lis boginning with the second can be roplaced by the resistance equal to the sought resistance Mt 3.153. Imagine the voltage V to be applied acroce the points A and ie Then Vin JR = Toft, where Ts the current carried by the Tend wires 7, is the eusront Carried by the eonduelgr AB. ‘The current 7, ean be represented as 2 superposition of two cur- eats If the cucrent T Mwed into polnt A and spread all over the Infinite wit grid, the condutor AB would carry (because of symimet= ty) the current 1/4, Similary, if the current 7 owed into the grid from infaity and eft the grid through point &, the conductor 4B ‘would slo carey the eurent 1/4, Superposing both of these solutions, wevablain I= 1/2. Therelore, R= Fy 2.194, RU iat) In bie) p lb aidzad. Tn the ease of b nce A = pléna Fnarabite — a) Co 9 k= pda: (ay) © atin: R = ofa. (3) j= 10/Dpr in ie): 0D) By 9. Pm VCipeeg = 49 A. C D cose | = ¥5 (os ont fn oso) Sele ea eV feos "= (ola) In Qa D sin aleew, Soma. sono Coeds + Pt) 0 = 0 i op ol fede — 2). faV Gy He Gy + 1. S400. () Ry = 2naiS% (b) B= Daas? Simo. facing Viv) = 00 ys BIL p= weve n= 1.4 109 O-m, B72. 1 = [ey 1) grad e-nune, Bim. v= Bly = 20 v. (E, — BY uly + Ry — By = —4 V. iy, Ap = On the source of tarnent with in- Pee pret ay RA | iy = 05 v, Feat cn a oS ong BAT. V = VateliRd + yd) ats = Vial SAMO. F = HAR, + Beal, + Ry), Rem RaBlCR, + Rey. Bast. 7 — Unis — REEMA + Ake + Ral) = O42 A, the ‘current is directed" fromthe loft to. the right. (eo Fig. 3.48), B.A82 (0) Lym (Rs (8, — 8) + RB, + BVH Aly + Halt) = 0.08 As) gs — op = By Tih, = 0.8 V BABS. 718 Ut + Hy BRR (RR) + Rutt) for RR Vw 3.184 9 n= Bay Ry > By) — A (ily SACRA + + Buty Tigh DY BRB Us tor = + Ra, — way + Ray + as ( marca SR eee —4)~ ‘The current fows from point ‘C to point B. UE elaine BSC 2p, 2A Oe — In DPR = (totic) VER. Fidei, +R.) ~ 128, RPA sh MAIR, + Ro 198. 0 = CEPA Ry) = 00 mI. 8-200; (a) WW — “even — 9) 0.15. m3; (0) Am = evigtt yy = 08 mr 20. AW "9, (e 1) CY = 05 mt, Arash = = ite cv = OS ms SRR a yey (C1) Vlog, where p ie the density of water S.BUS. (a) g aye teers (6) QM Nia” Hid) gene 3.204, (2) aol — eRe) = 018mC; 0) @= = Se aegsbe 89 aa (a) T= (VR 00; ty = HCI im = tavight = 18:10!" Chg, 3.207. p'™ Umle = 0.40 Ns B08. $= ent" (oy) ~'10° my, where is the concentestion of roo alecirons, (e) Ts the meen veloeity of thermsl motion of sn sdectron, 0) P= enipt 10. MN, 3.211, (@) pla)= — BRIE n= les + 33 OPV, (fc = Lal = BAO cms) 3.218, (n= )/V oa, —13 os, 3.216. t = egnUlend? = 46 days B2IT. T= ye B28. J = (0 — 1) cil, 3210, fo) Bm gl BR — 6.3 nT; (b) B ~ pgh*ZiQUR? + 274 B00, B — mel tan (in)2A, for n> oo B= pal BR 32a. 8 Uhifind sing 040 a? 5220 BS Gree stan) wy llDnh ~ Diy, dams. @) Bat (BEES); @y patel (SE BBY. Bm wilistRr whore 7 isthe distanes from the cat BBB = palit 8.206: (a) 5 (edn) (n0/Ry; 0) B = (ula) (+ 82) HR: (Bm eat) ay 8.227, B= (ihn) J V Bil = 2.001. HRI. (a) Bo (ula) Vea LIF 0.30 4) B= (ay) 6 XV EET aELN 0.344; () B= (ul4n) V EIR O41 pT 5.220. a) Bm sO) P= pal between the planet aad BO outset planes ait fos the plat, i outside the plate 3.201. tn the! Baeapaen with the sznght wie, A — ist dstagce from the wit, Ts the other hal space 8 = ad The en tar eal 6 2 Myelitfor r=, 828 BSL uate) Rr fr rR 8.234. B= aug Ith 0. field inside the cavity is uniform, B28. 716) Bing (he a) 3.250, B= 3.236, B = wynt/V TE ORTE, B27.) = Maul (1 2/V FER, where 20 outside the solenoid and 2-20 lose the solenoids see Pig. 281 @) sy ~ Raney aT a a sa on a eh ae 320 NO = Gigtay 1 3. etiam, = oft Baki = Oy2 = pynSi2, G ae the ot ela: a {or B thrash the ros sction {he folenotd far from it ends ar S242. = yibn) 2710 n= oe: = Shh. as Pn = 2FPBlny = 2 : ot fae <3 min Ba pg = NTE Pe 8261) BeAr 0) py = ttt (a ab 1) = 1S Amt 56 Sait 0) pm = Hoa Be Balt waa iat = 3.252 Hm oq/4RI — 8 KE, where om ie the strength of compe RSit. # = GpgS/1)tan 0 = 10 mT, where p te the density of enppe SEB a — amguaris = 04 1 3256, GF ida Gor 1) = O40 wN: Am fai) tn Lye? Ate 2 0.40 4 326. Fh Vile (nin =0.85 1 SB Rae se, oo Bein 2 5.250. = By ‘eo, tn Si thee" cases, = (Bt— By/Duy The foe ie ire a tothe Tight. Tho earrent fa the conducting plane fe directed Begg the dain ‘3201, dp = Zia = 0.5 UP 3282) pp ioa 3268. p= ayer BE aR, WHEREATH- pd/S); when R = pdlS, the powor y Soc, pmo V8 = 1240" ‘atin 3200.) Fa) P= ants) Sw, FEB (©) F = (ue 4x) 2pm, Pr. Sen Pesta dane 28h Bait. Fa rate ol Sam 7 = SES 05 ka ata, PV aninra core BAH) GAB athena (b) § Bdr=(1—p) Btsin, 3.225. (3) Tur = pti (0) Fo S218. Soe Fig. 24 7 3.277, B= aie ‘ Bunl(h = w * U2 4) S80. fT A Saal. a x bBined — 8.2R2. Whon b << A, the pore saeaility an Zaina Fig 28 wg) = 8.7 10% S285. 17 — 0.06 KA/m, tinar © 10-10% 3.286; From the theorem’ circulation of obtain ‘xh: in opposite divections 1¢ vector Ht we a Seth toy 4.51—0.08701 (kala). Besides, and Hare iterated as shown in Fig, 3.76. The reqie- ft vatiee off and must simultencoualy sty boih anton Scicng this system of equations by means of plttag, we oba i tiap kali B aatds Ty and j= Bll © 48 Bes Fs ASB B26.) zy 1 a (0) 1 paFae VOOR ber Diath 8.288. §) = By V Bola Rze0. 1" BoA, where Ry = Rly + 3200, (9) be= viataimie = 3.0 "2; ) Ag = twBa sant, { Bat —MoRat—10 a. 8.8-40%, on 3.292, Ey = alt)" BaBt, where n= 4. 2... is tho mum Der of the half-revolution thal the loop performs at the given mo- ‘ment £. The plot & (0) is shown in Fig, 25 where 4 = VIEWER. BIB Tt = ah whee a = nln 3.208. B= & 3.205. 8, ae] (ox cae amg sin aya. 3.206, omen | 3297, w~ tae BE 1m) 5) eae 30. Bm Sights 3.300, 9— Se dent of £ 30, 0) T= ABE In bs 0) Feng ( de 3.302, (a) ¢= vynFPB*, (>) Q = Sano 3.308, v= (ere, where = BPR, 43.304, (a) Im the round conductor the current flows clockwise, tore ts ho-current in the connector, i) ia the ouside conductor lost) hth rut conductor, een: o erent thogamgecen, hi th Teh e oftehe ih lrrae ‘3305. T= "o8, (a — Op = 0.5 An 2 3.306. Bim Vilatady 8.207. B= Hel Be = 1 mv, aos, eof Vgunir for ra, Mauonfatie for ra, 3.800. 1= VugnSalip = 2 mA, where p is the seistivity opper. SBTO.'E = ab (n — AYO +). 3311, o= — EB, 3.312 Fineem tt 3318 9 = at Fe ay oa. 3 lo VRT Re ba 3.36, b= Bt BE, where pnd py te the rity an the sony of ome Bis tn Enh AB 8.38. raft p07 ms, where p isthe resistivity, py isthe ensity of copper. 3.9, 3.300. 3.321 a_i f= Bitseey 205, whore HF a () Waa W wind 8.0; (8) mB 0.15 WY, = bata. BBY a: 1 Vim hs e (8) Ie Lyn HBS lo 0.26 Ht, Nia tn (1-4-2) ib = 25. alt tyxtelan, 3) 1 nats) A = Nate ToC) ean ute) Pett ct tem r= 8 q—e-ay, a : ace (6) yl 0) By = ane Pm = Dakalue Ei Fane, i NUiab 110, 0) Haas = 8) Ia ‘Re ipinaent seat "thauld be taken ito account in addition to the'eonduetion current. M9. Eq = TaleyoS = 7 View, B3G0, H Hq. (ot +a). where Hy and cris determined ftom the formula tan a Svar war - a9 ir tor rec, ‘yBROr for > Re Hore = peal sina 3.852. (0) j= Zs (0) be gy 8.853. tq =0, James Gas fe 3.364, HL 3.886. () 1B), then B= —omlot 0. The spatial derivtiver tthe fl E, however say tot Re aga to a (FE 0) aly tm the presence if an lc ll. OBO then PTE aa 6 But i the uniform tad bc Be {ols sssumed that E~ a (), wher ais vector which it indopendent ofthe eorinasn, 1) an abe fonction of tine ‘hen wablot "9 See thet the eld dacs nt gy wh tin, Generally speaking, this eontrades tbe eqetion Be HE ‘bra form heap i lehond sie doe fet pent on time ‘heres ie rgb aide oon. honky ened the. ee shen fin einor Futon Tn this case te adorn Bld Econ Ie Tinetapendent 5356 Let wh the divergnce of he two ses of th eqution v SHS aplae Stace te vvvczonta ofa tor aloaghequal to nero, wo get 0= Vf +L (V-D), Ht remains to take into cout th V-D ~ 0 Sire ap condor the divorgonce ofthe two sides of the fet equation. Since the vergence of core ehyape equal to suo, ¥- (MA) = Oot (VB) =O. Hence, WR = cnet whieh 3.364. 1 oes not contradict the second equation. ‘458. 0 50 Ela S350. FS Tel S360. ¢ = eo ~ 0.40 pCimt 3261. p= “Dea = — 0108 nCim8, oe =2 pCi 8.382, m= fe sl) .364. B= bel, whero Is the distance from the 2” axis 3.385, p’=216I, whore y isthe distance from the 3 axis, 307. «) = 8 Y/ EERE a0 avin; tana = whence a s51°; () we eS = 14 wT. #2 3.870. B' =8 VT (ERDF 0.45 wi. B.371. Suppose the charge g moves in the positive direction of the # axis of the reference frame A. Lat us pass into the fame J ‘at whore origin of coordinates this change it at reat (the © ond 2” taxes of the two frames coincide and the y and y° axes re parallel), Th the frame A’ the fed of the charge has tho timylest form: EP = = ape, with the following components inthe plane 2, y Be. 1 i ‘when the of the frame K, the = and J projections of Eas. 3.6), Ben fi, 8y-5/VTORR ee ean [Note that im this case (y = const) the voetor E is collinear with the "vector. 3.872, v= V Talal 16 kms, 3.373, nat / BBT4. (0) 2=28o; 1h) = gByim 8875, t- LTTPEEREY = 9.9 og, e 356. w= aE 8.377, (0) tan 0= EE VT= TIE, where ead my ate the charge sd the mast of «rans (0) v= ee/V PETES ET 4378, a=aesia (28 V/ gp) <3", 8.379. (@) v= reBim = 100 kms, P= 2amieB-=6.5 5 () NTE O81 oP a PI ot 8.380, (0) pmord; T= mgt(VTF CMa —1); (e) w Tim, 5 keV and 9 MoV respectively 3.382, Alm 2 V Ini TeDF cosa =2.0 ex, 3.383, gin = yee 3.384, r= 2p | in (9/2), wher p— BE sin a, 28, where b= fe = m= TTR 3387. @) n= 2B; O) una for 24 this equation reduces to 308, s=tua / FF y-GnBnon 2 Sie Pgs ox. 3390, t= 2822 tan g=6 om. 01, gm = SG 2 (at — sin at), y = a(t — cos wi) where a = abit 1Bim, The trajectory is a cycloid (Fig. 26). The = mba, a y a ig 28 motion of the particle isthe motion of @ point located at the cm Gln ctcle of roaivs @ rolling without slipping along the = axis 0 hat {ts entre travels withthe velocity v= 2B) (B) 5 = mB/@o, © w= BB. 308, v=? £( Mh) In 3301. bgt V BV. 2.305, yoke taint, 2~ aby (in at ot cot 02), (Eqn, The trajectory has the form of unwinding pial. 3.806. 1 > 2a'imrdrie = 010 MV. shore 8.397, (@) 7 = 12 MeVs 0) Sain 8.308, (a) 1-2 7 pss) AE Og, Instruction. Hore s~ 3 0,~ SV, where oy is the velocity of he particle after th mth peamge across the accelerating gap. Since W is tage, 3) Vn~ | Vide, et 8.900. n = 2avWetet = 0 3.400. 0 = of VT ai, where 0, = afin, @ S01. 8 aim, pr 3.402. 0 = Web = 5-10° revolutions, ¢ = 2arN ~ 8:40" km. 8.403. On the one handy Brae T. phere pis the momentam af the electron, ri the radius ofthe of Die the magnetic Mux acting inside the orbit On the other hand, dpidé can be found after diferentiating the elation p — eB for r = const. TC follows from the comparisen of the expressions obtained that Bqldt = "ad (B) / dun parton, this condition will be satislod if By = My (B), 3404, ry V 25S. 2405. ablar = Bry) — vy (By — 0. 3.406, SP — deed At ~'0.40 keV. 8.407. (a) W— (VT Vee — 1) me 4st. (0) See Fig. 27; 0) (atau)? (zl eBAWiamte, 0) 2 Warren, Vand ig = o's, 4.2. (2) The amplitude is equal to 0/2, and the period is P= = slo, se Fig, 28a; (b) ob = Suhr (a'— 3), se Figs 280. 43. 2 = aeos(ot +a) =—20 em, ty = — Bf ems = VEE Cale), a= arctan (glad). ‘where 5 {ab Ton V RB 02 5 cP O50 amy to) ta 449, T=2nV nlie— T= 1. 420, T=2V Tg 12+ wes (a). 4a ta 7 BLEED sta 9 422, T= VTamigge2.5 = 438, To2aV/ nil apie 0.43 » 4.24, T= 2a) mie pm) 425, T=2AV mix, ‘where x= noalles +m 426, = V Ta 427, 1= 2a) wipe ee} ~08 6, 28, Ta nV Bike = 1.8 420, 5+ (Um) = ws the dace | ogy relative to the conteo of the EarUh, His ite adie eo; 10mm Bani (9 0 = ondard freefall acelertiony () v= 8V'Rip = 32 aim, tern [Aeterna a on tan aln-tsin (otra) ms 08. 430, T= oe 08s, whee [gt Hore nis « whole aumber of the is Souie Viero—tprcorh ‘a roa, 7 4 eV R10 mi ane = 06 SBE gy 82S co Sse Fer b= Ob soase Oa scr con 4 1.0, Here t is expromed in coda ee ey a mae = Oa RT gs 433.) Fome (14 22 cosut), se Fig. 30; 2 = nut (0) gy ala? 8 cm eV Be 485-06 m. 4.36. (0) y = (1 — cos ot) mer, where © = VR; (b) 7 49. aPldz = tin VFO = Ing, Tain = 0. 4510."Tn both cases a = 7 (rot gute 3 : AL tgge = 2 GPa ete tla; ) y= (at — sin at aah, La cad sess, 154 fie 6) oy, a) Sat" = tcloche: ) w = at. ia, =" V EBIng, Baht me right Oe Oe a SSian, 50 Cn (many V rete ca hore the motion eatin tn pecetoos om the 416, Pots ie Nee the maton etn in Vain. “4B. Agee “> malk = 10 cm, = mig!Dk — 4.8 mi. Bi F< wi, = —02, where o Integrating these equations, with the initial eondslion® taken into scrote saa = la) (bcs th ola tn ence fe velo} yt = (ya), This the equation af circle of tadius vo with the denizen the potat my — yl Yo 4.43, Will increase VT (ATF times. It is taken into sceount here thatthe water {when i liguit phase) moves Wise, andthe system behaves as a mathematia! pendula. 14 we VE (2). 445, () P= Eyes 150) B= tama 0.08 3 448. on=eu)/ TemiPay Biel, Et ry “ign ae PL i ee OO ene 4.50, 0 VTuah + Ta > 4h. V3, Tun = 20 V Ue V 5. 452. T = x VORTR, Irae = WD. 4.58. w= V Hoa. 4.55, y= V GH TH, 4.55. = V GET 4.56. T=2n V (R= rhiae. tS roto LULL LL sh Gag ar ‘00, Peony Tithe Ps. 1st, oyese Toman 18, whee mo and me ae he nse a igen ad caer seme . 4.62. o= SV ByppimVo, whore 7 1s the adiabatic exponent. 4.63. q= 4h V Reymg (1) =2.0 pC. LL 4.65. 2= (0a) in of, where @=tAV aT. a ——L—r— 4.87. (a) a and ayo; (b) tym (atctan ina) , where Lrt—s——S—sC—CizsSC—S ab (tin SB Fan) we nes ms when 5,0, 660. wath, an ma ° {Tie tie 2Ze ls |VIFGOF, amarctan(—fio), with —a/2 my apd B11 dhe phase abruptly changes by © at the inter~ face. Im, & mth phase jump does not cecu 5.79, d=") (14+ 28) VAD, = O14 + 2k) mn, where a0, 5.80. dug 0.65 jam S.8t. d= Hk (1424) /Vi, where k=O, 1, 2 an 5.82, d= 1 LOE 15 ur, nyse 58. @) On Vamar= BS 9) 2 Tah Sr, =) ATTRA mm, BAR PV FER (E=TRR= 3.5 mem, where ka BAO 1 Hh (dl a/R (hy) = OD ym, where hy and 1, totic simi e dork age " Seas ke Noe ME = 24D, whee kis the unter of the Sgt Pag. BL (8) roe VIR TVD—3.8 mon, whee b= 10; (9) = V5 3.0 am, ‘wher mys the relive Inder of wale San! p= Vin aU Tag 8 mn, whee F Bi how = tty — A= 35h Hae candlla'iom'ean harp patter to another ecu itnfellowng condition fet = 1h = my, har & is « carthin itor. Th comespending deleeomcnt Sh Uf the mirror ie determined from the equation 24K°=- hy. Pros these iee'equations et V(b) AMA w Anil = 0.014 5.95, (a) The condition for maxims: 24 e088 = fy hence, the order of Inorforoncs & diminishes es tho angle 8, i. the radius of the rings, ineveases (eo Fig. 3.18). (b) Difeenting both sides of the foregeing equation and taking into acount that on transition rom one maximum to another the value of F changes BY unity, we obtain 60.~ Hild sin 0; this shows that the angular width of the fringes decreases with an inessgs of the angle 6, Le. with a decrease In the order of Interference. 5.90.0) kane = 247, = 1.040% () Sh-= Mi sat. tmshe rear 132d 5 pm, sa b= wie — a 5.0 0m. <8) (at H/%b = 0.60 pm. £400. (9 Fis aint Sale P= re 5.101. (0) 1 Oi) Ts Fo 2: (0) feta, Bet da Lym thal tym bP = PR TS thates eh Tet = Shale Tem eT (1 +"p!25) Fy. Here "p 1 tho angle’ covered “by ‘the S108, (a) = 2 (k 28)/in — 1) = 1205718) jm, (0) he 12k oF 1,2, Ge MAKn — 1), where =O, A vo "wh e+ 5/8) (mn — 1) = 2.5 pe, where = 2, TPO =0.90 VF mm, where k= 1, 3, 5, bint = 10m, Y'% yola = 8 mm; (0) mia ~ BEAD (a + B) = 1.2 (k + 318) pm: (Oh = Ge $id) yams Here B= “aa, Oh, abl(a + 8) = 0.8 m. This value corresponds to the veal point, apart Stom which there ate other point Here Tlic G) Totally 2 1.7, 1= 2 (AH b(64 — 0) w= O°T un, rhe l'on a the corresponding values ofthe ets tng Cnn oie 1 Tanty age, 28. Bats. aa oy the eomeaponding” vais of the sais hw hte Syn — tyr where BO. 12, SUS. 1, 18. Bate. PSS, Bae hil That 4:7. foes nk ayaX, where (ab) sn Bala = Bh, “13, ‘5.120: The condition for « maximum leads to the transcendental coquiifon tan =a where a (xbit) sin 8. The solution of this ‘uation (by teens of plotting or selection) provides the following ut value: y= 18a, 8) =2-46n, ay — Sata. Hence b sin 8, = aie bain 8, =2.90h, b sin, — 3.47). 52h. bein 8 = sin 0) = 1 for snglea 0 ace equal ta 3 and 27 respectively “wo “bt and k= 1 the 5.422, (a) 80 = aresin (n sin €) — 0 = 7.9% (b) from the com dition (sin 8, —nsin 0) = eA we obtain 0 = 8), 0 = = Ts, BABS. Re (= a) dR ~ 0.6 pm. Bul2s. 80% 3128. d= 2.8 pe 54m. oa bs asin yy S— Tamas. 0.54 jm. 52s. @) (oy-04" 29, 2 2Ri(n—1) V AA 5.180, From the condition. (ns obtain Oy = ABS, Oy = 0% Be Fig. 0. SASH. yk — U2Vle— 1), whee ae thay = hfe = 1.5 km 138, Bach nar produsas ts own didraction pattern in the njctive's focal Hands wih her cart asin Being operated i + Oh th we c + 78.5". See ket 2 Fig. bility occurs: the maxima of one system of minima ef the other eystern. Thun, the formals sin 8 = Ald we obtain y= 2/24 my 0.00". 5.184, (@) D=hid VI—(Pa}e= 6.8 ang. minjnm, where k= 2s () Deg VI (Eman) = 13 ang. minjam, where k=. 5.485, a0/dh = (ean OV 5.186. 80 20a’ 5.190. 8 76". 5.440. (0 nh fourth order: (0) fheun = 1 = 7 Sua 0.05 mm: (b) Tm 6a 55142, (3} 6 and 12 um: (0) not in the Brat onder, yea in the se- cond oder. a $418. According to Rayleigh’ exterion the maximum of the line ‘of wevelength i must coincide with the at miaimam ofthe Tine of vavotenath 8c Lat ue write both condsions forthe lest Geviation angle laters of tho optical. path iferenees for the seme ray (oe Pigs 828): = (OC + CE) =O, bln + 81) — DC + CB =A 4 Be Hao, bn ~ What follows is abyious Sette (a) aon = 206i 2-10" and 0.85-40¢ (4.0 ems 348, Shoot 20". BAM, RTOS hag 4 om SAAD. About 30m 5148, Suppose end Oy are the mitinum engulr separations rasorad ‘by the. tclesape’ objective andthe eye cspecively Gay a 12800, ay 1 32Hd) Then the sought agitation te scecope iTase = Ay Tag = bids 1. i dnue = O.iMTain = 1's uo, Stn Sova dau the mio, eparation sand by the ihe abea ‘AY fete mince singular separtion resnived by the aye FLED Ble, te suet mngttstinn of ntodope ‘Nia 2 ay ln ew m8. 5.5 bo to, oP and 15 S4g2 gO im, 6 S41 ni, 5.183, Suppose ‘Fate the angles between the dicetion to the dieatlon mains a te dtsetios of the srrey along the pious a,b aad respoetivly. Thon the valune ofthese angles ean Betoun irom ‘ue ollowing ‘conditions: a (1 oo a) hy Bros fm Hay and ooh y= Fh Recalling that conte ost ety SR ee obtain = Gar sash a= 2 tVEene br, whi 2, mi the ats ot 0 Nach ae S188. d= phony VTFESEGR GATE 0.28 pm, where ‘ey and hy are the orders al elton, Sulg6. rm Ian 24 ~ 3.5 cm, where a is the glancing ang found from the condition 2d sla a! = i S50. I Suse. 5.50 5.160 S01. Ll = ct = 0 502 Tat Ie = BUM = P) = 08. 5.169. P om (n — ACL ~ 9 c08 2) = 0.8 UGK, (a) La urteprecent the antural light ag gum of two mutual- ty perpendicular components with Intensities Ty. Suppose that each polusier transite In its plane the fraction Gof the lightwith {Sselation plane parallel to the polasiner's plano, and fhe faction {2 with oscillation plane. perpendicular to" the polarier's plane. ‘heinenty of Tigh texted cough th yen of wo Pl Titers fe then Iya ile + oily ‘when thei planes are parallel, and to Ty = aitale + exeilo ‘hen this planes ae perpendicular; acconing to the condition, it, =. ‘Gn the other handy the degre of polarization produced separately by each polarizers (agile, + a) ing ty and fom these equations, we Pe Va H 0.005. 0.85. Intensity variations of both beams in the Elim ) Payi 5.185. The relat cases Avand Dane (AID, = 4 ot (WD-B9, (AZ) a Hence 4 tan (92}-69, = (ALIN ATID)» = cot? (92, = 15". 5.168, O0. a) 0 = tot — a 4A = 0.074 (b) P= pi(t—p) = CER REE 0.080, Here mis the refractive Index of glass 5.168. T= I, (1 — pin = 0.72, whore n is tho refractive indy of ater ‘.168, p= ((nt — /(nt + 1) sinty = 0.098, where nm is the 2 (0 oat S71, (9 tn at Enis of ie eh sucnar tiie pn eagaal sop gt = Hla re andere eT pee alone tl 0 as Ene ee 046, whee pi the cos: who elette ector sls te ieuone ne nt 1, ive index of glass; (2) 0.48, 0.81, 067, and 0.2 ret tpeetively (n= 39 5.73. (2) 9 $2" 20.080; (0) A0@ = 1 — = (1) 0.94 here 1 is the mumbor of lenses. 5.179. (a) 0.8; (b) 0.044 o Veo o Pin righthanded system of eoordinetes: (f cirenlar anticlockwise polarization, ‘when obeerved. toward the Incoming waves (@) elliptical clockwise polarastion, when observed toward. the incoming. wave: the major axis of the ellipse colucides Se ee } plane polasization, along the stralght ine y — — S48. )'0.400 am (yO mm” 55480, 1 Addn/@h +1); 0.58, 0.5 and 0, st Bn 15,16 and 17 5.181. Four S.182. 0.69 and 0.43 ym. 5.185: d= (r= 12) fin ~ 0.95 mm, wnere k = 4 Jods = d.00, lnote the intensity of tragsmitted light by I, cise ofthe cromed Poarlds and by yi the tase of ib um respectively Ty = Hal sint 2g? (62, Ty = Hag [1 — sin® ges (6/2). The conditions for the maximum aod the minimom: Jade Ian i ote 1 | 21, for any @ SS uHin ear ere is the optical path dilfoence for the ordinary and extreor- dinary rays, B= 0, ty 2, 5.187. (a) The light with righthand cizeulat polarization fom ‘the observer's viewpoint) becomes plane polarned on passing trough A quarter-wave plate. Tn thls eas the direction of oscillations of thovalecrie vector of the electromagnetic wave forms an angie of “45° with the axis 00" of the crystal (Fig. 4a); in the cas of Hand polatiation this angle wil be oqual to 45" (Fig. 434) (0) Hl for any poaition of th plate the roction of the Polaroid (Qocated Batiind the plate) dows fot bring about any Vatltion ia the intensity of the Cranemitd Hight, the Gaiial Tight i natural: o 7) Fi 8. if tho intensity of the trensmitted light varies and drops to 200, the Initial light ie elreulany polarized; if Wt varies but docs aot Arop to zero, then the initial light is composed of natural and cir- culany polarized’ Tight BABB (a) Az = Hd (ny — me) = T2 (0 ~ n,) Ob <0. B.t80, Gn'= aha = 0.74-10-, where a fe the rotational con- stant 13.190, c= a/Az tan = 2a whore isthe dstencs from the 5.191. date — (H/o) areata YH ‘Sitge! Yam 51193. [al 7 BABE (2) En ruptions pee second 5.195. An = 2eH V7, where ¢ isthe velocity of light in vacuum. B98. Vo (gy — gall = D018 ang. min/A. 5.47 toe fnew tranumie Bam acount the positive direction clockwise, then = (@— VE) ly where {he number of times the beam pases through the substanct (In, Fig, 34 the number is N=). ‘5.198, Hoya = A160! = 40 KA/m, where V is the Verdet con- stant. The ditection slong which the’ ligt is transmitted. changes tw the opposite. ‘3199. f= meugh! — 42hours, hough the elect is very small, was observed bath for visible ight and for SHP radiation. 0) atin Ag.tmm, (2) ~ os (xz!) . dogl(ém-glem?). IV TBI = 10.86 Vien: (b) 2.2-40" intere 5.200. (a) a= eFimot =5-10- em, where Ey Vie, v= = 1.7 exis; (b) F/R 2.9:10"%, S201 (9) et natin, ve TH Ree RETT, 5.202. n 5.303. the sincentzation of elton Inchon S200" (2 Sa ean (ot bps wire 2 and gan died by the formal : = ett una a, VaR oa Here Bm, atm, m is the mass ofan elton. () «P= viata aa, Ponar= ( 22)? for ate ora 5 e 5.205. Lat us write the wave equstion in th form A = Agt=M, where km Bake IE an bey then = (hyn and A= Aypbetneliet-ton, fo in the real form a Le the light propagates as a plane wave whose amplitude depends a fon z When se 0, the amplitude dimnnishes (the stienustion of the Wave dye to absorption). When n! = fx, then le =c0s at —K'2), An der ‘his Iss standing wave whose amplitude decrosen exponentially (i 2 dn hin ete the light eupeinte ft intra efeten tn tarmealum (ithoat asopion B06. ng SatenIelg = 20°10 ea 208 (0) a = tees ya = su = Nm Same, et + Mlat whore fea constant 5.210,» = elm = LAA tls wh (i) (nl) ln Posto a S211 1 a tlllan to discus thre hnnie components of tue za of waves (erent ith the nlp a S212 T Stas sint 9, whee @ = Mle PAR Td Th tt ae PL Ps te iad oR Fett ad = Wert = pay 5.215. x 084 en, 5.216, v= (1 — pl exp tl (ey + 2) Ue Tm Iy(h—9) She Bo Tai Im age 0 Yl decease exp (a) = 0.640" tine 2-03 am. 2= (a 2yu 8 mm. N= (ayy? 56. Feceiver. Suppose that the source emits short pulses withthe fater= vals Ty. Then in the eefezenceTreme hed tothe receiver the distance between tno succissive pulses Is equal to he?) — 0,7 mien measured slong tho observation line. Hore 0, Is the projection af the SDurce seloety onthe obmerstion las ( St veceivd pulses y= 2k (ae (06 cos 6 <4 V Bact cos 0 = —26 am, cB = 2 days, whore Fis the cadius of the 25d — (Annet = BAP hah m = (AN IAIDR = ‘where"y is the gravitational constant. Samet ae EO nate wot + 2), 5.080. "v = shaw ~ 900 lam per hous, 5.204, Substituting the expressions for! and 2" (fom the Locents teanafortiation) into. the equation af — fz = it — Kz, we brain oa 8) The frequency sult vd), whore ve-= A/T. Hence Paving ow OT kek (+ BYVTS RR, ‘where 6 = Vile, Horo it is taken into assount that a” = el’. 5.232. From the formula of =a V—BVG+B) we get B= sole 1.98 32st ome Bat, 5.284, = 0 V7 5.285. Ak S"hTimaet = 0.70 nm, where my is the mass of the whore d= d VT — (IF. The corresponding “natural” frequency Js vf = ld", Duo tothe Doppler effect the observed frequency fs Vim nw ver igen Tea ‘The coresponding wavelenath ej, — ey — dole — cos). When oe Babee the wavdlngth ih © 06 ph 525. a) Lat op be the projection ofthe Seacty vector of th radinting stom on fw obmevation retin, The meaber of atoms i projections falling witha th interval bay abe 1) ~04p (—muDHDY ey ‘The fequeny of light smite By the stom moving with velocity es om oy (hs From the expreson the fequney ase Sion of atoms canbe loand- no) do"= 3) don Aad nally st thould be takes ino acount thatthe seer edison intensity Town (oe ) Seog 2 V Ele eT a2. un EY Ive, then wwe eV (1a 5.210, = Ny = 99 kk 53 pe S248 The ri ioduond by « chaesed particle moving with selocty V excites the atoms ofthe medium turning them into surest STEM "woven Le un codes to ‘rbiteery ‘points and along the ? Path of the particle, The Tight waves fitted from these. points whe the By the liad wave to ropag ‘ , {te points to the point Cis equ a fo thet taken by to particle toy : = cha te tho phase velocity of ht sey itn nose. 3 pV Tet, O14 MOY and 08 GF sta i ade San, noun ha paeetien SBE ING 105 yn Sou rpAaaM 2 1g'R" vine Wh eae co al eat a ed, ue ible only it ¥ >, Le, when. [the phase velocity of light ia 4) met = 0,28 Mev 75 ek. S251. t= (P= 0 epdltSo7} = 9 hours, wha & is the spe cite eat capaty of enpper, psa dency. $22. Tee, VB mOS kK 5.28 (i) Cy = (BUID)y a8 oTWV/e—e3 ADK, whose = = ASPViee 0) 'S — 4borVide = 1.0 aHiK 525k (a) Opp = Tle = TAB AG" 35) yp = DaealsT = 10 me 5.280. (0) ta (KPIa869 0%) y= late) oberon, 3:26, y= SS aaa th 257. ap = Saiderani lemons) = 01 Wiemt, shoe bi the constant in Wiens diaplscoment law. BEE) 14 um 0) OT CO PAP, = VIO A — yaa) = 5.289. ngdo= six gantay+ OM 5.280, (9) ) = Palit? 640 om () rm Paitaige = 9 Saat plan 8c 5282. (p= 4(t + 6) Bletes 5250 atm 5.263, p=(Ele) V+ pF} Spcosdb— 35 aN.s, 5.264 p = fla + pow = 08 Nem Sins, P= allie ~ 048 WX 9.260, F = Pde (t + nf. 267. () dp 22 VISE, @) apm 2 Le, Hw B= vie. Te is evident iat in tho reference Trobe fixe tthe mite the later bin the Smaller omentum Sane sin (02) ~ Bime VF, 0-05 2280, Avloy= he) <0, he tin ito dear slic y Sake — Wien Sai, Vo ahledsina 31 BV. 5.272. huen = 2a/me{y—1)=2.8 pm, where y=1/V4 S218, Hom, 66-10 mle 5.274, A= Inch We Path. the frequency of 5.20. 5.278. 2 (a, + @) — A, = 038 ov. S277. w= Pachdleh = 0.020. BI2TB. fag = 64-10" ml 279, 0°3 Vi: the polarity of the contact potential diference is opposite to that of axtarnal. voltage ‘3.280. hime the Compton wavelength forthe given particle, 5.281, Let us write the energy and momentum conservation laws ln the esference framo eed to the cleceon for the moment preceding the eolisian withthe photon: Ma my = me, Male moy where m= V1— (le. From this it follows thet v= 0 or ve ‘The results have no physical meaning 282 (Tigh scatrnd byt to atone (icra of the number of electrons that turn fee (the Tree electrons hase the Binding energy much lower than the esergs transerred to them by the phatons) (c) te presenes of a non-displacad component is Je 1 scattering by the stongly bound electrons and the nucle 5.288. Am fate [sin (05.2) — sin 204 — 1) = 1.2 pm. BBL T= Raw bx) = 0.30 MeV. 5.285, (a) 0" — Brel} 2an/me re by, 5.288, fa! — Tre aT MeV 5.287. sin (02) me(p—pyORpr. Hence O= 120", 5.288. ho [1+ VT 2meP sink }] 712 =0.68 MeV. 5.289, = Qahime) (VC ITT ee 1) = 87 pm 2.9.10 rads; 5.201, » =BUtb ee 5.4 om, 5.292, ah — (Uninc) an? (0/2) = 4.2 pm Gil. 7 = STF = 0.15 um, k= Qnele) Vin = 0.24 wm. 82. b= 073 pm 83, (rnin 0.39 pm: B) Fun = (227) CL + male Eat ‘ai (Ze1T) cot (012) = 0.23 pms (0) Fain = 8.4. () Pain =F cosee ian 26/7 = 0.58 85 px 2V BATH TZA 88, Te mempto de en) ea whee ne BE fos =H: () AP — shasta Oa: (0) P ~ YS 3-102 B10. d= dense sink (0/2) coneeattation of cll 5 ym, where m is the 045, aNyw = 28 (07 40.9.2) pagent = 1.440%, he Zand tare she omic neta, of copper and ic Brand hy ae heir mole sae, ‘Na ts Avogadro's number ta. Ab ee 2A cg) O78 CAE (09 Novy ty dah Bork nt 7D 818 PL Gingeah ing = Sa 849. 1S mill 1p ino, By = nko, where n= 1% ..n 0 = 1. kbp. tia = neh pale Bo mivich ~ 135 2. ‘The Brackett series, Jo=y = 2.08 ym. (a) 057, 487 and 480 not) HBR 15-10% For nib fisinO = abit, whence 6 = Wa thn ow — 9) 97.8, 1028 ond 124.6 nm, wea elm, th = Bo 2207-408 =, Zo) (HUTT)RTRSE 3, 15 898, hm (aeida)(@) Fido — 1102 V Rho —t) 07 pm Ce br OSU he te S38 Bes aa ey. 830. v—V2iho— AAA 23-10 mis, where @— 220 Fenn eS At: ¢2"skRme 328 mls whoce mie the mas of the fe ceeeeeeree 5B BBEBERSEEE 2 (= em SARIBnc = O.SSA0-*%, haem the ras of the stom BAB, = 2V Rn the electron, Gil 9 ARAN oo = OTA me 8.45, (o) By — ne !In, (by Ey WRI, (0) By = 5 () By me. 11-10" o/s, where mis the mass of 6.40. B, = w/2h*, 1 — uet!29°, where 4 is the reduced mass oft sein: motion of he nc att taken ato secant, thee values fn the ease of « hydrogen atom) are grvcter by iM = Sy where m and Mare the masses of an eletton and 3 9- ‘647. Ep — By = 3.7 meV, dy — hy = 83. pm. 5.48. 2) 0.289 pm, 2.53 keV, 065 amy (b)'100 po, 6.8 «Vy 0.248 um, .48."123, 2.85 and 0.485 pm. B50, 0.48 ev 54, For both particles = 28 (L 4 meimg/V Tag? = 8.6 pm. 6.52, To Dah V REE. 8.58. = 20h Dak 128 po. SSK, Fin, lot us fd the Uitbution of mlecues over de Broglie wavelensthe Prom the relation f (@) do = 9 (0) where {this Meswells darbution of vlotties we eta 90) = ANA, gn DRY Wk Tha condition dpidheO provides tyr ah/V EF~0.09 am, 885. i= 20NV RTC FTDA, P< 4mchahii 20 SteV (or an elec) and 97-5 Mev (or pots) 86, 7 ~(V'F-1) me 0.21 Mev wV Tenet lah 3.3 pm, 858 bila to ae 630, Ae-—2nhlld V Ome? 4.9 pm S80. j D5 kev. V 2A os 0/2) =0.21 na, where #4. 6.02, d= xRA/V InP sin 8 =0.23 0.04 0m, where k=3 and the angle 9s determined bythe formula tan 30 D/2t 6.63. (3) n= VET = 1.05; (b) V/V, t 2-40) ~ 50. 6.06. Ey — nnchilanl, whore m= 1, 2, 140%, "140 and 4-10" om Br. Ax Alm! = 1-40" mis: v, 8.09. 80 5 ye 10-8 S10: Tin VR VDn = eV Here we assed thet p = Ap and Ar = ‘811, Avie ~ All InP B22. FS iin 73, Taking favo scout that p~ 4p ~ Alda ~ ls eet 2.406 mi 10-4 B= T+ Uw tims! Ie), From the sondition dias — we find zp and then Eng ~ AV Ei = ho, where « Is the osellat oF's angular frequency. ‘The rigorous calculations furnish the valu hala 7 874, Taking ta acoust shat, p ~49 ~Aldr and Ar. welt Ba pitin = cir dae ei? Fm the eon AE lée = 0 we Bnd ray Mine! = 58 pia, Engg = —met ase Bb, The width of the image is Am 5-4 A ~ 8 + ps, sxhere Alen additional widening atociated withthe uncertainty {the momentum py (wien the hydrogen atoms pase though the Sl), pis the momentam of the Ineent hydcogen toms. The ane: Aion A(8) has the ininum when 8 VAT ‘3.0, "The aoltion ath Set ma the til equation ith sue nt sparati fe? remus ino equations The aoutions att y ©) eh swhoce = VSmBiQ, Bis the eouegy ofthe partite, and )() ~ a whose‘ = Ein. Finally, Waste, wtb a i 9 0 877. P=ts+VIi2a~ 001 fre pat Aeebiaaidy th m=1, 8,3, oy 9 ( Aentma Hnad, 68 Here A= 7 6.80, ANGE — Glam VRB, Mt = 1 ¥, then age =0,8=10" lavels pe eV. 6.81, (a) Ins ease’ the Sehriding equation takes the form PEM a nga0, Meter, Lt a ake th origin of cordate atone ofthe corners of the wal“ sds of the gull he faction bpm tee ne ‘eo acaring tthe condo) an theres Is evenant to ‘51 ts Tontln iste te wall isthe foe tae ese ee Shin hg ice on the two ides (2 =O nad J 0) y= Osutome: tically. The poseble value of fy sod By ere found fom the condi Hon l'g arn int laren hopes sides of he wel WU =O, Km eG) my ma HDI ees P10 ky (allay mate 2 8. The substitution of the wave function into the SchrSdingr tion Teads tothe relation tf-+ fs whence : Ey = (IE nfl) 2282m, (0) 9.87, 2.7, 30.5, and 404 unite of An 6.82, = 113 — Sig = 195% 88, (@) Bm Got fae © mi REG, wher my may my ae nlegers nat equal to'z0r0: (b) AE — ah'/ma; (0) for'the beth level non tnt 1 apd B= tana the numberof star equal six Gi io ogual tthe nomber of peutatons of steed £203) rast a 84, Tot us integrate the SehrSdiagr equation over 8 smal ity nie Pet estinas 2 within whieh ew t's icon EST foc ekample athe pownts oF 4.) 2(-9~ | Beyer Since the discontinuity U is into the integral tonds to rao as Bor 0. What follows ts obvious, “ets. (a) Let us write the Schrodinger equation for two regions ocz By (my + molng = 3.3 MeV. S273. Between 1.89 and 206 MeV, ERA. Om hig Tam 07 My, 0221s 158 and 87a enpectivey, Se. AE Mv, 53.0 al 8270, Pam AS [mom T= 27] 2.2 MeV, where smo, may may my ie Hh masses of Bouin, & C noses, an sip par,‘and a Bot avcles, 4.218. By By2me 0.06%, wher m isthe mas of deen, 6219, £0 tar ~ 6.5 MeV 6.280, Ey= By + PE —T)= 167, 10, 17.5 and 17.7 MeV, vrhero Bs is the binding Grey of « deuteron the transitional C281. © = (AFLVp d) In n= 2.5 kb, where Af fs the molar ass of cadmium, 17 Is the Avogadto number, pis the density of cadtstum Tail = exp (ar toy) nal = Bd, where 8 the conc wt Sy aloft wnt eap te + 09 nl) oon +0. worn iets encatitiog of Feat) Seih ob) w= deen od! = 085, 6.387. J = aa 1 in = 6.40" part. emt: 4s the decay constant» Ny fe the momber of Av duel ‘288 et = '.3.408, where fis the number of geaere 08:40" 10% (b) T= we — 1) = 40 6.201: 0.05, 0, and 9 GeV respectively. 6.292, (meq VTFD=15 m. 6.208. ty = Ine FOP Fm = mass of & pion. 6.204, JJu

You might also like